You are on page 1of 336

Problems

in
Matrix Calculus

by
Yorick Hardy
Department of Mathematical Sciences
at
University of South Africa

Willi-Hans Steeb
International School for Scientific Computing
at
University of Johannesburg, South Africa
Preface

The purpose of this book is to supply a collection of problems in matrix calculus.

Prescribed books for problems.

1) Matrix Calculus and Kronecker Product:


A Pratcial Approach to Linear and Multilinear Algebra, 2nd edition

by Willi-Hans Steeb and Yorick Hardy


World Scientific Publishing, Singapore 2011
ISBN 978 981 4335 31 7
http://www.worldscibooks.com/mathematics/8030.html

2) Problems and Solutions in Introductory and Advanced Matrix Calculus

by Willi-Hans Steeb
World Scientific Publishing, Singapore 2006
ISBN 981 256 916 2
http://www.worldscibooks.com/mathematics/6202.html

3) Continous Symmetries, Lie Algebras, Differential Equations and Computer


Algebra, second edition

by Willi-Hans Steeb
World Scientific Publishing, Singapore 2007
ISBN 981-256-916-2
http://www.worldscibooks.com/physics/6515.html

4) Problems and Solutions in Quantum Computing and Quantum Information,


second edition

by Willi-Hans Steeb and Yorick Hardy


World Scientific, Singapore, 2006
ISBN 981-256-916-2
http://www.worldscibooks.com/physics/6077.html

v
The International School for Scientific Computing (ISSC) provides certificate
courses for this subject. Please contact the author if you want to do this course
or other courses of the ISSC.

e-mail addresses of the author:

steebwilli@gmail.com
steeb_wh@yahoo.com
Home page of the author:

http://issc.uj.ac.za

vi
vii
Contents

Preface v

Notation xi

1 Basic Operations 1

2 Linear Equations 30

3 Traces, Determinants and Hyperdeterminants 38

4 Eigenvalues and Eigenvectors 65

5 Commutators and Anticommutators 120

6 Decomposition of Matrices 133

7 Functions of Matrices 137

8 Cayley-Hamilton Theorem 165

9 Linear Differential Equations 168

10 Norms and Scalar Products 172

11 Graphs and Matrices 179

12 Hadamard Product 182

13 Unitary Matrices 187

14 Numerical Methods 207

15 Binary Matrices 212

16 Groups 215

viii
17 Lie Groups 232

18 Lie Algebras 242

19 Inequalities 254

20 Braid Group 256

21 vec Operator 266

22 Star Product 269

23 Nonnormal Matrices 275

24 Spectral Theorem 283

25 Mutually Unbiased Bases 289

26 Integration 292

27 Differentiation 294

28 Hilbert Spaces 299

29 Miscellaneous 301

Bibliography 321

Index 326

ix
xi

Notation

:= is defined as
belongs to (a set)

/ does not belong to (a set)


intersection of sets
union of sets
empty set
N set of natural numbers
N0 set of natural numbers including 0
Z set of integers
Q set of rational numbers
R set of real numbers
R+ set of nonnegative real numbers
C set of complex numbers
Rn n-dimensional Euclidean space
space of column vectors with n real components
Cn n-dimensional complex linear space
space of column vectors with n complex components
H Hilbert
space
i 1
<z real part of the complex number z
=z imaginary part of the complex number z
|z| modulus of complex number z
|x + iy| = (x2 + y 2 )1/2 , x, y R
T S subset T of set S
ST the intersection of the sets S and T
ST the union of the sets S and T
f (S) image of set S under mapping f
f g composition of two mappings (f g)(x) = f (g(x))
x column vector in Cn
xT transpose of x (row vector)
0 zero (column) vector
k.k norm
x y x y scalar product (inner product) in Cn
xy vector product in R3
A, B, C m n matrices
det(A) determinant of a square matrix A
tr(A) trace of a square matrix A
rank(A) rank of matrix A
xii

AT transpose of matrix A
A conjugate of matrix A
A conjugate transpose of matrix A
A conjugate transpose of matrix A
(notation used in physics)
A1 inverse of square matrix A (if it exists)
In n n unit matrix
I unit operator
0n n n zero matrix
AB matrix product of m n matrix A
and n p matrix B
AB Hadamard product (entry-wise product)
of m n matrices A and B
[A, B] := AB BA commutator for square matrices A and B
[A, B]+ := AB + BA anticommutator for square matrices A and B
AB Kronecker product of matrices A and B
AB Direct sum of matrices A and B
jk Kronecker delta with jk = 1 for j = k
and jk = 0 for j 6= k
eigenvalue
 real parameter
t time variable
H Hamilton operator

The Pauli spin matrices are used extensively in the book. They are given by
     
0 1 0 i 1 0
1 := , 2 := , 3 := .
1 0 i 0 0 1

In some cases we will also use x , y and z to denote 1 , 2 and 3 .


Chapter 1

Basic Operations

Problem 1. Let e1 , e2 , e3 be the standard basis in R3



1 0 0
e1 = 0 , e2 = 1 , e3 = 0 .
0 0 1

(i) Consider the normalized vectors

1 1
a = (e1 + e2 + e3 ), b = (e1 e2 + e3 ),
3 3

1 1
c = (e1 + e2 e3 ), d = (e1 e2 e3 ) .
3 3
These vectors are the unit vectors giving the direction of the four bonds of an
atom in the diamond lattice. Show that the four vectors are linearly dependent.
(ii) Find the scalar products aT b, bT c, cT d, dT a. Discuss.

Problem 2. Consider the 4 4 matrix A and the vector b in R4

0 1 0 0 1

1 0 1 0 1
A= , b = .
0 1 0 1 1
0 0 1 0 1

(i) Are the vectors in R4 b, Ab, A2 b, A3 b linearly independent?

1
2 Problems and Solutions

(ii) Show that the matrix A is invertible. Look at the column vectors of the
matrix A
0 1 0 0

1 0 1 0
, , , .
0 1 0 1
0 0 1 0
Find the inverse of A.

Problem 3. Consider the normalized vector v in R3 and the permutation


matrix P , respectively

1 0 1 0
1
v= 1 , P = 0 0 1.
3 1 1 0 0

Are the vectors v, P v, P 2 v linearly independent?

Problem 4. (i) Consider the Hilbert space M2 (R) of the 2 2 matrices over
R. Show that the matrices
       
1 0 1 1 1 1 1 1
, , ,
0 0 0 0 1 0 1 1

are linearly independent.


(ii) Use the Gram-Schmidt orthonormalization technique to find an orthonormal
basis for M2 (R).

Problem 5. Consider the vector space of 2 2 matrices over R and the


matrices
       
1 1 0 1 0 0 1 0
A1 = , A2 = , A3 = , A4 = .
0 0 0 1 1 1 1 0

(i) Are these matrices linearly independent?


(ii) An n n matrix is called normal if M M = M M . Which of the matrices
Aj (j = 1, 2, 3, 4) are normal matrices?

Problem 6. Are the four 2 2 matrices


   
1 1 1 1 0 0
P = , Q=
2 0 0 2 1 1
   
1 1 1 1 0 0
R= , S=
2 0 0 2 1 1
linearly independent?
Basic Operations 3

Problem 7. Let 1 , 2 , 3 be the Pauli spin matrices and 0 = I2 . Consider


the vector space of 2 2 matrices with underlying field C. Show that the 2 2
matrices
1 1 1 1
I2 , 1 , 2 , 3
2 2 2 2
are linearly independent.

T
Problem 8. Consider the normalized vector v0 = ( 1 0 0) in R3 . Find
three normalized vectors v1 , v2 , v3 such that
3
X 1
vj = 0, vjT vk = (j 6= k).
j=0
3

Problem 9. (i) Find four normalized vectors a1 , a2 , a3 , a4 in R3 such that



4 1 1 for j = k
aTj ak = jk = .
3 3 1/3 for j 6= k
(ii) Calculate the vector and the matrix
4 4
X 3X
aj , aj aTj .
j=1
4 j=1

Discuss.

Problem 10. One can describe a tetrahedron in the vector space R3 by spec-
ifying vectors v1 , v2 , v3 , v4 normal to its faces with lengths equal to the faces
area. Give an example.

Problem 11. Find the set of all four (column) vectors u1 , u2 , v1 , v2 in R2


such that the following conditions are satisfied
v1T u2 = 0, v2T u1 = 0, v1T u1 = 1, v2T u2 = 1.

Problem 12. Let u, v be (column) vectors in Rn . What does


q
A = |(uT u)(vT v) (uT v)2 |

calculate?

Problem 13. Consider the 2 2 matrices


   
a11 a12 0 1
A= , C=
a12 a11 1 0
4 Problems and Solutions

where a11 , a12 R. Can the expression


A3 + 3AC(A + C) + C 3
be simplified for computation?

Problem 14. Let A, B be 2 2 matrices. Let AB = 02 and BA = 02 . Can


we conclude that at least one of the two matrices is the 2 2 zero matrix? Prove
or disprove.

Problem 15. Consider the four 2 2 matrices


   
1 0 0 1
A= , B= , A , B.
1 0 0 1
Do the matrices form a basis in the vector space of 2 2 matrices over R?

Problem 16. Let A, C be n n matrices over R. Let x, y, b, d be column


vectors in Rn . Write the system of equations
(A + iC)(x + iy) = (b + id)
as a 2n 2n set of real equations.

Problem 17. Let A, B be n n symmetric matrices over R. What is the


condition on A, B such that AB is symmetric?

Problem 18. Let A, B be positive definite matrices. Is AB also positive


definite? If not, what is the condition on A, B such that AB is positive definite.

Problem 19. Let m 1 and N 2. Assume that N > m. Let X be an


N m matrix over R such that X X = Im , where Im is the m m unit matrix.
(i) We define
P := XX .
Calculate P 2 , P and tr(P ).
(ii) Give an example for such a matrix X, where m = 1 and N = 2.

Problem 20. (i) Compute the matrix product



4 1 2 x1
( x1 x2 x3 ) 1 0 1 x2 .
2 1 0 x3
(ii) Write the quadratic polynomial
3x21 8x1 x2 + 2x22 + 6x1 x3 3x23
Basic Operations 5

in matrix form.

Problem 21. Given the 2 2 matrix A. Find all 2 2 matrices X such that

AX = XA.

Problem 22. Let A be an n n matrix over C and x Cn . Show that

1
<(x Ax) x (A + A )x.
2

Problem 23. Let A, B be normal n n matrices. Assume that AB = B A


and BA = A B.
(i) Show that their sum A + B is normal.
(ii) Show that their product AB is normal.

Problem 24. Let B be an n n hermitian matrix. Is iB skew-hermitian?

Problem 25. Let A be an n n normal matrix, i.e. AA = A A. Show that


ker(A) = ker(A ), where ker denotes the kernel.

Problem 26. Let A be an n n hermitian matrix. Show that Am is a


hermitian matrix for all m N.

Problem 27. Let A be a hermitian n n matrix and A 6= 0. Show that


Am 6= 0 for all m N.

Problem 28. An n n matrix is called normal if AA = A A. Obviously,


a hermitian matrix is normal. Give a 3 3 matrix which is normal but not
hermitian.

Problem 29. Let A be an n n matrix with A2 = 0n . Is the matrix In + A


invertible?

Problem 30. Let A be an n n matrix with A3 = 0. Show that In + A has


an inverse.

Problem 31. Let A, B be n n matrices and c a constant. Assume that the


inverses of (A cIn ) and (A + B cIn ) exist. Show that

(A cIn )1 B(A + B cIn )1 (A cIn )1 (A + B cIn )1 .


6 Problems and Solutions

Problem 32. Represent the 3 3 matrix



1 0 1
A= 0 2 0 (relative to the natural basis)
1 0 1

relative to the orthonormal basis



1 1 0 1
1
0, 1, 0 .
2 2 1
1 0

Problem 33. Consider the rotation matrix


 
cos() sin()
R() = .
sin() cos()

Let n be a positive integer. Calculate Rn ().

Problem 34. An n n matrix K is called a Cartan matrix if it satisfies the


following properties
(i) Kjj = 2 for j = 1, . . . , N .
(ii) Kjk is a nonpositive integer if j 6= k.
(iii) Kjk = 0 if and only if Kjk = 0.
(iv) K is positive definite, i.e. it has rank n.

Find a 2 2 Cartan matrix.

Problem 35. Let B, C be n n matrices and 0n the n n zero matrix.


Consider the 2n 2n matrix
 
0n B
A= .
C 0n

Find A2 .

Problem 36. Find a 2 2 matrix which is normal but not hermitian.

Problem 37. Consider the 2 2 matrix


 
1
A=
1

where , R. Find the condition on , such that the inverse matrix exists.
Find the inverse in this case.
Basic Operations 7

Problem 38. A 3 3 matrix over R is orthogonal if and only if the columns


of A form an orthogonal basis in R3 . Show that the matrix

3/3 0 6/3
3/3
2/2 6/6

3/3 2/2 6/6

is orthogonal.

Problem 39. Let


0 1 0 0 0 0 1 0

1 0 0 0 0 0 0 1
A= , B= .
0 0 0 1 1 0 0 0
0 0 1 0 0 1 0 0

Can one find a permutation matrix such that A = P BP T ?

Problem 40. Let A be an n n matrix. Then A can be written as A = HU ,


where H is an n n positive semi-definite matrix and U a unitary matrix. Show
that H 2 U = U H 2 if A is normal, i.e. A A = AA .

Problem 41. Can one find an orthogonal matrix over R such that
   
T 0 1 0 0
R R= ?
0 0 1 0

Problem 42. Let 0n be the n n zero matrix and In be the n n identity


matrix. Find an invertible 2n 2n matrix T such that
   
1 0n In 0n In
T T = .
In 0n In 0n

Problem 43. Find all 2 2 matrices g over C such that

det(g) = 1, g = g 1

where is the diagonal matrix = diag(1, 1).

Problem 44. The (n + 1) (n + 1) Hadamard matrix H(n) of any dimension


is generated recursively as follows
 
H(n 1) H(n 1)
H(n) =
H(n 1) H(n 1)
8 Problems and Solutions

where n = 1, 2, . . . and H(0) is the 1 1 matrix H(0) = (1).


(i) Find H(1), H(2), and H(3). Find the eigenvalues of H(1) and H(2).
(ii) Find the inverse of H(1), H(2) and H(3).

Problem 45. Let M be an 2n 2n matrix with n 1. Then M can be


written in block form  
A B
M=
C D
where A, B, C, D are nn matrices. Assume that M 1 exists and that the nn
matrix D is also nonsingular. Find M 1 using this condition.

Problem 46. Let A be an m n matrix with m n. Assume that A has


rank n. Show that there exists an m n matrix B such that the n n matrix
B A is nonsingular. The matrix B can be chosen such that B A = In .

Problem 47. Let A be an m n matrix over R. Let

x := (1, x, x2 , . . . , xm1 )T , y := (1, y, y 2 , . . . , y n1 )T .

Find the extrema of the function

p(x, y) = xT Ay.

Problem 48. Let A be an n n matrix over C. Let u, v Cn considered as


column vectors. Is
v Au = u A v ?

Problem 49. Let u, v be normalized (column) vectors in Cn . Let A be an


n n positive semidefinite matrix over C. Show that

(u v)(u Av) 0.

Problem 50. Let A be an arbitrary n n matrix over R. Can we conclude


that A2 is positive semi-definite?

Problem 51. Show that if hermitian matrices S and T are positive semi-
definite and commute (ST = T S), then their product ST is also positive semi-
definite. We have to show that

(ST u) u 0

for all u Cn .
Basic Operations 9

Problem 52. Let  [0, 1]. Show that the 2 2 matrix


 
  2
=
 2 1

is a projection matrix. What are the eigenvalues of . Be clever.

Problem 53. Let A Rmn be a nonzero matrix. Let x Rn , y Rm be


vectors such that c := yT Ax 6= 0. Show that the matrix

B := A c1 AxyT A

has rank exactly one less than the rank of A.

Problem 54. Let A, B be n n idempotent matrices. Show that A + B are


idempotent if and only if AB = BA = 0.

Problem 55. Let A, B be n n matrices over C. Assume that A + B is


invertible. Show that

(A + B)1 A = In (A + B)1 B
A(A + B)1 = In B(A + B)1 .

Problem 56. Let , C. What is the condition on , such that


 
0
A(, ) =
0

is a normal matrix?

Problem 57. Find all invertible 2 2 matrices S such that


       
0 1 0 0 0 0 0 1
S = S .
0 0 1 0 1 0 0 0

Problem 58. (i) Consider the two-dimensional Euclidean space and let e1 , e2
be the standard basis
   
1 0
e1 = , e2 = .
0 1

Consider the vectors



1 3 1 3
v0 = 0, v1 = e1 + e2 , v2 = e1 e2 .
2 2 2 2
10 Problems and Solutions

1 3 1 3
v3 = e1 + e2 , v4 = e1 e2 , v5 = e1 , v6 = e1 .
2 2 2 2
Find the distance between the vectors and select the vectors pairs with the
shortest distance.

Problem 59. Given four points xi , xj , xk , x` (pairwise different) in R2 . One


can define their cross-ratio
|xi xj ||xk x` |
rijk` := .
|xi x` ||xk xj |
Show that the cross-rations are invariant under conformal transformation.

Problem 60. Consider the vector space M2 (R) of 2 2 matrices over R. Can
one find a basis of M2 (R) such that all four matrices are normal and invertible?

Problem 61. Find all 2 2 matrices A over C such that A2 = I2 , A = A.


Extend to 3 3 matrices.

Problem 62. Let a, b R and a 6= 0. Find the inverse of the transformation


 0   
x a b x
= .
1 0 1 1

Problem 63. Consider the 4 4 matrix


cosh 0 0 sinh

sin sinh cos 0 sin cosh
A(, , ) = .
sin cos sinh sin sin cos sin cos cosh
cos cos sinh cos sin sin cos cos cosh
(i) Is each column a normalized vector in R4 ?
(ii) Calculate the scalar product between the column vectors. Discuss.

Problem 64. (i) Consider the 3 3 permutation matrix



0 1 0
P = 0 0 1.
1 0 0
Find all 3 3 matrices A such that P AP T = A.
(ii) Consider the 4 4 permutation matrix
0 1 0 0

0 0 1 0
P = .
0 0 0 1
1 0 0 0
Basic Operations 11

Find all 4 4 matrices A such that P AP T = A.

Problem 65. Let 0 < /4. Note that sec(x) := 1/ cos(x). Consider the
matrix  
sec(2) i tan(2)
A() = .
i tan(2) sec(2)
Is the matrix hermitian? Is the matrix orthogonal? Is the matrix unitary? Is
the inverse of A() given by A()?

Problem 66. Are the 4 4 matrices



1 1/ 2 0 1/ 2

1 1/ 2 1 1/ 2 0
P = , Pe = I4 P
2 0 1/ 2 1 1/ 2
1/ 2 0 1/ 2 1

projection matrices? If so describe the subspaces of R4 they project into.

Problem 67. Let A be a positive definite n n matrix over R. Let x Rn .


Show that A + xxT is positive definite.

Problem 68. Let A be a positive definite n n matrix over R. Let x Rn .


Show that A + xxT is positive definite.

Problem 69. Let A be an n n matrix over R. Assume that A2 = 0n . Find


the inverse of In + iA.

Problem 70. Write the matrix


 
1 1
0 0

as a linear combination of the Pauli spin matrices and the 2 2 identity matrix.

Problem 71. Consider the 2 2 matrices


   
r t 1 1 1
S= , R= .
t r 2 1 1

Calculate RSRT . Discuss.

Problem 72. The vectors u, v, w point to the vertices of a equilateral triangle


     
1/ 3 1/(2 3) 1/(2 3)
u= , v= , w= .
0 1/2 1/2
12 Problems and Solutions

Find the area of this triangle.

Problem 73. Consider the 4 4 matrix

1 1 1

1 1 1
A= .
1 1 1
1 1 1

Find the 4-th column non-zero vector in the matrix A so that this vector is
orthogonal to each of three other column vectors of the matrix.

Problem 74. Assume that two planes in R3 given by

kx1 + `x2 + mx3 + n = 0, k 0 x1 + `0 x2 + m0 x3 + n0 = 0

be the mirror images with respect to a third plane in R3 given by

ax1 + bx2 + cx3 + d = 0.

Show that
0 2
k a b2 c2 2ab 2ac k
`0 = 1 2ab a2 + b2 c2 2bc ` .
a2 + b2 + c2
m0 2ac 2bc a2 b2 + c2 m

Problem 75. Consider


0 1 0 0 1

1 0 1 0 1 1
A= , v0 = .
0 1 0 1 2 1
0 0 1 0 1

Find
v0T Av0
v1 = Av0 v0
v0T v0

v1T Av1 v1T v1


v2 = Av1 v 1 v0 .
v1T v1 v0T v0
Are the vectors v0 , v1 , v2 linearly independent?

Problem 76. (i) Find the area of the set

S2 := { (x1 , x2 ) : 1 x1 x2 0 }.
Basic Operations 13

(ii) Find the volume of the set

S3 := { (x1 , x2 , x3 ) : 1 x1 x2 x3 0 }.

Extend the n-dimensions.

Problem 77. Let A be a hermitian n n matrix over C with A2 = In . Find


the matrix
(A1 + iIn )1 .

Problem 78. Find the 2 2 matrices F and F 0 from the two equations
   
0 0 1 0 1 1
= (F + iF 0 ), = (F iF 0 ).
1 0 2 0 0 2
Find the anticommutator of F and F 0 , i.e. [F, F 0 ]+ F F 0 + F 0 F .

Problem 79. Let A be an n n matrix over C. Find the condition on A such


that
(In + iA)(In iA) = In .
This means that (In iA) is the inverse of (In + iA).

Problem 80. Show that the 4 4 matrices



1 1/ 2 0 1/ 2

1 1/ 2 1 1/ 2 0
P =
2 0 1/ 2 1 1/ 2

1/ 2 0 1/ 2 1
and I4 P are projection matrices.

Problem 81. Find the corresponding permutation matrix for the permutation
 
1 2 3 4
.
3 4 1 2

Problem 82. Consider the vector space Rd . Suppose that {vj }dj=1 and
{wk }dk=1 are two bases in Rd . Then there is an invertible d d matrix

T = (tjk )dj,k=1

so that
d
X
vj = tjk wk , j = 1, 2, . . . , d.
k=1
14 Problems and Solutions

The bases {vj }dj=1 and {wk }dk=1 are said to have the same orientation if det(T ) >
0. If det(T ) < 0, then they have the opposite orientation. Consider the two bases
in R2
         
1 0 1 1 1 1
v1 = , v2 = , w1 = , w2 = .
0 1 2 1 2 1
Find the orientation.

Problem 83. (i) Let In be the n n matrix. Show that the 2n 2n matrix
 
1 In In
R=
2 In In
is invertible. Find the inverse.
(ii) Show that the 2n 2n matrix
 
1 In In
T =
2 iIn iIn

invertible. Find the inverse.

Problem 84. Let 1 , 2 R. Consider the vector in R3



cos(1 ) cos(2 )
v(1 , 2 ) = sin(2 ) cos(1 )
sin(1 )

(i) Find the 3 3 matrix v(1 , 2 )vT (1 , 2 ). What type of matrix do we have?
(ii) Find the eigenvalues of the 3 3 matrix v(1 , 2 )vT (1 , 2 ). Compare with
vT (1 , 2 )v(1 , 2 ).

Problem 85. Let A, B be n n matrices over C. We define (Jordan product)

1
AB = (AB + BA).
2
Show that A B is commutative and satisfies

A (A2 B) = A2 (A B)

but is not associative in general, i.e. (A B) C 6= A (B C) in general.

Problem 86. Let A, B be n n matrices over C. Is

tr(AA BB ) tr(AA )tr(BB ) ?


Basic Operations 15

Problem 87. Consider the six 3 3 permutation matrices denoted by P0 , P1 ,


P2 , P3 , P4 , P5 , where P0 is the 33 identity matrix. Find all triples (Q1 , Q2 .Q3 )
of these permutation matrices such that

Q1 Q2 Q3 = Q3 Q2 Q1

where Q1 6= Q2 , Q2 6= Q3 , Q3 6= Q1 .

Problem 88. (i) Find all 2 2 matrices A such that


   
0 1 0 1
A = A.
1 0 1 0

(ii) Impose the additional conditions such that tr(A) = 0 and det(A) = +1.
(iii) Find all 4 4 matrices B such that
       
0 1 0 1 0 1 0 1
B = B.
1 0 1 0 1 0 1 0

Problem 89. Find all hermitian 2 2 matrices H such that

H 2 = 02 .

Problem 90. Can one a find such that


cos()ei i sin()ei
   
1 1+i 1i
= .
i sin()ei cos()ei 2 1i 1+i

Problem 91. Can one find 4 4 matrices A and B such that


0 0 0 1 1 0 0 1

0 0 1 0 0 0 0 0
A B = ?
0 1 0 0 0 0 0 0
1 0 0 0 1 0 0 1
Of course at least one of the matrices A and B must be singular. The underlying
field could be R or char(F) = 2.

Problem 92. Find all 2 2 matrices such that A2 6= 02 but A3 = 02 .

Problem 93. Let R. Is the matrix


 
1 1+
T () =
(1 + ) 1
16 Problems and Solutions

invertible for all ?

Problem 94. Consider the four 8 8 binary matrices

0 0 0 0 0 0 0 0 0 0 0 0 0 0 0 0

0 0 0 0 0 0 0 0 0 0 0 0 0 0 0 0
0 0 0 0 0 0 0 0 0 0 1 0 0 1 0 0

0 0 0 1 1 0 0 0 0 0 0 0 0 0 0 0

S0 = , S1 = ,
0 0 0 1 1 0 0 0 0 0 0 0 0 0 0 0
0 0 0 0 0 0 0 0 0 0 1 0 0 1 0 0

0 0 0 0 0 0 0 0 0 0 0 0 0 0 0 0

0 0 0 0 0 0 0 0 0 0 0 0 0 0 0 0

0 0 0 0 0 0 0 0 1 0 0 0 0 0 0 1

0 1 0 0 0 0 1 0 0 0 0 0 0 0 0 0
0 0 0 0 0 0 0 0 0 0 0 0 0 0 0 0

0 0 0 0 0 0 0 0 0 0 0 0 0 0 0 0

S2 = , S3 = .
0 0 0 0 0 0 0 0 0 0 0 0 0 0 0 0
0 0 0 0 0 0 0 0 0 0 0 0 0 0 0 0

0 1 0 0 0 0 1 0 0 0 0 0 0 0 0 0

0 0 0 0 0 0 0 0 1 0 0 0 0 0 0 1
Find the 8 8 matrices Q0 , Q1 , Q2 , Q3 such that

Q0 S0 QT0 = S1 , Q1 S1 QT1 = S2 , Q2 S2 QT2 = S3 , Q3 S3 QT3 = S0 .

Problem 95. Can one find a (column) vector in R2 such that vvT is an
invertible 2 2 matrix?

Problem 96. Find the conditions on 1 , 2 , 3 R such that the 4 4 matrix


is invertible
0 1 0 0

1 1 2 3
A(1 , 2 , 3 ) =
0 2 1 0

0 3 0 1
is invertible.

Problem 97. Let A, B be invertible n n matrices. We define

A B := AB A1 B 1 .

Find matrices A, B such that A B is invertible.

Problem 98. Find all non-hermitian 2 2 matrices A such that AA = I2 .


Basic Operations 17

Problem 99. The standard simplex n is defined by the set in Rn


n
X
n := { (x1 , . . . , xn )T : xj 0, xj = 1 }.
j=1

Consider n affinely independent points B1 , . . . , Bn n . They span an (n 1)-


simplex denoted by = Con(B1 , . . . , Bn ), that is
n
X
= Con(B1 , . . . , Bn ) = { 1 B1 + + n Bn : j = 1, 1 , . . . , n 0 }.
j=1

The set corresponds to an invertible n n matrix [] whose columns are B1 ,


. . . , Bn . Conversely, consider the matrix C = (bjk ), where Ck = (b1k , . . . , bnk )T
(k = 1, . . . , n). If det(C) 6= 0 and the sum of the entries in each column is 1,
then the matrix C corresponds to an (n 1)-simplex Con(B1 , . . . , Bn ) in n .
Let C1 and C2 be n n matrices with nonnegative entries and all the columns
of each matrix add up to 1.
(i) Show that C1 C2 and C2 C1 are also such matrices.
(ii) Are the n2 n2 matrices C1 C2 , C2 C1 such matrices?

Problem 100. Let v be a normalized (column) vector in Cn . Consider the


n n matrix
1
A = vv In .
2
(i) Find A and AA .
(ii) Is the matrix A invertible?

Problem 101. Let > 0 (fixed) and t 0. Is the 4 4 matrix

1 + 3et 1 et 1 et 1 et

1 1 et 1 + 3et 1 et 1e t
P (t) =

4 1 et 1 et 1 + 3et 1 et

1 et 1 et 1 et 1 + 3e t

a stochastic matrix (probabilistic matrix)? Find the eigenvalues of P (t). Let


1
= (1 0 0 1).
2
Find the vectors P (t), (P (t))P (t) etc.. Is P (t1 + t2 ) = P (t1 )P (t2 )?

Problem 102. Let [0, 1]. Consider the stochastic matrix


 
1
P () = .
1
18 Problems and Solutions

Let n = 1, 2, . . .. Show that


 
1 1 + (2 1)n 1 (2 1)n
P n () = .
2 1 (2 1)n 1 + (2 1)n

Problem 103. Can any skew-hermitian matrix K be written as K = iH,


where H is a hermitian matrix?

Problem 104. Are the matrices


1 0 0 1

  1 0 1
1 1 0 0 0 0
, 0 0 0,
1 1 0 0 0 0

1 0 1
1 0 0 1
nonnegative? Extend to the n n case.

Problem 105. Let a b 0 and integers. Find the rank of the 4 4 matrix
a a b b

a b a b
M (a, b) =
b a b a

b b a a
Is the matrix nonnegative?

Problem 106. Find all 2 2 matrices A and B such that

A2 = B 2 = I2 , AB = I2 .

Problem 107. Find all 2 2 matrices


 
a11 a12
A=
0 a22

with a12 6= 0 such that A2 = I2 .

Problem 108. Consider the real symmetric matrix A and the vector v

0 1 0 1
1
A = 1 0 1, v = 1.
0 1 0 3 1

Calculate Sj = v Aj v for j = 1, 2, 3. Can A be reconstructed from S1 , S2 , S3


and the information that A is real symmetric?
Basic Operations 19

Problem 109. Let a, b R and c2 := a2 + b2 with c2 > 0. Consider the


matrix  
1 a b
M (a, b) = .
c b a
Find the matrix M (a, b)M (a, b). Discuss.

Problem 110. Let a, b, c, d R. Let

M = aI2 + b1 + c2 + d3 .

Find M 2 and express it as a linear combination of M and I2 .

Problem 111. Consider the vectors in R2


 
cos((k 1)/4
vk = , k = 1, 3, 5, 7
sin((k 1)/4)

and

 
cos((k 1)/4)
vk = 2 , k = 2, 4, 6, 8
sin(((k 1)/4)
which play a role for the lattice Boltzmann model. Find the angles between the
vectors. Find the angles between the vectors.

Problem 112. (i) Consider the normalized vectors in R4

1 0 1 0

1 0 1 1 1 0 1 1
v1 = , v2 = , v3 = , v4 = .
2 1 2 1 2 1 2 0
0 0 0 1

(i) Do the vectors form a basis in R4 ?


(ii) Find the 4 4 matrix v1 v1 + v2 v2 + v3 v3 + v4 v4 and then the eigenvalues.
(iii) Find the 4 4 matrix v1 v2 + v2 v3 + v3 v4 + v4 v1 and then the eigenvalues.
(iv) Consider the normalized vectors in R4

1 0 0 1

1 1 1 1 1 0 1 0
w1 = , w2 = , w3 = , w4 = .
2 0 2 1 2 1 2 0
0 0 1 1

(v) Do the vectors form a basis in R4 ?


(vi) Find the 44 matrix w1 w1 +w2 w2 +w3 w3 +w4 w4 and then the eigenvalues.
(vii) Find the 44 matrix w1 w2 +w2 w3 +w3 w4 +w4 w1 and then the eigenvalues.
20 Problems and Solutions

Problem 113. Let z = x + iy with x, y R and thus z = x iy. Consider


the maps  
x y
x + iy = A(x, y)
y x
and    
1 0
1 , i .
0 1
(i) Calculate z 2 and A2 (x, y). Discuss.
(ii) Find the eigenvalues and normalized eigenvectors of A(x, y).
(iii) Calculate A(x, y) A(x, y) and find the eigenvalues and normalized eigen-
vectors.

Problem 114. Let t 6= 0. Find the inverse of the matrix


 
t s
M= .
0 1

Problem 115. Consider the normalized vectors


     
0 3/2 3/2
v1 = , v2 = , v3 =
1 1/2 1/2
and the vector  
w1
w=
w2
in the Hilbert space C2 . Find
3
X
|vj w|2 .
j=1

Problem 116. Let S, T be n n matrices over C with

S 2 = In , (T S)2 = In .

Thus S and T are invertible. Show that ST S 1 = T 1 , ST 1 S = T .

Problem 117. Let A be a positive definite n n matrix over R. Let x Rn .


Show that A + xxT is positive definite.

Problem 118. (i) Show that the four 2 2 matrices


       
1 0 1 1 1 1 1 1
, , ,
0 0 0 0 0 1 1 1
Basic Operations 21

form a basis in the vector space of the 2 2 matrices. Which of these matrices
are nonnormal?
(ii) Show that the nine 3 3 matrices

1 0 0 1 1 0 1 1 1 1 1 1 1 1 1
0 0 0, 0 0 0, 0 0 0, 0 0 1, 0 0 1,
0 0 0 0 0 0 0 0 0 0 0 0 0 0 1

1 1 1 1 1 1 1 1 1 1 1 1
0 0 1, 0 0 1, 1 0 1, 1 1 1
0 1 1 1 1 1 1 1 1 1 1 1
form a basis in the vector space of the 3 3 matrices. This basis is called the
spiral basis (and can be extended to any dimension). Which of these matrices
are nonnormal?

Problem 119. (i) Consider the 3 2 matrix



1 1
A = 1 1.
1 1

Can one find 2 3 matrices B such that


 
1 0
AB = I2 = .
0 1

(ii) Consider the 3 2 matrix



1 2
X = 1 1.
1 1

Can one find 2 3 matrices Y such that


 
1 0
XY = I2 = .
0 1

Problem 120. Let A Cnm with n m and rank(A) = m.


(i) Show that the matrix m m matrix A A is invertible.
(ii) We set = A(A A)1 A. Show that is a projection matrix, i.e. 2 =
and = .

Problem 121. Find nonzero 2 2 matrices X and Y such that X + cY is


invertible for all c C.
22 Problems and Solutions

Problem 122. Consider a three state Markov model (states S1 , S2 , S3 ) with


the transition matrix between the states given by the stochastic matrix

0.4 0.3 0.3
T = 0.2 0.6 0.2 .
0.1 0.1 0.8

Given the system is in the state S3 at t = 0. Find the probabilty p(S3 , S3 , S3 , S1 , S1 , S3 , S2 , S3 ).


Basic Operations 23

Nonnormal Matrices

Problem 123. Let A be an n n hermitian matrix and P be an n n


projection matrix. Then P AP is again a hermitian matrix. Is this still true if
A is a normal matrix, i.e. AA = A A?

Problem 124. An n n matrix over C is called normal if M M = M M .


Let a, b C. What is the condition on a, b such that the 2 2 matrix
 
0 a
M=
b 0

is normal?

Problem 125. Two n n matrices A, B are called similar if there exists an


invertible n n matrix P such that

A = P BP 1 .

Show that the matrices


   
0 1 0 0
A= , B=
0 0 1 0

are similar. Show that the matrices are A, B are nonnormal. Find the commu-
tator [A, B] = AB BA. Is [A, B] nonnormal?

Problem 126. Let A be an n n matrix over C with A2 = In . Can we


conclude that A is normal?
24 Problems and Solutions

Nilpotent Matrices

Problem 127. An n n matrix is called nilpotent if some power of it is the


zero matrix, i.e. there is a positive integer p such that Ap = 0n . Show that
every nonzero nilpotent matrix is nondiagonalizable.

Problem 128. Consider the 4 4 matrix


0 0 0 0

1 0 0 0
N = .
0 1 0 0
0 0 1 0

Calculate N 2 , N 3 , N 4 . Is the matrix nilpotent?

Problem 129. Is the product of two n n nilpotent matrices nilpotent?

Problem 130. Find x, y R such that


    
2 3 3 0 2/3 x
= .
1 2 0 1 y 2
Basic Operations 25

Vector Product

Problem 131. Let



x1 y1
x = x2 , y = y2
x3 y3

be two normalized vectors in R3 . Assume that xT y = 0, i.e. the vectors are


orthogonal. Is the vector x y a unit vector again? Here denotes the vector
product. We have
x2 y3 x3 y2
x y = x3 y1 x1 y3 (1)
x1 y2 x2 y1
and
x21 + x22 + x23 = 1, y12 + y22 + y32 = 1 (2)
and
xT y = x1 y1 + x2 y2 + x3 y3 = 0. (3)

Problem 132. Let a, b, c R3 . Show that

a (b c) b (c a) c (a b)

where denotes the scalar product and the vector product.

Problem 133. Let v1 , v2 , v3 , v4 be column vectors in R3 . Show that

(v1 v2 )T (v3 v4 ) (v1T v3 )(v2T v4 ) (v2T v3 )(v1T v4 ).

A special case with v3 = v1 , v4 = v2 is

(v1 v2 )T (v1 v2 ) (v1T v1 )(v2T v2 ) (v2T v1 )(v1T v2 ).

Problem 134. Let a, b R3 and 1 , 2 , 3 be the Pauli spin matrices. We


define
a := a1 1 + a2 2 + a3 3 .
What is the condition on a, b such that

(a )(b ) (a b)I2 + i(a b) ?

Here denotes the vector product and I2 is the 2 2 identity matrix.


26 Problems and Solutions

Problem 135. Consider the three linear independent normalized column vec-
tors in R3


1 0 1
1 1
a1 = 0 , a2 = 1 , a3 = 0 .
2 1 0 2 1

(i) Find the volume


Va := aT1 (a2 a3 ).
(ii) From the three vectors a1 , a2 , a3 we form the 3 3 matrix

1/ 2 0 1/ 2
A = 0 1 0 .
1/ 2 0 1/ 2

Find the determinant and trace. Discuss.


(iii) Find the vectors
1 1 1
b1 = a2 a3 , b2 = a3 a1 , b3 = a1 a2
Va Va Va
where denotes the vector product. Are the vectors linearly independent?

Problem 136. Let x R3 and be the vector product.


(i) Find all solutions of

1 x1 0
0 x2 = 0 .
0 x3 1

(ii) Find all solutions of



0 x1 1/ 2
1 x2 = 0 .

0 x3 1/ 2

Problem 137. Let u, v R3 . Show that

(u v) (u v) (u u)(v v) (u v)2 .

Problem 138. Let a, b, c, d be vectors in R3 . Show that (Lagrange identity)


 
ac bc
(a b) (c d) = det .
ad bd
Basic Operations 27

Problem 139. (i) Consider a tetrahedron defined by the triple of linearly


independent vectors vj R3 , j = 1, 2, 3. Show that the normal vectors to the
faces defined by two of these vectors, normalized to the area of the face, is given
by
1 1 1
n1 = v2 v3 , n2 = v3 v1 , n3 = v1 v2 .
2 2 2
(ii) Show that
2 2 2
v1 = n2 n3 , v2 = n3 n1 , v3 = n1 n2
3V 3V 3V
where V is the volume of the tetrahedron given by
r
1 2
V = (v1 v2 ) v3 = (n1 n2 ) n3 .
3! 9
(iii) Apply it to normalized vectors

1 0 1
1 1
n1 = 0 , n2 = 1 , n3 = 0
2 1 0 2 1

which form an orthonormal basis in R3 .

Problem 140. Let v1 , v2 , v3 be three normalized linearly independent vectors


in R3 . Give an interpretation of A defined by
1 1 + v1 v2 + v2 v3 + v3 v1
cos( A) = p
2 2(1 + v1 v2 )(1 + v2 v3 )(1 + v3 v1 )
where denotes the scalar product. Consider first the case where v1 , v2 , v3
denote the standard basis in R3 .

Problem 141. Let A be an 3 3 matrix over R and u, v R3 . Find the


conditions on A, u, v such that

A(u v) = (Au) (Av).

Problem 142. Consider the three vectors v1 , v2 , v3 in R3 . Show that

v1 (v2 v3 ) = 0

if v1 , v2 , v3 are linearly dependent.

Problem 143. Consider four nonzero vectors v1 , v2 , v3 , v4 in R3 . Let

w := (v1 v2 ) (v3 v4 ) 6= 0.
28 Problems and Solutions

Find w (v1 v2 ) and w (v3 v4 ). Discuss. Note that v1 and v2 span a


plane in R3 and v3 and v4 span a plane in R3 .

Problem 144. Let u, v, w be vectors in R3 . Show that

u (v w) + w (u v) + v (w u) = 0.

Problem 145. Consider the normalized vectors



0 1
v1 = 1 , v2 = 0 .
0 1

Do the vectors v1 , v2 , v1 v)2 form an orthonormal basis in R3 ?

Problem 146. Consider the coordinates

p1 = (x1 , y1 , z1 )T , p2 = (x2 , y2 , z2 )T , p3 = (x3 , y3 , z3 )T

with p1 6= p2 , p2 6= p3 , p3 6= p1 . We form the vectors



x2 x1 x3 x1
v21 = y2 y1 , v31 = y3 y1 .
z2 z1 z3 z1

What does
1
|v21 v31 |
2
calculate? Apply it to p1 = (0, 0, 0)T , p2 = (1, 0, 1)T , p3 = (1, 1, 1)T .

Problem 147. Let 1 , 2 , 3 be the Pauli spin matrices and



a1 b1 1
a = a2 R3 , b = b2 R3 , = 2 .
a3 b3 3

Calculate (a)T )(b), where denotes the vector product and the scalar
product.

Problem 148. Find the 6 6 matrix



  0 1 0   0 0 1  
a11 a12 a13 a14 a15 a16
I3 + 0 0 1
+1 0 0
.
a16 a11 a12 a13 a14 a15
1 0 0 0 1 0
Basic Operations 29

Discuss.

Problem 149. Let A, B be n n circulant matrices over C. Is AB circulant?


Is A B circulant?

Problem 150. Consider the 3 3 matrices



0 1 0 a11 a12 a13
P = 0 0 1, A = a13 a11 a12 .
1 0 0 a12 a13 a11

Both matrices are circulant matrices. Find the commutator [A, B]. Discuss.
Chapter 2

Linear Equations

Let m, n be positive integers and F as field and M (mn, F) be the vector space of
m n matrices over F. Let A = (ajk M (m n, F) and b = (b1 , . . . , bm ) Fm .
Then

a11 x1 + + a1n xn = b1
.. .. ..
. . .
am1 x1 + + amn xn = bm

is called a system of linear equations with xj F. The x1 , . . . , xn are called the


unkowns of the linear system.

Problem 1. (i) Find all 2 2 matrices A over R with

det(A) = a11 a22 a12 a21 = 1 (1)

and    
1 1 1 1
A = . (2)
2 1 2 1
(ii) Do these matrices form a group under matrix multiplication?

Problem 2. Find all solutions of the linear system

x1 + 2x2 4x3 + x4 = 3
2x1 3x2 + x3 + 5x4 = 4
7x1 10x3 + 13x4 = 0.

30
Linear Equations 31

Problem 3. Consider the area-preserving map of the two-dimensional torus


(modulo 1)
x0
     
x 4 15
=A , A=
y0 y 1 4
where det A = 1 (area-preserving). Consider a rational point on the torus
   
x n1 /p
=
y n2 /p

where p is a prime number (except 2, 3, 5) and n1 , n2 are integers between 0


and p 1. One finds that the orbit has the following property. It is periodic
and its period T depends on p alone. Consider p = 7, n1 = 2, n2 = 3. Find the
orbit and the period T .

Problem 4. Solve the linear equation



1 2 3
( x1 x2 x3 ) 1 2 3 = ( x1 x2 x3 ) .
1 2 3

Problem 5. Gordans theorem tells us the following. Let A be an mn matrix


over R and c be an n-vector in Rn . Then exactly one of the following systems
has a solution:

System 1: Ax < 0 for some x Rn .


System 2: AT p = 0 and p 0 for some p Rm .

Let
1 0 1
A = 0 1 0.
1 0 1
Find out whether system (1) or system (2) has a solution.

Problem 6. Gordans theorem tells us the following: Let A be an mn matrix


over R. Exactly one of the following systems has a solution:

System 1: Ax < 0 for some x Rn


System 2: AT p = 0 and p 0 for some nonzero p Rn .

Let
2 1 1
A = 1 2 1.
1 1 2
32 Problems and Solutions

Find out whether system (1) or system (2) has a solution.

Problem 7. Farkas theorem tells us the following. Let A be an m n matrix


over R and c be an n-vector in Rn . Then exactly one of the following systems
has a solution:

System 1: Ax 0 and cT x > 0 for some x Rn .


System 2: AT y = c and y 0 for some y Rm .

Let
1 0 1 1
A = 0 1 0, c = 1.
1 0 1 1
Find out whether system (1) or system (2) has a solution.

Problem 8. Apply the Gauss-Seidel method to solve the linear system


4 1 1 0 x1 1

1 4 0 1 x2 0
= .
1 0 4 1 x3 0

0 1 1 4 x4 0

Problem 9. Let A be an n n matrix. Consider the linear equation Ax = 0.


If the matrix A has rank r, then there are n r linearly independent solutions
of Ax = 0. Let n = 3 and

0 1 1
A = 0 0 1.
0 0 0
Find the rank of A and the linearly independent solutions.

Problem 10. Consider the curve described by the equation


2x2 + 4xy y 2 + 4x 2y + 5 = 0 (1)
T T
relative to the natural basis (standard basis e1 = ( 1 0 ) , e2 = ( 0 1 ) ).
(i) Write the equation in matrix form.
(ii) Find an orthogonal change of basis so that the equation relative to the new
basis has no crossterms, i.e. no x0 y 0 term. This change of coordinate system
does not change the origin.

Problem 11. Consider the 2 2 matrix


 
b a
a b
Linear Equations 33

with a, b R and positive determinant, i.e. a2 + b2 > 0.


(i) Solve the equation
     
b a x1 b a x0
=
a b y1 a b y0

for the vector (x1 y1 )T with a given vector (x0 y0 )T .


(ii) Let
 1  
b a b a
M :=
a b a b
and  
0 1
J= .
1 0
Calculate M T JM .

Problem 12. Suppose that V is a vector space over a field F and U V is a


subspace. We define an equivalence relation on V by x y iff x y U . Let
V /U = V / . Define addition and scalar multiplication on V /U by [x] + [y] =
[x] + [y], c[x] = [cx], where c F and

[x] = { y V : y x }.

Show that these operations do not depend on which representative x we choose.

Problem 13. Consider the vector space V = C2 and the subspace U =


{(x1 , x2 ) : x1 = 2x2 }. Find V /U .

Problem 14. Find all solutions of the system of linear equations



5 2 4 x1 x1
2 2 2 x2 = x2 .
4 2 5 x3 x3

Problem 15. Let b > a. Consider the system of linear equations


1 1 1 . . . 1 w0 ba
x0 x1 x2 xn w1 (b2 a2 )/2
2
x0 x21 x22 . . . x2n w2 = (b3 a3 )/3

.
.
. .. .. .. .. ..
..
. . . . . . .

xn0 xn1 xn2 . . . xn n wn (bn+1
a n+1
)/(n + 1)

Let n = 2, a = 0, b = 1, x0 = 0, x1 = 1/2, x2 = 1. Find w0 , w1 , w2 .


34 Problems and Solutions

Problem 16. Let Y, X, A, B, C, E n n matrices over R. Consider the system


of matrix equations

Y + CE + DX = 0n , AE + BX = 0n .

Assume that A has an inverse. Eliminate the matrix E and solve the system for
Y

Problem 17. Let V be a vector space over a field F. Let W be a subspace of V .


We define an equivalence relation on V by stating that v1 v2 if v1 v2 W .
The quotient space V /W is the set of equivalence classes [v] where v1 v2 W .
Thus we can say that v1 is equivalent to v2 modulo W if v1 = v2 + w for some
w W . Let   
2 x1
V =R = : x1 , x2 R
x2
and   
x1
W = : x1 R .
0
(i) Is            
3 1 4 3 3 4
, , ?
0 0 1 1 0 1
(ii) Give the quotient space.

Problem 18. For the three-body problem the following linear transformation
plays a role
1
X(x1 , x2 , x3 ) = (x1 + x2 + x3 )
3
1
x(x1 , x2 , x3 ) = (x1 x2 )
2
1
y(x1 , x2 , x3 ) = (x1 + x2 2x3 ).
6
(i) Find the inverse transformation.
(ii) Introduce polar coordinates
1
x(r, ) = r sin , y(r, ) = r cos , r2 = ((x1 x2 )2 +(x2 x3 )2 +(x3 x1 )2 ).
3
Express (x1 x2 ), (x2 x3 ), (x3 x1 ) using this coordinates.

Problem 19. Let [0, 2). Find all solutions of the linear equation
    
cos() sin() x1 b1
= .
sin() cos() x2 b2
Linear Equations 35

Thus x1 and x2 depends on .

Problem 20. Consider the partial differential equation (Laplace equation)

2u 2u
+ 2 =0 on [0, 1] [0, 1]
x2 y
with the boundary conditions

u(x, 0) = 1, u(x, 1) = 2, u(0, y) = 1, u(1, y) = 2.

Apply the central difference scheme


 2 
2u
 
u uj1,k 2uj,k + uj+1,k uj,k1 2uj,k + uj,k+1
2
,
x j,k (x)2 y 2 j,k (y)2

and then solve the linear equation. Consider the cases x = y = 1/3 and
x = y = 1/4.

Problem 21. Let n and p be vectors in Rn with n 6= 0. The set of all vectors
x in Rn which satisfy the equation

n (x p) = 0

is called a hyperplane through the point p R. We call n a normal vector for


the hyperplane and call n (x p) = 0 a normal equation for the hyperplane.
Find n and p in R4 such that we obtain the hyperplane given by
7
x1 + x2 + x3 + x4 = .
2
Note that any hyperplane of the Euclidean space Rn has exactly two unit normal
vectors.

Problem 22. (i) The equation of a line in the Euclidean space R2 passing
through the points (x1 , y1 ) and (x2 , y2 ) is given by

(y y1 )(x2 x1 ) = (y2 y1 )(x x1 ).

Apply this equation to the points in R2 given by (x1 , y1 ) = (1, 1/2), (x2 , y2 ) =
(1/2, 1). Consider the unit square with the corner points (0, 0), (0, 1), (1, 0),
(1, 1) and the map

(0, 0) 0, (0, 1) 0, (1, 0 0, (1, 1) 1.

We can consider this as a 2 input AND-gate. Show that the line constructed
above classifies this map.
36 Problems and Solutions

(ii) The equation of a plane in R3 passing through the points (x1 , y1 , z1 ), (x2 , y2 , z2 ),
(x3 , y3 , z3 ) in R3 is given by

x x1 y y1 z z1
det x2 x1 y2 y1 z2 z1 = 0.
x3 x1 y3 y1 z3 z1

Apply this equations to the points

(1, 1, 1/2), (1, 1/2, 1), (1/2, 1, 1).

Consider the unit cube in R3 with the corner points (vertices)

(0, 0, 0), (0, 0, 1), (0, 1, 0), (0, 1, 1)

(1, 0, 0), (1, 0, 1), (1, 1, 0), (1, 1, 1)


and the map where all corner points are mapped to 0 except for (1, 1, 1) which
is mapped to 1. We can consider this as a 3 input AND-gate. Show that the
plane constructed in (i) separates these solutions.

Problem 23. Find the system of linear equations for a and b given by
x + 15 a b
= + .
(x + 3)(x 1) x+3 x1

Solve the system of linear equations.

Problem 24. Consider the linear equation written in matrix form



1 0 1 x1 1
0 1 1 x2 = 2 .
1 0 2 x3 1

First show that the determinant of the 3 3 matrix is nonzero. Apply two
different methods (Gauss elimination and the Leverriers method) to find the
solution. Compare the two methods and discuss.

Problem 25. Let A be a given 3 3 matrix over R with det(A) 6= 0. Is the


transformation
a11 x + a12 y + a13
x0 (x, y) =
a31 x + a32 y + a33
a21 x + a22 y + a23
y 0 (x, y) =
a31 x + a32 y + a33
invertible? If so find the inverse.
Linear Equations 37

Problem 26. Let k = 1, 2, 3, 4 and x0 = 1, x5 = 0. Solve


xk1 2xk + xk+1 = 0.

Problem 27. Find the solution of the system of linear equations


x1 + x2 + x3 = 0, x1 + 2x2 + x3 = 1, 2x1 + x2 + x3 = 2.

Problem 28. Consider the polynomial


p(x) = a + bx + c2 .
Find a, b, c from the conditions
p(0) = 0, p(1) = 1, p(2) = 0.

Problem 29. Let A Cnm with n m and rank(A) = m.


(i) Show that the m m matrix A A is invertible.
(ii) We set P := A(A A)1 A. Show that P is a projection matrix, i.e. P 2 = P
and P = P .

Problem 30. (i) Find the solutions of the equation


    
1 2 x1 x2 0 0
= .
3 4 x3 x4 0 0
(ii) Find the solutions of the equation
    
1 2 x1 x2 1 0
= .
3 4 x3 x4 0 1

Problem 31. Consider the system of three linear equations



1 1 1 x1 1
1 2 4 x2 = 
1 4 10 x3 2
with  R. Find the condition on  such that the linear system admits a
solution.

Problem 32. Let a, b, c R and abc 6= 0. Find the solution of the system of
linear equations
0 c b cos() a
c 0 a cos() = b .
b a 0 cos() c
Chapter 3

Traces, Determinants and


Hyperdeterminants

Let A be an n n matrix over C. The trace is defined as the sum of the diagonal
elements
Xn
tr(A) = ajj .
j=1
Problem 1. Find all 2 2 matrices A over C that satisfy the three conditions
tr(A) = 0, A = A , A2 = I2 .

Problem 2. Find all 2 2 matrices A such that


A2 = tr(A)A.
Calculate det(A) and det(A2 ) of such a matrix.

Problem 3. Let A, B be 2 2 matrices over C with tr(A) = 0 and tr(B) = 0.


What are conditions on A and B such that tr(AB) = 0?

Problem 4. Let 1 , 2 , 3 be the Pauli spin matrices. Calculate the trace of


1 , 2 , 3 , 1 2 , 1 3 , 2 3 , 1 2 3 .

Problem 5. Let A be an n n matrix with A2 = In . Let B be a matrix with


AB = BA, i.e. [A, B]+ = 0n .

38
Traces, Determinants and Hyperdeterminants 39

(i) Show that tr(B) = 0.


(ii) Find tr(A B).

Problem 6. (i) Consider the two 2 2 matrices


   
a11 1 a11 0
A= , B= .
a21 0 a21 1

The first column of the matrices A and B agree, but the second column of the
two matrices differ. Is

det(A + B) = 2(det(A) + det(B))?

Problem 7. Let A, B be 2 2 matrices. Assume that det(A) = 0 and


det(B) = 0. Can we conclude that det(A + B) = 0?

Problem 8. Let u, v, w be column vectors in R3 . We form the 3 3 matrix

M = (u v w).

Is det(M ) = (u v) w, where denotes the vector product and the scalar


product?

Problem 9. The oriented volume of an n-simplex in n-dimensional Euclidean


space with vertices v0 , v1 , . . . , vn is given by
1
det(S)
n!
where S is the n n matrix

S := (v1 v0 v2 v0 . . . vn1 v0 vn v0 ).

Thus each column of the n n matrix is the difference between the vectors
representing two vertices.
(i) Let      
0 1 1/2
v0 = , v1 = , v2 = .
0 1 1
Find the oriented volume.
(ii) Let

0 1 0 0
v0 = 0 , v1 = 0 , v2 = 1 , v3 = 0 .
0 0 0 1
40 Problems and Solutions

Find the oriented volume.

Problem 10. The area A of a triangle given by the coordinates of its vertices

(x0 , y0 ), (x1 , y1 ), (x2 , y2 )

is
x0 y0 1
1
A = det x1 y1 1.
2
x2 y2 1
(i) Let (x0 , y0 ) = (0, 0), (x1 , y1 ) = (1, 0), (x2 , y2 ) = (0, 1). Find A.
(ii) A tetrahedron is a polyhedron composed of four triangular faces, three of
which meet at each vertex. A tetrahedron can be defined by the coordinates of
the vertices

(x0 , y0 , z0 ), (x1 , y1 , z1 ), (x2 , y2 , z2 ), (x3 , y3 , z3 ).

The volume V of the tetrahedron is given by

x0 y0 z0 1

1 x1 y1 z1 1
V = det .
6 x2 y2 z2 1
x3 y3 z3 1

Let
(x0 , y0 , z0 ) = (0, 0, 0), (x1 , y1 , z1 ) = (0, 0, 1),
(x2 , y2 , z2 ) = (0, 1, 0), (x3 , y3 , z3 ) = (1, 0, 0).
Find the volume V .
(iii) Let

(x0 , y0 , z0 ) = (+1, +1, +1), (x1 , y1 , z1 ) = (1, 1, +1),

(x2 , y2 , z2 ) = (1, +1, 1), (x3 , y3 , z3 ) = (+1, 1, 1).


Find the volume V .

Problem 11. Let A, B be n n matrices. Assume that [A, B] = A. What


can be said about the trace of A?

Problem 12. Find all linearly independent diagonal 3 3 matrices over R


with trace zero.

Problem 13. Let A be a 2 2 matrix over C. Assume that A2 = I2 and thus


tr(A2 ) = 2. What can be said about the trace of A?
Traces, Determinants and Hyperdeterminants 41

Problem 14. Let A, B be n n matrices over C. Assume that

tr(AB) = 0.

(i) Can we conclude that tr(AB ) = 0?


(ii) Consider the case that B is skew-hermitian.

Problem 15. Find all 2 2 matrices A with

tr(A2 ) (tr(A))2 = 0

Problem 16. Let A, B be n n positive semidefinite matrices.


(i) Show that
1
tr(A B) (tr(A) + tr(B))2
4
(ii) Show that
1
tr(A B) (tr(A A + B B).
2

Problem 17. Let

A = (a1 , a2 , . . . , an1 , u), B = (a1 , a2 , . . . , an1 , v)

be n n matrices, where the first n 1 columns a1 , . . . , an1 are the same and
for the last column u 6= v. Show that

det(A + B) = 2n1 (det(A) + det(B)).

Problem 18. An n n tridiagonal matrix (n 3) has nonzero elements


only in the main diagonal, the first diagonal below this, and the first diagonal
above the main diagonal. The determinant of an n n tridiagonal matrix can
be calculated by the recursive formula

det(A) = an,n det[A]{1,...,n1} an,n1 an1,n det[A]{1,...,n2 }

where det[A]{1,...,k} denotes the k-th principal minor, that is, [A]{1,...,k} is the
submatrix by the first k rows and columns of A. The cost of computing the
determinant of a tridiagonal matrix using this recursion is linear in n, while the
cost is cubic for a general matrix. Apply this recursion relation to calculate the
determinant of the 4 4 matrix
0 1 0 0

1 1 2 0
A= .
0 2 2 3
0 0 3 3
42 Problems and Solutions

Problem 19. (i) Let A be a 2 2 matrix over R. Assume that

tr(A) = tr(A2 ) = tr(A3 ) = tr(A4 ) = 0.

Can we conclude that A is the 2 2 zero matrix?


(ii) Assume that A is a normal matrix and satisfies these conditions. Can we
conclude that A is the 2 2 zero matrix?

Problem 20. Find all 2 2 matrices A with

tr(A2 ) (tr(A))2 = 0.

Problem 21. Consider the symmetric n n band matrix (n 3)

1 1 0 ... 0 0

1 1 1 ... 0 0
0 1 1 ... 0 0

Mn = ... ... ... . . . .. ..
. .
0 0 0 ... 1 1
0 0 0 ... 1 1

with the elements in the field F2 . Show that

det(Mn ) = det(Mn1 ) det(Mn2 )

with the initial conditions det M3 = det M4 = 1. Show that the solution is

2 3  n 
det(Mn ) = cos (mod 2).
3 3 6

Problem 22. Let H be the 8 8 matrix


1 0 0 0

 
0 0 1 0 0
H= , = .
0 I4 0 0 1 0
0 0 0 1

Let A, D, X, Y be 4 4 matrices over C and


 
A X
M= .
Y D

Find the conditions on the matrix M such that

HM + M H = 08
Traces, Determinants and Hyperdeterminants 43

and trA trD = 0.

Problem 23. Consider the symmetric 3 3 matrix



1 1
A() = 1 1 , R.
1 1

(i) Find the maxima and minima of the function

f () = det(A()).

(ii) For which values of is the matrix noninvertible?

Problem 24. Let A, B be n n hermitian positive definite matrices. Show


that
tr(AB) > 0.

Problem 25. Let A, B be n n matrices over R. Assume that A is invertible.


Let t be a nonzero real number. Show that

det(A + tB) = tn det(A) det(A1 B + t1 In ).

Problem 26. Let A be an n n invertible matrix over R. Show that AT is


also invertible. Is (AT )1 = (A1 )T ?

Problem 27. Consider the nonnormal matrices



  0 1 1
0 1
A= , B = 0 0 1.
0 0
0 0 0

Find (det(A A))1/2 and (det(B B))1/2 .

Problem 28. Let A be an 2 2 matrix over R. Let I2 be the 2 2 unit matrix


and R. Find the determinant of the 4 4 matrix
 
I2 A
.
AT I2

Problem 29. Let A be an 2 2 matrix over R. Calculate

r = tr(A2 ) (tr(A))2 .
44 Problems and Solutions

What are the conditions on ajk such that r = 0?

Problem 30. Let A be a 2 2 symmetric matrix


 
a11 a12
A=
a12 a22

over R. We define  
0 1
A= .
a12 1 0
Show that    
A
trA2 = tr A2 = tr 2A .
a12 a12 a12

Problem 31. Let A, B be n n matrices over C. Is

tr(A B) = tr(AB ) ?

Problem 32. Let A, B be 2 2 matrices. Show that

[A, B]+ AB + BA = (tr(AB) tr(A)tr(B))I2 + tr(A)B + tr(B)A.

Can this identity be extended to 3 3 matrices?

Problem 33. Find all nonzero 2 2 matrices A and B such that

BA = tr(AA )A.

Problem 34. Consider the Hilbert space M4 (C) of all 4 4 matrices over
C with the scalar product hA, Bi := tr(AB ), where A, B M4 (C). The -
matrices are given by

0 0 0 i 0 0 0 1

0 0 i 0 0 0 1 0
1 = , 2 =
0 i 0 0 0 1 0 0

i 0 0 0 1 0 0 0

0 0 i 0 1 0 0 0

0 0 0 i 0 1 0 0
3 = , 4 =
i 0 0 0 0 0 1 0

0 i 0 0 0 0 0 1
Traces, Determinants and Hyperdeterminants 45

and
0 0 1 0

0 0 0 1
5 = 1 2 3 4 = .
1 0 0 0
0 1 0 0
We define the 4 4 matrices
i
jk := [j , k ], j<k
2
where j = 1, 2, 3, k = 2, 3, 4 and [ , ] denotes the commutator.
(i) Calculate 12 , 13 , 14 , 23 , 24 , 34 .
(ii) Do the 16 matrices

I4 , 1 , 2 , 3 , 4 , 5 , 5 1 , 5 2 , 5 3 , 5 4 , 12 , 13 , 14 , 23 , 24 , 34

form a basis in the Hilbert space M4 (C)? If so is the basis orthogonal?

Problem 35. Let n be even. Consider a skew-symmetric n n matrix over R.


Let B be a symmetric n n matrix over R with the entries of B = (bjk ) given
by bjk = bj bk (bj , bk R). Show that

det(A + B) = det(A).

Problem 36. Find the determinant and the inverse of the matrix
 x 
e cos(x) ex sin(x)
.
ex sin(x) ex cos(x)

Problem 37. Let h > 0 and b > a. Show that the matrix
 
1 ah
M (a, b, h) =
1 bh
has an inverse. Find the inverse matrix.

Problem 38. Let A, B be 2 2 matrices. Show that


   
a11 a12 b11 b12
det(A + B) = det(A) + det(B) + det + det .
b21 b22 a21 a22

Problem 39. Let n 2. An invertible integer matrix, A GL(n, Z), gener-


ates a toral automorphism f : Tn Tn via the formula

f = A, : Rn Tn := Rn /Zn .
46 Problems and Solutions

The set of fixed points of f is given by

Fix(f ) := { x Tn : f (x ) = x }.

Let ]Fix(f ) be the number of fixed points of f . Now we have: if det(In A) 6= 0,


then
]Fix(f ) = | det(In A)|.
Let n = 2 and  
2 1
A= .
1 1
Show that det(I2 A) 6= 0 and find ]Fix(f ).

Problem 40. Calculate the determinant of the 4 4 matrix


1 0 0 1

0 1 1 0
A=
0 1 1 0

1 0 0 1

using the exterior product. This means calculate

1 0 0 1

0 1 1 0
.
0 1 1 0
1 0 0 1

Problem 41. (i) Let R. Find the determinant of the matrices


   
cos sin cos i sin
A() = , B() = .
sin cos i sin cos

(ii) Let R. Find the determinant of the matrices


   
cosh sinh cosh i sinh
A() = , B() = .
sinh cosh i sinh cosh

Problem 42. The 3 3 diagonal matrices over R with trace equal to 0 form
a vector space. Provide a basis for this vector space. Using the scalar product
tr(AB T ) for n n matrices A, B over R the elements of the basis should be
orthogonal to each other.

Problem 43. Let A be a n n matrix with det A = 1. Find det(A1 ).


Traces, Determinants and Hyperdeterminants 47

Problem 44. The Hilbert-Schmidt norm of an n n matrix over C is given


by p
kAk2 = tr(A A).
Another norm is the trace norm given by
p
kAk1 = tr (A A).

Calculate the two norms for the matrix


 
0 2i
A= .
i 0

Problem 45. The n n permutation matrices form a group under matrix


multiplications. Show that
det(In P ) = 0
for any n n permutation matrices.

Problem 46. What is the condition on entries of the 3 3 matrix



a11 0 a13
A = a21 a22 a23
a31 0 a33

such that the matrix is invertible?

Problem 47. Let


x1 y1
x2 , y2 R3 .
x3 y3
What does
1 x1 y1
1
det 1 x2 y2
2
1 x3 y3
calculate?

Problem 48. Let A be a 3 3 matrix over R. Consider the permutation


matrix
0 0 1
P = 0 1 0.
1 0 0
Assume that AP = A. Is A invertible?
48 Problems and Solutions

Problem 49. Let A = (aij ) be a 2n2n skew-symmetric matrix. The Pfaffian


is defined as
n
1 X Y
Pf(A) := n sgn() a(2j1),(2j)
2 n! j=1
S2n

where S2n is the symmetric group and sgn() is the signature of permutation .
Consider the case with n = 2, i.e.
0 a12 a13 a14

a 0 a23 a24
A = 12 .
a13 a23 0 a34
a14 a24 a34 0

Calculate Pf(A).

Problem 50. Let A be a skew-symmetric 2n 2n matrix. For the Pfaffian we


have the properties

(Pf(A))2 = det(A), Pf(BAB T ) = det(B)Pf(A)

Pf(A) = n Pf(A), Pf(AT ) = (1)n Pf(A).


where B is an arbitrary 2n 2n matrix. Let J be a 2n 2n skew-symmetric
matrix with Pf(J) 6= 0. Let B be a 2n 2n matrix such that B T JB = J. Show
that det(B) = 1.

Problem 51. Consider the Legendre polynomials Pj , where


1
p0 (x) = 1, p1 (x) = x, p2 (x) = (3x2 1)
2
1 1
p3 (x) = (5x3 3x), p4 (x) = (35x4 30x2 + 3).
2 8
Show that

p0 (x) p1 (x) p2 (x) p0 (0) 0 p2 (0)
det p1 (x) p2 (x) p3 (x) = (1 x2 )3 0 p2 (0) 0 .
p2 (x) p3 (x) p4 (x) p2 (0) 0 p4 (0)

Problem 52. Let n 2. Consider the n n matrix


1 2 3 ... n1 n

2 3 4 ... n 1
3 4 5 ... 1 2

A= . . .
.
.. .. .. .. .. ..
. . .
n 1 2 ... n2 n1
Traces, Determinants and Hyperdeterminants 49

Show that
1
det(A) = (1)n(n1)/2 (n + 1)nn1 .
2

Problem 53. Let n 2. Consider the n n matrix


c1 x x . . . x x

x c2 x . . . x x
x x c3 . . . x x

A(x) = .
.
.. .. .. . . . .
. . . .. ..
x x x ... x cn
Show that
det(A) = (1)n (P (n) xP 0 (x))
where P (x) = (x c1 )(x c2 ) (x cn ).

Problem 54. Let A1 , A2 , A3 be 2 2 matrices over R. We define

cj = tr(Aj ) j = 1, 2, 3
cjk = tr(Aj Ak ) j, k = 1, 2, 3

(i) Given the coefficients cj , cjk reconstruct the matrices A1 , A2 , A3 .


(ii) Apply the result to the case

c1 = c2 = c3 = 0

c11 = 0, c12 = 1, c13 = 0,


c21 = 1, c22 = 0, c23 = 0,
c31 = 0, c32 = 0, c33 = 2.

Problem 55. Let V1 be a hermitian n n matrix. Let V2 be a positive


semidefinite n n matrix. Let k be a positive integer. Show that

tr((V2 V1 )k )
1/2 1/2
can be written as tr(V k ), where V := V2 V1 V2 .

Problem 56. Consider the 2 2 matrix


 
cosh(r) sinh(r) cos(2) sinh(r) sin(2)
M= .
sinh(r) sin(2) cosh(r) + sinh(r) cos(2)

Find the determinant of M . Thus show that the inverse of M exists. Find the
inverse of M .
50 Problems and Solutions

Problem 57. Let A, B be n n matrices over C. Is tr(AB ) = tr(A B)?

Problem 58. Consider the matrix


 
0 1
A= .
1 1

Show that
tr(Ak ) = tr(Ak1 ) + tr(Ak2 ), k = 3, 4, . . . .

Problem 59. Let A be an n n matrix. Assume that the inverse of A exists,


i.e. det(A) 6= 0. Then the inverse B = A1 can be calculated as

ln(det(A)) = bkj .
ajk
Apply this formula to the 2 2 matrix A
 
a11 a12
A=
a21 a22

with det(A) = a11 a22 a12 a21 6= 0.

Problem 60. Show that the determinant of the matrix



1/2 0 1/ 2
A = 1/ 2 0 1/ 2
0 1 0

is nonzero. Find the inverse of the matrix.

Problem 61. Consider the 2 2 matrix over C


 
c11 c12
C= .
c21 c22

Calculate det(CC ) and show that det(CC ) 0.

Problem 62. Let : Rn Rn R be an analytic function, where (x, y) =


(x1 , . . . , xn , y1 , . . . , yn ) Rn Rn . The Monge-Ampere determinant M () is
defined by
1/x ... /x
n
/y1 2 /x1 y1 ... 2 /xn y1
M () := det .. .. .. .. .

. . . .

/yn 2 /x1 yn ... 2 /xn yn
Traces, Determinants and Hyperdeterminants 51

Let n = 2 and

(x1 , x2 , y1 , y2 ) = x21 + x22 + (x1 y1 )2 + (x2 y2 )2 + y12 + y22 .

Find the Monge-Ampere determinant and the conditions on x1 , x2 , y1 , y2 such


that M () = 0.

Problem 63. (i) Let z C. Find the determinant of


 
1 z
A= .
z z z

Is the matrix
1
P2 = I 2 A
1 + z z
a projection matrix?
(ii) Let z1 , z2 C. Find the determinant of

1 z1 z2
B = z1 z1 z1 z2 z1 .
z2 z1 z2 z2 z2

Is the matrix
1
P3 = I 3 B
1 + z1 z1 + z2 z2
a projection matrix?

Problem 64. Find the determinant of the 4 4 matrices

a11 a12 0 0 1 0 a13 a14



a21 a22 0 0 0 1 a23 a24
, .
a31 a32 1 0 0 0 a33 a34

a41 a42 0 1 0 0 a43 a44

Problem 65. Let T be the 2 2 matrix


 
1 1 1
T = .
3 1 1

Calculate ln(det(I2 T )) using the right-hand side of the identity



X 1
ln(det(I2 T )) = tr(T k ).
k
k=1
52 Problems and Solutions

Problem 66. Let A be an n n matrix. Assume that

tr(Aj ) = 0, for j = 1, 2, . . . , n.

Can we conclude that det(A) = 0?


Problem 67. Consider the golden mean number = ( 5 1)/2 and the
matrix  

F = .

Find tr(F ) and det(F ). Since det(F ) 6= 0 we have an inverse. Find F 1 .

Problem 68. Let A be an n n matrix and B be an invertible n n matrix.


Show that
det(In + A) = det(In + BAB 1 ).

Problem 69. Let A be an 2 2 matrix. Show that

det(I2 + A) = 1 + tr(A) + det(A).

Can the result extended to det(I3 + A)?

Problem 70. Let A, B be n n matrices over R. Assume that A = AT


(symmetric) and B = B T (skew-symmetric). Show that [A, B] is symmetric.

Problem 71. Let A be a 2 2 matrix over C. Let

t1 = tr(A), t2 = tr(A2 ), t3 = tr(A3 ), t4 = tr(A4 ).

Can we reconstruct A from t1 , t2 , t3 , t4 ?

Problem 72. The Levi-Civita symbol (also called completely antisymmetric


constant tensor) is defined by

+1 if j1 , j2 , . . . , jn is an even permutation of 12 n
j1 ,j2 ,...,jn := 1 if j1 , j2 , . . . , jn is an odd permutation of 12 n
0 otherwise

Let jk be the Kronecker delta. Show that


j2 k1 ... jn k 1
j1 k1
j1 k2 j2 k2 ... jn k 2
j1 ,j2 ,...,jn k1 ,k2 ,...,kn = det
.. .. .
. .

j1 kn j2 kn ... jn kn
Traces, Determinants and Hyperdeterminants 53

Problem 73. Let x,  R. Find the determinant of the symmetric n n


matrix
x+ x x

x x+ x
A= ... .. .
.
x x x+

Problem 74. Let  R. Let A() be an invertible n n matrix. Assume that


the entries ajk are analytic functions of . Show that
 
1 d 1 d
tr A () A() = det(A()).
d det(A()) d

Problem 75. Let { ej } be the three orthonormal vectors in Z3



1 0 0
e1 = 0 , e2 = 1 , e3 = 0 .
0 0 1

We consider the face-centered cubic lattice as a sublattice of Z3 generated by


the three primitive vectors

e1 + e2 , e1 + e3 , e2 + e3 .

(i) Form the 3 3 matrix

(e1 + e2 e1 + e3 e2 + e3 ).

(ii) Show that this matrix has an inverse and find the inverse.

Problem 76. Let A, B, C be n n matrices. Show that

tr([A, B]C) = tr(A[B, C]).

Problem 77. (i) Let M be a 2 2 matrix over R. Assume that tr(M ) = 0.


Show that
M 2 = det(M )I2 .
(ii) Show that

p
M sin( det(M ))
e = cos( M )I2 + p M.
det(M )
If det(M ) = 0 then sin(0)/0 = 1. Both cos() and sin()/ are even functions
of and thus exp(M ) is independent of the choice of the square root of det(M ).
54 Problems and Solutions

Problem 78. Consider the m m matrix F (x) = (fjk (x)) (j, k = 1, 2, . . . , m),
where fjk : Rn R are analytic functions. Assume that F (x) is invertible for
all x Rn . Then we have the identities (j = 1, 2, . . . , m)
 
(det(F (x))) 1 F (x)
det(F (x))tr F (x)
xj xj

and
F 1 (x) F (x) 1
F 1 (x) F (x).
xj xj
The differentiation is understood entrywise. Apply the identities to the matrix
(m = 2, n = 1)  
cos(x) sin(x)
F (x) = .
sin(x) cos(x)

Problem 79. Let f1 , f2 , f3 : R3 R be continuously differentiable functions.


Find the determinant of the 3 3 matrix A = (ajk )

fj fk
ajk := .
xk xj

Problem 80. Consider the 3 3 permutation matrix



0 0 1
P = 0 1 0.
1 0 0

Can we find a 3 3 matrix A such that [P, A] is invertible?

Problem 81. Let n 2. Consider the n n symmetric tridiagonal matrix


over R
c 1 0 0 0

1 c 1 0 0
0 1 c 1 0





An =







0 0 0 1 c 1 0

0 0 0 0 1 c 1

0 0 0 0 0 1 c
where c R. Find the determinant of An .
Traces, Determinants and Hyperdeterminants 55

Problem 82. An n n matrix A is called idempotent if A2 = A. Show that


rank(A) = tr(A).

Problem 83. Let A, B, C be n n matrices over C. Let aj , bj , cj (j =


1, 2, . . . , n) be the j-th column of A, B, C, respectively. Show that if for some
k {1, 2, . . . , n }
ck = ak + bk
and
cj = aj = bj , j = 1, . . . , k 1, k + 1, . . . , n
then
det(C) = det(A) + det(B).

Problem 84. Let R be an nonsingular n n matrix over C. Let A be an n n


matrix over C of rank one.
(i) Show that the matrix R + A is nonsingular if and only if
tr(R1 A) 6= 1.
(ii) Show that in this case we have
(R + A)1 = R1 (1 + tr(R1 A))1 R1 AR1 .
(iii) Simplify to the case that R = In .

Problem 85. Let A be an n n diagonal matrix over C. Let B be an n n


matrix over C with bjj = 0 for all j = 1, . . . , n. Can we conclude that all diagonal
elements of the commutator [A, B] are 0?

Problem 86. (i) Find a nonzero 2 2 matrix V such that


V 2 = tr(V )V.
(ii) Can such a matrix be invertible?

Problem 87. Consider the (n + 1) (n + 1) matrix over C


1 0 0 . . . . . . 0 z1

0 1 0 . . . . . . 0 z2

0 0 1 . . . . . . 0 z3
. .. .. . . . ..
.
A = .. . . . . . .. . .

. .. .. . . . . ..
.. . . . . . .. .

0 0 0 ... ... 1 z
n
z1 z2 z3 . . . . . . zn 1
56 Problems and Solutions

Find the determinant. What is the condition on the zj s such that A is invertible?

Problem 88. Let R. Consider the unitary matrix


i
e 0 0
U () = 0 1 0 .
0 0 ei
Find the minima and maxima of the function tr(U ()).

Problem 89. Let A, B be n n matrices over C. Assume that B is invertible.


Find
det(In + BAB 1 ).

Problem 90. Let zk = xk + iyk , where xk , yk R and k = 1, . . . , n. Find the


2n 2n matrix A such that
z1 x1

.. ..
. .
zn xn

= A .
z1 y1
. .
.. ..
zn yn
Find the determinant of the matrix A.

Problem 91. Let A, B be n n matrices over C. We define the product


1 1
A ? B := (AB + BA) tr(AB)In .
2 n
(i) Find the trace of A ? B.
(ii) Is the product commutative? Is the product associative?

Problem 92. Let A, B be n n matrices over R with det(A) = 1 and


det(B) = 1. This means A, B are elements of the Lie group SL(n, R). Can we
conclude that
tr(AB) + tr(AB 1 ) = tr(A)tr(B).

Problem 93. Let A, B be two 2 2 matrices. We define the product


a11 0 0 a12

0 b11 b12 0
A ? B := .
0 b21 b22 0
a21 0 0 a22
Traces, Determinants and Hyperdeterminants 57

(i) Find the determinant and trace of A ? B. Express the result using tr(A),
tr(B), det(A), det(B).
(ii) Assume that the inverse of A and B exists. Is

(A ? B)1 = A1 ? B 1 ?

Problem 94. Let A be an n n invertible matrix over C. Let x, y Cn .


Then we have the identity

det(A + xy ) det(A)(1 + y A1 x).

Can we conclude that A + xy is also invertible?

Problem 95. Consider a triangle embedded in R3 . Let vj = (xj , yj , zj )


(j = 1, 2, 3) be the coordinates of the vertices. Then the area A of the triangle
is given by
1 1
A= k(v2 v1 ) (v1 v3 )k = k(v3 v1 ) (v3 v2 )k
2 2
where denotes the vector product and k.k denotes the Euclidean norm. The
area of the triangle can also be found via
v 2 2 2
u
u x1 y1 1 y1 z1 1 z1 x1 1
1 u
A = t det x2 y2 1 + det y2 z2 1 + det z2 x2 1 .
2
x3 y3 1 y3 z3 1 z3 x3 1

Consider
v1 = (1, 0, 0), v2 = (0, 1, 0), v3 = (0, 0, 1).
Find the area of the triangle using both expressions. Discuss. The triangle could
be one of the faces of a tetrahedron.

Problem 96. A tetrahedron has four triangular faces. Given the coordinates
of the four vertices

(x0 , y0 , z0 ), (x1 , y1 , z1 ), (x2 , y2 , z2 ), (x3 , y3 , z3 )

the volume of the tetrahedron is given by


x0 y0 z0 1

1 x1 y1 z1 1
V = det .
3! x2 y2 z2 1
x3 y3 z3 1

(i) Given the four vertices (1, 0, 2), (2, 0, 0), (0, 0, 0), (1, 2, 0) find the
volume.
58 Problems and Solutions

(ii) Derive an equation for surface area of a tetrahedron given by coordinates.


Apply it to the vertices given in (i).

Problem 97. Let v1 , v2 , . . . , vm be vectors in Rn . Show that the paral-


lelepiped determined by those vectors has m-dimensional area
q
det(V T V )

where V is the n m matrix with v1 , v2 , . . . , vm as its columns.

Problem 98. Let H be a hermitian nn matrix. Show that det(H +iIn ) 6= 0.

Problem 99. Find the determinant of the matrices



  1 1/2 1/3
1 1/2
, 1/2 1/3 1/4
1/2 1/3
1/3 1/4 1/5

1 1/2 1/3 1/4



1/2 1/3 1/4 1/5
.
1/3 1/4 1/5 1/6

1/4 1/5 1/6 1/7
Extended to n n matrices. Then consider the limit n .

Problem 100. Let aj R with j = 1, 2, 3. Consider the 4 4 matrices

0 a1 0 0 0 a1 0 0

1 a1 0 a2 0 1 a1 0 a2 0
A= , B= .
2 0 a2 0 a3 2i 0 a2 0 a3

0 0 a3 0 0 0 a3 0

Find the spectrum of A and B. Find the spectrum of [A, B].

Problem 101. (i) Let A be a 2 2 matrix over C. Given

t1 = tr(A), t2 = tr(A2 ), t3 = tr(A3 ), t4 = tr(A4 ).

Can we reconstruct A from t1 , t2 , t3 , t4 . Does it depend on whether the matrix


A is normal?
(ii) Let A be a 2 2 matrix over C. Given

d1 = det(A), d2 = det(A2 ), d3 = det(A3 ), d4 = det(A4 ).

Can we reconstruct A from d1 , d2 , d3 , d4 . Does it depend on whether the matrix


A is normal?
Traces, Determinants and Hyperdeterminants 59

Problem 102. Let A = (ajk ) be an n n skew-symmetric matrix over R, i.e.


j, k = 1, . . . , n. Let B = (bjk ) be an n n symmetric matrix over R defined by
bjk = bj bk , i.e. j, k = 1, . . . , n. Let n be even. Show that

det(A + B) = det(A).

Problem 103. Consider the 3 3 matrix M with entries

(M )jk = xkj1 , j, k = 1, 2, 3

Find the determinant of this matrix.

Problem 104. Let  


s11 s12
S=
s21 s22
be an invertible matrix over R. Thus
 
1 1 s22 s12
S =
det(S) s21 s11

with det(S) = s11 s22 s12 s21 . Find the condition on the entries sjk such that
   
0 1 1 0 1
S S = .
1 0 1 0

Assume that det(S) = 1.

Problem 105. Let f11 , f22 , f33 , f44 be analytic functions fjj : R R.
Consider the 4 4 matrix
f11 0 0 0

1 f22 0 1
M =
0 0 f33 1

0 0 0
f11 f22 f33 0

where 0 denotes differentiation with respect to x. Find the determinant of the


matrix and write down the ordinary differential equation which follows from
det(M ) = 0. Find solutions of the differential equation.

Problem 106. (i) Find all 2 2 matrices A1 , A2 , A3 over C such that

tr(A1 ) = tr(A2 ) = tr(A3 ) = 0

and
tr(A1 A2 ) = tr(A2 A3 ) = tr(A3 A1 ) = 0.
60 Problems and Solutions

(ii) Find all 3 3 matrices B1 , B2 , B3 over C such that

tr(B1 ) = tr(B2 ) = tr(B3 ) = 0

and
tr(B1 B2 ) = tr(B2 B3 ) = tr(B3 B1 ) = 0.

Problem 107. Find all 2 2 matrices A such that

det(A) = 1, tr(A) = 0.

Do these matrices form a group under matrix multiplication?

Problem 108. Find the determinant of the matrix



0 cos() sin()
A() = cos() sin() 0 .
0 cos() sin()

Problem 109. Let n 1. Consider the 2n 2n matrices A and B. Assume


that A2 = I2n , B 2 = I2n and [A, B]+ AB + BA = 02n . Show that
n
det(A + B) = 22 /2
.

Problem 110. Let fj : R R (j = 1, . . . , n) be analytic functions. Consider


the determinant (Wronskian)
f1 (x) f2 (x) fn (x)
f10 (x) f20 (x) fn0 (x)
W (x) = det .. .. ..

. . .
(n1) (n1) (n1)
f1 (x) f2 (x) fn (x)

denotes fj (n 1) denotes the (n 1) derivative of fj . If the Wronskian of


these functions is not identially 0 on R, then these functions form a linearly
independent set. Let n = 3 and

f1 (x) = x, f2 (x) = ex , f3 (x) = e2x .

Find the Wronskian. Discuss.

Problem 111. Let A, B be nn matrices over C. Assume that B is invertible.


Show that there exists c C such that A + cB is not invertible.
Traces, Determinants and Hyperdeterminants 61

Hint. Start of with the identity A + cB (AB 1 + cIn )B and apply the
determinant.

Problem 112. The permanent and the determinant of an n n matrix M


over C are respectively defined as

X Y n X n
Y
perm(M ) := Mj,(j) , det(M ) := (1)sgn() Mj,(j)
Sn j=1 Sn j=1

where Sn denotes the symmetric group on a set of n symbols. For an n n


matrix A and an m m matrix B we know that

det(A B) (det(A))m (det(B))n

where denotes the Kronecker product. Study perm(A B).

Problem 113. Let A, B be 2 2 matrices over C. What are the conditions


on A, B such that
tr(AB) = tr(A)tr(B) ?

Problem 114. Let A be a 4 4 matrix we write as


 
A1 A2
A=
A3 A4

where A1 , A2 , A3 , A4 are the (block) 2 2 matrices in the matrix A. Consider


the map    
A1 A2 A1 AT2
7
A3 A4 AT3 A4
where T denotes the transpose. Is the trace preserved under this map? Is the
determinant preserved under this map?

Problem 115. (i) What is condition on R such that



1
F () = 1
1
is invertible?
(ii) What is the condition on , R such that
1

1
F (, ) =
1

1
62 Problems and Solutions

is invertible?

Problem 116. Let A, B be n n matrices over C. Then tr([A, B]) = 0.


Consider n = 3 and
1 0 0
C = 0 0 0
0 0 1
with tr(C) = 0. Construct 3 3 matrices A and B such that [A, B] = C.

Problem 117. Consider the k vectors


v
1j
v2j
vj =
..
j = 1, 2, . . . , k
.
vnj

in the Euclidean space Rn . The parallelepiped determined by these vectors has


the k-dimensional area q
det(V T V )
where V is the n k matrix with v1 , v2 , . . . , vk as its columns. Apply it to the
two vectors
1 1
1 1
v1 = 0 , v2 = 0 .
2 1 2 1

Problem 118. The equation of a hyperplane passing through the n points x1 ,


x2 , . . . , xn in the Euclidean space Rn is given by
 
1 1 1 ... 1
det = 0.
x x1 x2 . . . xn

Let n = 3. Apply it to the vectors



1 0 1
1 1
x1 = 0 , x2 = 1 , x3 = 0 .
2 1 0 2 1
Traces, Determinants and Hyperdeterminants 63

Hyperdeterminant

Problem 119. The hyperdeterminant Det(A) of the three-dimensional array


A = (aijk ) R222 can be calculated as follows
    
1 a000 a010 a100 a110
Det(A) = det +
4 a001 a011 a101 a111
   2
a000 a010 a100 a110
det
a001 a011 a101 a111
   
a000 a010 a100 a110
4 det det .
a001 a011 a101 a111
Assume that only one of the coefficients aijk is nonzero. Calculate the hyperde-
terminant.

Problem 120. Let 00 = 11 = 0, 01 = 1, 10 = 1, i.e. we consider the 2 2
matrix  
0 1
= .
1 0
Then the determinant of a 2 2 matrix A2 = (aij ) with i, j = 0, 1 can be defined
as
1 1 1 1
1 XXX X
det(A2 ) := ij `m ai` ajm .
2 i=0 j=0 m=0`=0

Thus
det(A2 ) = a00 a11 a01 a10 .
In analogy the hyperdeterminant of the 2 2 2 array A3 = (aijk ) with i, j, k =
0, 1 is defined as
1 1 1 1 1 1
1 X X X X X X
DetA3 := ii0 jj 0 kk0 mm0 nn0 pp0 aijk ai0 j 0 m anpk0 an0 p0 m0 .
2 0
ii =0 jj =0 kk =0 mm0 =0 nn0 =0 pp0 =0
0 0

Calculate DetA3 .

Problem 121. Given a 2 2 2 hypermatrix


A = (ajk` ), j, k, ` = 0, 1
and the 2 2 matrix  
s00 s01
S= .
s10 s11
The multiplication AS which is again a 2 2 hypermatrix is defined by
1
X
(AS)jk` := ajkr sr` .
r=0
64 Problems and Solutions

Assume that det(S) = 1, i.e. S SL(2, C). Show that Det(AS) = Det(A). This
is a typical problem to apply computer algebra. Write a SymbolicC++ program
or Maxima program that solves the problem.
Chapter 4

Eigenvalues and
Eigenvectors

Let A be an n n matrix over C. The eigenvalue problem is given by

Av = v

with v 6= 0, is called the eigenvalue and v is called the eigenvector. The


eigenvalues are found from the equation

det(A In ) = 0.

Problem 1. (i) Let A, B be 2 2 matrices over R and vectors x, y in R2 such


that
Ax = y, By = x
xT y = 0 and xT x = 1, yT y = 1. Show that AB and BA have an eigenvalue
+1.
(ii) Find all 2 2 matrices A, B which satisfy the conditions given in (i). Use
   
cos() sin()
x= , y= .
sin() cos()

Problem 2. Find the trace, rank, determinant and eigenvalues of the hermi-

65
66 Problems and Solutions

tian 4 4 matrix
0 0 0 1

0 0 0 1
A= .
0 0 0 1
1 1 1 0

Problem 3. Let A be an arbitrary 2 2 matrix. Show that

A2 Atr(A) + I2 det(A) = 0

and therefore
(trA)2 = tr(A2 ) + 2 det(A).

Hint. Apply the Cayley-Hamilton theorem.

Problem 4. Let A be an n n matrix. The matrix A is called nilpotent if


there is a positive integer r such that Ar = 0n .
(i) Show that the smallest integer r such that Ar = 0n is smaller or equal to n.
(ii) Find the characteristic polynomial of A.

Problem 5. Find all 2 2 matrices over R that admit only one eigenvector.

Problem 6. Let x be a nonzero column vector in Rn and n 2. Consider


the n n matrix xxT . Find one nonzero eigenvalue and the corresponding
eigenvector of this matrix.

Problem 7. Consider the 2 2 matrix


 
1 0
A(a) = , a R.
a 1

Can one find a condition on the parameter a so that A has only one eigenvector?

Problem 8. If { Aj }m j=1 is a commuting family of matrices that is to say


Aj Ak = Ak Aj for every pair from the set, then there exists a unitary matrix V
such that for all Aj in the set the matrix

ej = V Aj V
A

is upper triangular. Apply this to the matrices


   
1 1 1 1
A1 = , A2 = .
1 1 1 1
Eigenvalues and Eigenvectors 67

Problem 9. Consider the 2 2 matrix


 
1/4 1/2
A= .
1/2 1/4
Let (spectral radius)
(A) := max |j |
1j2
where j are the eigenvalues of A.
(i) Check that (A) < 1.
(ii) If (A) < 1, then
(I2 A)1 = I2 + A + A2 +
Calculate (I2 A)1 .
(iii) Calculate
(I2 A)(I2 + A + A2 + + Ak ).

Problem 10. Consider a symmetric 2 2 matrix A over R with a11 > 0,


a22 > 0, a12 < 0 and ajj > |a12 | for j = 1, 2. Is the matrix A positive definite?

Problem 11. Let A be a positive definite n n matrix. Show that A1 exists


and is also positive definite.

Problem 12. Let cj R. Find the eigenvalues of the matrices


0 1 0 0

  0 1 0
0 1 0 0 1 0
, 0 0 1 , .
c1 c2 0 0 0 1

c1 c2 c3
c1 c2 c3 c4
Generalize to the n n case.

Problem 13. Let n be a positive integer. Consider the 3 3 matrix with rows
of elements summing to unity

nab a b
1
M= a n 2a c a+c
n
c a nac
where the values of a, b, c are such that, 0 a, 0 b, a + b n, 2a + c n.
Thus the matrix is a stochastic matrix. Find the eigenvalues of M .

Problem 14. (i) Find the eigenvalues and normalized eigenvectors of the 3 3
matrix
0 1 0
M = 1 0 1.
0 1 0
68 Problems and Solutions

(ii) Use the normalized eigenvectors to construct a 3 3 matrix R such that


RM R1 is a diagonal matrix.
(iii) Can M be written as
X3
M= j vj vjT
j=1

where 1 , 2 , 3 are the eigenvalues and v1 , v2 , v3 are the (column) normalized


eigenvectors of M . Prove or disprove.

Problem 15. (i) Find the eigenvalues of the symmetric matrices

0 1 0 0 1

0 1 0 1

0 1 1 1 0 1 0 0
1 0 1 0
A3 = 1 0 1, A4 = , A5 = 0 1 0 1 0.

0 1 0 1
1 1 0 0 0 1 0 1

1 0 1 0
1 0 0 1 0

(ii) Extend the results from (i) to find the largest eigenvalue of the symmetric
n n matrix
0 1 0 0 0 1

1 0 1 0 0 0
0 1 0 0 0 0

An =
... ... ... .. .. .... .
. . . .
0 0 0 ... 1 0 1
1 0 0 ... 0 1 0

Problem 16. Find the eigenvalues of the 4 4 symmetric matrix

1 0 0

1/2 0
.
0 1/2

0 0 1

Discuss the eigenvalues j () as functions of . Can the eigenvalues cross as


function of ?

Problem 17. Consider the n n cyclic matrix


a a12 a13 a14 . . . a1n1 a1n
11
a1n a11 a12 a13 . . . a1n2 a1n1
a1n1 a1n a11 a12 . . . an3 an2

A=
.
. .. .. .. .. .. ..
. . . . . . .

a12 a13 a14 a15 ... a1n a11
Eigenvalues and Eigenvectors 69

where ajk R. Show that

2k

4k
1 ..
 ei/n ,

. , 1 k n.
n
2(n1)k
1

is a normalized eigenvector of A. Find the eigenvalues.

Problem 18. Let a, b R. Find on inspection two eigenvectors and the


corresponding eigenvalues of the 4 4 matrix

a 0 0 b

0 a 0 b
.
0 0 a b

b b b 0

Problem 19. Let a, b R. Find on inspection two eigenvectors and the


corresponding eigenvalues of the 4 4 matrix

a 0 0 b

0 a 0 b
.
0 0 a b

b b b 0

Problem 20. Let z C. Find the eigenvalues and eigenvectors of the 3 3


matrix

0 z z
A = z 0 z.
z z 0

Discuss the dependence of the eigenvalues on z.

Problem 21. Find the eigenvalues and normalized eigenvectors of the matrix
( [0, 2))
 
1 1 ei
A() = .
2 1 ei

Is the matrix invertible? Make the decision by looking at the eigenvalues. If so


find the inverse matrix.
70 Problems and Solutions

Problem 22. Consider the two permutation matrices

0 0 1 0 1 0 0 0

0 1 0 0 0 0 0 1
S= , T = .
1 0 0 0 0 0 1 0
0 0 0 1 0 1 0 0

Show that the two matrices have the same (normalized) eigenvectors. Find the
commutator [S, T ].

Problem 23. Consider the following 3 3 matrix A and vector v in R3



0 1 0 sin()
A = 1 0 1, v = sin(2)
0 1 0 sin(3)

where R and 6= n with n Z. Show that using this vector we can find
the eigenvalues and eigenvectors of A. Start of with Av = v.

Problem 24. Consider the symmetric matrix over R



2 1 1
A= 1 1 0 .
1 0 1

Find a invertible matrix B such that B 1 AB is a diagonal matrix.

Problem 25. Let 1 , 2 , 3 be the Pauli spin matrices. Consider the 4 4


gamma matrices
     
0 2 1 02 2 02 3
1 = , 2 = , 3 =
1 02 2 02 3 02

and  
I2 02
0 = .
02 I2
Find 1 2 3 0 and tr(1 2 3 0 ).

Problem 26. Let c R and consider the symmetric 3 3 matrix



c 1 0
A = 1 c 1.
0 1 c

(i) Show that c is an eigenvalue of A and find the corresponding eigenvector.


Eigenvalues and Eigenvectors 71

(ii) Find the two other eigenvalues and eigenvectors.

Problem 27. Let c R. Consider the symmetric 4 4 matrix

1 c 0 0

c 2 2c 0
A= .
0 2c 3 c
0 0 c 4

(i) Find the characteristic equation.


(ii) Show that

1 + 2 + 3 + 4 = 10
1 2 + 1 3 + 1 4 + 2 3 + 2 4 + 3 4 = 35 6c2
1 2 3 + 1 2 4 + 1 3 4 + 2 3 4 = 50 30c2
1 2 3 4 = 24 30c2 + c4

where 1 , 2 , 3 , 4 denote the eigenvalues.

Problem 28. Consider the matrix

1 0 0 0 1

0 1 1 1 0
A = 0 1 1 1 0.

0 1 1 1 0

1 0 0 0 1

(i) Find the rank of the matrix. Explain.


(ii) Find the determinant and trace of the matrix.
(iii) Find all eigenvalues of the matrix.
(iv) Find one eigenvector.
(v) Is the matrix positive semidefinite?

Problem 29. Find the eigenvalues of the 3 3 matrix



2 1 1
A = 1 2 1
1 1 2

using the trace and the determinant of the matrix and the information that two
eigenvalues are the same.

Problem 30. Let B be a 2 2 matrix with eigenvalues 1 and 2 . Find the


72 Problems and Solutions

eigenvalues of the 4 4 matrix


0 0 1 0

0 0 0 1
X= .
b11 b12 0 0
b21 b22 0 0
Let v be an eigenvector of B with eigenvalue . What can be said about an
eigenvector of the 4 4 matrix X given by eigenvector v and eigenvalue of B.

Problem 31. Consider the 2 2 identity matrix I2 and the 2 2 matrix


 
0 1
N= .
1 0
Find the eigenvalues 1 , 2 and the corresponding normalized eigenvectors u1
and u2 of I2 . Then find the eigenvalues 1 , 2 and the corresponding normalized
eigenvectors v1 and v2 of N . Using the normalized eigenvectors u1 , u2 and v1 ,
v2 form the 2 2 matrix

u1 v1 u1 v2

H= .
u2 v1 u2 v2
Find the eigenvalues and eigenvectors of H. Discuss.

Problem 32. Let A, B be two n n matrices over C. The set of all matrices
of the form A B with C is said to be a pencil. The eigenvalues of the
pencil are elements of the set (A, B) defined by
(A, B) := { z C : det(A zB) = 0 }.
If (A, B) and
Ax = Bx, x 6= 0
then x is referred to as an eigenvector of A B. Note that may be finite,
empty or infinite.

Let
0 0 0 1 1 0 0 1

0 0 1 0 1 0 1 1 0
A= , B= .
0 1 0 0 2 0 1 1 0
1 0 0 0 1 0 0 1
Find the eigenvalue of the pencil.

Problem 33. Let a, b R. Find the eigenvalues and eigenvectors of the 4 4


matrix
a 0 0 b

0 a b 0
M = .
0 b a 0
b 0 0 a
Eigenvalues and Eigenvectors 73

Problem 34. Let R. Find the eigenvalues and eigenvectors of the matrix

1 0
A() = 0 .
0 1

Discuss the dependence of the eigenvalues and eigenvectors of .

Problem 35. Let u, v be nonzero column vectors in Rn and u 6= v. Consider


the n n matrix A over R

A = uuT + uvT vuT vvT .

Find the nonzero eigenvalues of A and the corresponding eigenvector.

Problem 36. Let A be an n n matrix with eigenvalues 1 ,. . . , n . Let


c C \ {0}. What are the eigenvalues of cA?

Problem 37. Let A be an n n matrix over R. Let 1 , . . . , n be the


eigenvalues. What can be said about the eigenvalues of the 2n 2n matrix
 
0n A
AT 0 n

where 0n is the n n zero matrix?

Problem 38. Let A, B be n n matrices over C. Let , C. Assume


that A2 = In and B 2 = In and AB + BA = 0. What can be said about the
eigenvalues of A + B?

Problem 39. Let A be an n n normal matrix, i.e. AA = A A. Let u be an


eigenvector of A, i.e. Au = u. Show that u is also an eigenvector of A with
eigenvalue , i.e.
A u = u.

Problem 40. Show that eigenvectors of a normal matrix A corresponding to


distinct eigenvalues are orthogonal.

Problem 41. Let A, B be square matrices. Show that AB and BA have the
same eigenvalues.

Problem 42. Show that if A is an n m matrix and if B is an m n matrix,


then 6= 0 is an eigenvalue of the n n matrix AB if and only if is an
74 Problems and Solutions

eigenvalue of the m m matrix BA. Show that if m = n then the conclusion is


true even for = 0.

Problem 43. Let AT = (1/2 , 1/2)T . Find the eigenvalues of AAT and AT A.

Problem 44. We know that a hermitian matrix has only real eigenvalues.
Can we conclude that a matrix with only real eigenvalues is hermitian?

Problem 45. Let A be an n n matrix over C. Show that the eigenvalues of


A A are nonnegative.

Problem 46. Let 0 x < 1. Consider the N N matrix C (correlation


matrix) with the entries
Cjk := x|jk| , j, k = 1, . . . , N.
Find the eigenvalues of C. Show that if N the distribution of its eigenvalues
becomes a continuous function of [0, 2]
1 x2
() = .
1 2x cos + x2

Problem 47. Let n be a positive integer. Consider the 2 2 matrix


 
2n 4n2 1
Tn = .
1 2n
Show that the eigenvalues of Tn are real and not of absolute value 1.

Problem 48. Let A be an n n matrix over C. Show that A is normal if and


only if there exists an n n unitary matrix U and an n n diagonal matrix D
such that D = U 1 AU . Note that U 1 = U .

Problem 49. Let A be a normal n n matrix over C.


(i) Show that A has a set of n orthonormal eigenvectors.
(ii) Show that if A has a set of n orthonormal eigenvectors, then A is normal.

Problem 50. The Leverriers method finds the characteristic polynomial of


an n n matrix. Find the characteristic polynomial for
   
0 1 1 1
A B, A= , B=
1 0 1 1
using this method. How are the coefficients ci of the polynomial related to the
eigenvalues?
Eigenvalues and Eigenvectors 75

Problem 51. Consider the symmetric matrix



a11 a12 a13
A = a12 a22 a23
a13 a23 a33

over R. Write down the characteristic polynomial det(I3 A) and express it


using the trace and determinant of A.

Problem 52. Let Ln be the n n matrix


n1 1 1 1

1 n 1 1 1
. .. .. .. ..
. . .
Ln = . . . .

..
1 1 1 . 1
1 1 1 n1
Find the eigenvalues.

Problem 53. The Pascal matrix of order n is defined as


 
(i + j 2)!
Pn := , i, j = 1, . . . , n.
(i 1)!(j 1)!
Thus
1 1 1 1

  1 1 1
1 1 1 2 3 4
P2 = , P3 = 1 2 3, P4 = .
1 2 1 3 6 10
1 3 6
1 4 10 20
(i) Find the determinant of P2 , P3 , P4 . Find the inverse of P2 , P3 , P4 .
(ii) Find the determinant for Pn . Is Pn an element of the group SL(n, R)?

Problem 54. Let A be an m n matrix (m < n) over R.


(i) Show that at least one eigenvalue of the n n matrix AT A is equal to 0.
(ii) Show that the eigenvalues of the m m matrix AAT are also eigenvalues of
AT A.

Problem 55. Find the determinant and eigenvalues of the matrices


0 0 0 a14

  0 0 a13
0 a12 1 0 0 a24
A2 = , A3 = 1 0 a23 , A4 = .
1 a22 0 1 0 a34
0 1 a33
0 0 1 a44
Extend to the n-dimensional case.
76 Problems and Solutions

Problem 56. Let j = 1/2, 1, 3/2, 2, . . . and R. Consider the (2j + 1)


(2j + 1) matrices
0 1

0 0 1
0 0 0 1


H= . ..



1
ei 0 0
D = diag(1, , 2 , . . . , 2j )
where := exp(i2/(2j + 1). Is H unitary? Find DH HD.

Problem 57. (i) Find the eigenvalues of the 3 3 matrix



e 1 1
A() = 1 e 1 .
1 1 e

For which values of is the matrix A() not invertible.


(ii) Extend (i) to the n n matrix

e 1 1

1 e 1
B() =
... .. .. .. .
. . .
1 1 e

This matrix plays a role for the Potts model.

Problem 58. Let A be an 2 2 matrix over R. Let

tr(A) = c1 , tr(A2 ) = c2 .

Can det(A) be calculated from c1 , c2 ?

Problem 59. Let A be an n n matrix with entries ajk 0 and with positive
spectral radius . Then there is a (column) vector x with xj 0 and a (column)
vector y such that the following conditions hold:

Ax = x, yT A = y, yT x = 1.

Consider the 2 2 matrix  


2 1
B= .
1 2
Show that B has a positive spectral radius. Find the vectors x and y.
Eigenvalues and Eigenvectors 77

Problem 60. Let A be an n n matrix over C. If is not an eigenvalue of


A, then the matrix (A In ) has an inverse, namely the resolvent

R = (A In )1 .

Let j be the eigenvalues of A. For || a, where a is any positive constant


greater than all the |j | the resolvent can be expanded as
 
1 1 1
R = I n + A + 2 A2 + .

Calculate I
1
m R d, m = 0, 1, 2, . . . .
2i ||=a

Problem 61. Show that the resolvent satisfies the so-called resolvent equation

R R = ( )R R .


Problem 62. Let = (1 + 5)/2 be the golden ratio. Consider the modular
matrix  
1 1
M= .
1 0
Find the eigenvalues 1 , 2 and the corresponding normalized eigenvectors u1 ,
u2 . Find the projection matrices 1 and 2 onto the associated eigendirections.

Problem 63. Let A be an n n matrix over C. Assume that A2 = In .


What can be said about the eigenvalues of A?

Problem 64. An n n unitary matrix U is defined by

U U = In or U = U 1 .

What can be concluded about the eigenvalues of U if U = U T ?

Problem 65. Let R. Consider the symmetric matrix


1+ 0 1 0

1 0 1+ 0 1
A= .
2 1 0 1+ 0
0 1 0 1+
Find an inertible matrix B such that

A = B 1 DB
78 Problems and Solutions

where D is a diagonal matrix and thus find the eigenvalues of A.

Problem 66. The additive inverse eigenvalue problem is as follows: Let A


be an n n symmetric matrix over R with ajj = 0 for j = 1, 2, . . . , n. Find a
real diagonal n n matrix D such that the matrix A + D has the prescribed
eigenvalues 1 , . . . , n . The number of solutions for the real matrix D varies
from 0 to n!. Consider the 2 2 matrix
 
0 1
A=
1 0

and the prescribed eigenvalues 1 = 2, 2 = 3. Can one find a D?

Problem 67. Let A be an n n matrix over A. Assume that A is normal.


Show that A has a set of n orthonormal eigenvectors.

Problem 68. Let R. Consider the n n matrix

0 1

0 1
0 1


H= .. .

.

1
ei 0

(i) Show that the matrix is unitary.


(ii) Find the eigenvalues of H.
(iii) Consider the n n diagonal matrix

G = diag(1, , 2 , n1 )

where := exp(i2/n. Find GH HG.

Problem 69. Consider the symmetric 6 6 matrix over R

0 1 1 1 1 1

1 0 1 1 1 1
1 1 0 1 1 1

A= .
1 1 1 0 1 1
1 1 1 1 0 1

1 1 1 1 1 0

This matrix plays a role in the construction of the icosahedron which is a reg-
ular polyhedron with 20 identical equilateral triangular faces, 30 edges and 12
vertices.
Eigenvalues and Eigenvectors 79

(i) Find the eigenvalues of this


matrix.
(ii) Consider the matrix
A + 5I6 . Find the eigenvalues.
(iii) The matrix
A + 5I 6 induces an Euclidean structure
on the quotient space
R6 /ker(A + 5I6 ). Find the dimension of ker(A + 5I6 ).

Problem 70. Let R. Find the eigenvalues and eigenvectors of


 
1 1
A() = .
1 + 2 1
For which is A() not invertible?

Problem 71. Let  R. Find the eigenvalues and eigenvectors of the matrices
0 1 0 0

  0 1 0
0 1 0 0 1 0
, 0 0 1, .
 0 0 0 0 1
 0 0
 0 0 0
Extend to the n n case.

Problem 72. Let A, B be two n n matrices over C.


(i) Show that every eigenvalue of AB is also an eigenvalue of BA.
(ii) Can we conclude that every eigenvector of AB is also an eigenvector of BA?

Problem 73. (i) Find the eigenvalues and eigenvectors of the orthogonal
matrices

1 0 0 cos sin 0
R = 0 cos sin , S = sin cos 0.
0 sin cos 0 0 1
(ii) Find the eigenvalues and eigenvectors of RS.

Problem 74. Find the eigenvalues and normalized eigenvectors of the 2 2


matrix  
cos ei sin
.
ei sin cos

Problem 75. Find the eigenvalues and eigenvectors of the matrices


1 0 0 0 1


1 0 1 0 1 0 1 0
0 1 0 , 0 0 1 0

0 .

1 0 1 0 1 0 1 0

1 0 0 0 1
80 Problems and Solutions

Extend to the general case n odd.

Problem 76. (i) Consider the matrices



  a1 0 a2
a1 a2
A= , B= 0 0 0 .
a3 a4
a3 0 a4

How are the eigenvalues of A and B related?


(ii) Let  
11 12
=
21 22
be a density matrix. Is
11 0 12
= 0 0 0
21 0 22
a density matrix?

Problem 77. Let a, b, c R. Find the eigenvalues of the 4 4 matrix


0 a b c

a 0 ic ib
A= .
b ic 0 ia
c ib ia 0

Problem 78. Find all 2 2 matrices over the real numbers with only one
1-dimensional eigenspace, i.e. all eigenvectors are linearly dependent.

Problem 79. Let A be an n n matrix over Cn . Let be an eigenvalue of


A. A generalized eigenvector x Cn of A corresponding to the eigenvalue is
a nontrivial solution of
(A In )j x = 0n
for some j {1, 2, . . .}, where In is the n n identity matrix and 0n is the
n-dimensional zero vector. For j = 1 we find the eigenvectors. It follows that x
is a generalized eigenvector of A corresponding to if and only if

(A In )n x = 0n .

Find the eigenvectors and generalized eigenvectors of



0 1 0
0 0 0.
0 1 0
Eigenvalues and Eigenvectors 81

Problem 80. Find all 2 2 matrices over R which commute with


 
0 1
.
0 0
What is the relation between the eigenvectors of these matrices?

Problem 81. Let n be odd and n 3. Consider the matrices



1/ 2 0 0 0 1/ 2
1/ 2 0 1/ 2 0 1/ 2 0 1/ 2 0
A3 = 0 1 0 , A5 = 0 0 1 0 0

1/ 2 0 1/ 2 0 1/ 2 0 1/ 2 0

1/ 2 0 0 0 1/ 2
and generally

1/ 2 0 ... 0 0 0 ... 0 1/ 2
0 1/ 2 ... 0 0 0 ... 1/ 2 0
.. .. .. .. .. .. ..


. . ... . . . ... . .



0 0 ... 1/ 2 0 1/ 2 ... 0 0

An = 0 0 ... 0 1 0 ... 0 0 .

0
0 ... 1/ 2 0 1/ 2 ... 0 0

. .. .. .. .. .. ..
..

. ... . . . ... . .

0
1/ 2 ... 0 0 0 . . . 1/ 2 0
1/ 2 0 ... 0 0 0 ... 0 1/ 2
Find the eigenvalues and eigenvectors of A3 , A5 . The solve the general case.

Problem 82. Assume we know the eigenvalues 1 , 2 of the 2 2 matrix


 
a11 a12
A=
a21 a22
over C. What can be said about the eigenvalues of the 3 3 matrix

a11 0 a12
B= 0 c 0
a21 0 a22
where c C.

Problem 83. Let  R. Find the eigenvalues and eigenvectors of the matrices
1 0 0 

  1 0 
1  1 1 0 0
, 1 1 0, .
1 1 1 1 1 0
1 1 1
1 1 1 1
82 Problems and Solutions

Extend to n n matrices.

Problem 84. Let A, B, C, D, E, F , G, H be 2 2 matrices over C. We


define the product
A O2 O2 B

   
A B E F 02 E F 02
? := .
C D G H 02 G H 02
C 02 02 D
Thus the right-hand side is an 8 8 matrix. Assume we know the eigenvalues
and eigenvectors of the two 4 4 matrices on the left-hand side. What can be
said about the eigenvalues and eigenvectors of the 8 8 matrix of the right-hand
side.

Problem 85. The symmetric 3 3 matrix



0 1 1
A = 1 0 1
1 1 0

plays a role for the chemical compounds ZnS and N aCl. Find the eigenvalues
and eigenvectors of A. Then find the inverse of A. Find all x such that Ax = x.

Problem 86. Let  


a11 a12
A=
a21 a22
be a normal matrix over C with eigenvalues 1 , 2 and corresponding eigenvec-
tors u1 , u2 , respectively. What can be said about the eigenvalues and eigenvec-
tors of the 3 3 matrices

1 0 0 a11 0 a12 a11 a12 0
B1 = 0 a11 a12 , B2 = 0 1 0 , B3 = a21 a22 0 ?
0 a21 a22 a21 0 a22 0 0 1

Problem 87. Let In be the n n identity matrix and


 
0n In
J := R2n2n .
In 0n

A matrix S R2n2n is called a symplectic matrix if

S T JS = J.

(i) Show that symplectic matrices are nonsingular.


Eigenvalues and Eigenvectors 83

(ii) Show that the product of two symplectic matrices S1 and S2 is also sym-
plectic.
(iii) Show that if S is symplectic S 1 and S T are also symplectic.
(iv) Let S be a symplectic matrix. Show that if (S), then 1 (S),
where (S) denotes the spectrum of S.

Problem 88. Let A, B be n n matrices over C and u a nonzero vector in


Cn . Assume that [A, B] = A and Au = u. Find (AB)u.

Problem 89. Consider the Hilbert space Cn . Let A, B, C be n n matrices


acting in Cn . We consider the nonlinear eigenvalue problem

Au = Bu + 2 Cu

where u Cn and u 6= 0.
(i) Let 1 , 2 , 3 be the Pauli spin matrices. Find the solutions of the nonlinear
eigenvalue problem
1 u = 2 u + 2 3 u

where u C2 and u 6= 0.
(ii) Consider the basis of the simple Lie algebra s`(2, R)
     
1 0 0 1 0 0
H= , E= , F = .
0 1 0 0 1 0

Solve the nonlinear eigenvalue problem

Hu = Eu + 2 F u

where u C2 and u 6= 0.
(iii) Consider the basis of the simple Lie algebra so(3, R)

0 0 0 0 0 1 0 1 0
A = 0 0 1, B = 0 0 0 , C = 1 0 0.
0 1 0 1 0 0 0 0 0

Solve the nonlinear eigenvalue problem.

Problem 90. (i) Let A, B be n n matrices over c C with [A, B] = 0,


where [A, B] denotes the commutator of A and B. Calculate [A + cIn , B + cIn ],
where c C and In is the n n identity matrix.
(ii) Let x be an eigenvector of the n n matrix A with eigenvalue . Show that
x is also an eigenvector of A + cIn , where c C.
84 Problems and Solutions

Problem 91. Consider the n n tridiagonal matrix

1 1 . .

1 2 1 .
1 3 1 .


H = .. .

.

. 1
1 n

It is used to describe an electron on a linear chain of length n. Find the eigen-


values. Find the eigenvectors. Make the ansatz

c1

c2
.
..
cn

for the eigenvectors and find a recursion relation for cj /cj+1 .

Problem 92. Find the eigenvalues and eigenvectors of the Hamilton operator

H = E0 I2 B1 1 B2 2 B3 3 .

Problem 93. Let  R. Find the eigenvalues of



0  0
A() =  0  .
0  0

Do the eigenvalues cross as a function of ?

Problem 94. Let  [0, 1]. Consider the 2 2 matrix


 
1 1 
A() = .
1 + 2  1

For  = 0 we have the Pauli spin matrix 3 and for  = 1 we have the Hadamad
matrix. Find the eigenvalues and eigenvectors of A().

Problem 95. (i) Consider the matrix


 
0 i
A= .
i 0
Eigenvalues and Eigenvectors 85

Find the function (characteristic polynomial)

p() = det(A I2 ).

Find the eigenvalues of A by solving p() = 0. Find the minima of the function

f () = |p()|.

Discuss.
(ii) Consider the matrix
0 0 1
A = 0 1 0.
1 0 0
Find the function (characteristic polynomial)

p() = det(A I3 ).

Find the eigenvalues of A by solving p() = 0. Find the minima of the function

f () = |p()|.

Discuss.

Problem 96. Let A be an n n normal matrix with eigenvalues 1 , . . . , n


and pairwise orthogonal eigenvectors uj (j = 1, 2, . . . , n). Then
n
X
A= j uj uj .
j=1

Find exp(A) and sin(A).

Problem 97. Consider the normalized vector in C3



sin() cos()
n = sin() sin() .
cos()

(i) Calculate the 2 2 matrix

U (, ) = n n1 1 + n2 2 + n3 3

where 1 , 2 , 3 are the Pauli spin matrices.


(ii) Is the matrix U (, ) unitary? Find the trace and the determinant. Is the
matrix U (, ) hermitian?
(iii) Find the eigenvalues and normalized eigenvectors of U (, ).
86 Problems and Solutions

Problem 98. Let A be an nn normal matrix. Assume that j (j = 1, . . . , n)


are the eigenvalues of A. Calculate
n
Y
(1 + k )
k=1

without using the eigenvalues.

Problem 99. Let A be an n n matrix over C. Then any eigenvalue of A


satisfies the inequality
n
X
|| max |ajk |.
1jn
k=1

Write a C++ program that calculates the right-hand side of the inequality for
a given matrix. Apply the complex class of STL. Apply it to the matrix

i 0 0 i

0 2i 2i 0
A= .
0 3i 3i 0
4i 0 0 4i

Problem 100. (i) Find the eigenvalues of the matrices

1 0 0 1

  1 0 1
1 1 0 2 2 0
A2 = , A3 = 0 2 0, A4 = .
2 2 0 3 3 0
3 0 3
4 0 0 4

Extend to the n n case.


(ii) Find the eigenvalues of the matrices

1 0 0 1

  1 0 1
1 1 0 1/2 1/2 0
B2 = , B3 = 0 1/2 0 , B4 = .
1/2 1/2 0 1/3 1/3 0
1/3 0 1/3
1/4 0 0 1/4

Extend to the n n case.

Problem 101. The 2n 2n symplectic matrix is defined by


 
0n In
S=
In 0n

where In is the n n identity matrix. The matrix S is unitary and skew-


hermitian. Find the eigenvalues of S from this information.
Eigenvalues and Eigenvectors 87

Problem 102. Find the condition on a11 , a12 , b11 , b12 such that
a11 0 0 a12 1 1

0 b11 b12 0 1 1
=
0 b12 b11 0 1 1

a12 0 0 a11 1 1
i.e. we have an eigenvalue equation.

Problem 103. Find the eigenvalues of the 4 4 matrix


0 0 0 a14

1 0 0 a24
A= .
0 1 0 a34
0 0 1 a44

Problem 104. Find the eigenvalues and eigenvectors of the 4 4 matrix


0 a12 a13 a14

a 0 a23 a24
A = 12 .
a13 a23 0 a34
a14 a24 a34 0

Problem 105. Find the eigenvalues and eigenvectors of the 4 4 matrix


0 a12 a13 a14

a12 0 a23 a24
A= .
a13 a23 0 a34
a14 a24 a34 0

Problem 106. Find the eigenvalues and eigenvectors of 1 2 3 .

Problem 107. Find the eigenvalues of the 7 7 matrix


a11 0 0 a14 0 0 a17

0 a22 a 23 0 a25 a26 0
0 a32 a33 0 a35 a36 0

A = a41 0 0 a44 0 0 a47 .

0 a a 0 a55 a56 0

52 53
0 a62 a63 0 a65 a66 0

a71 0 0 a74 0 0 a77

Problem 108. (i) Find the eigenvalues and eigenvectors of


 
1 i
3 + i1 = .
i 1
88 Problems and Solutions

(ii) Is this matrix normal?

Problem 109. Let A be an n n matrix over C, which satisfies

A2 AA = cA

where c C is a constant. Obviously the equation is satisfied by the zero matrix


with c = 0. Assume that A 6= 0n . Then we have a type of eigenvalue equation.
(i) Is c an eigenvalue of A.
(ii) Take the determinant of both sides of the equation. Discuss. Study the cases
that A is invertible and non-invertible.
(iii) Study the case  z 
e 1
A(z) = , z C.
1 ez
(iv) Study
(A A)2 = c(A A).
(v) Let A be a 2 2 matrix and

a11 0 0 a12

0 a11 a12 0
A ? A := .
0 a21 a22 0
a21 0 0 a22

Study the case (A ? A)2 = c(A ? A).


(vi) Study the case that A3 = cA.

Problem 110. (i) Consider the Pauli spin matrices for describing a spin- 21
system      
0 1 0 i 1 0
1 = , 2 = , 3 = .
1 0 i 0 0 1
Consider the matrix
3 + i1 .
Is the matrix normal? Find the eigenvalues and eigenvectors of the matrix.
Discuss. Find the eigenvalues and eigenvectors of 3 3 + i1 1 .
(ii) Consider the Pauli spin matrices for describing a spin-1 system

0 1 0 0 i 0 1 0 0
1 1
s1 = 1 0 1 , s2 = i 0 i , s3 = 0 0 0 .
2 0 1 0 2 0 i 0 0 0 1

Consider the matrix


s3 + is1 .
Eigenvalues and Eigenvectors 89

Is the matrix normal? Find the eigenvalues and eigenvectors of the matrix.
Discuss. Find the eigenvalues and eigenvectors of s3 s3 + is1 s1 .

Problem 111. Let s1 , s2 , s3 be the (2s + 1) (2s + 1) spin matrices for spin
s = 1/2, s = 1, s = 3/2, s = 2, . . ..
(i) For s = 1/2 we have the 2 2 matrices
     
1 0 1 1 0 i 1 1 0
s1 = , s2 = , s3 = .
2 1 0 2 i 0 2 0 1

Let n R3 and knk = 1. Calculate the eigenvalues and eigenvectors of

n1 s1 + n2 s2 + n3 s3 .

(ii) For s = 1 we have the 3 3 matrices



0 1 1 0 i 0 1 0 0
1 1 1
s1 = 1 0 1 , s2 = i 0 0 , s3 = 0 0 0 .
2 2 2
1 0 1 0 i 0 0 0 1

Let n R3 and knk = 1. Calculate the eigenvalues and eigenvectors of

n1 s1 + n2 s2 + n3 s3 .

Problem 112. Find the eigenvalues and eigenvectors of the matrices


0 0 0 a14

0 0 a13
1 0 a23 , 1 0 0 a24
.
0 1 0 a34

0 1 a33
0 0 1 a44

Problem 113. Let K be an n n skew-hermitian matrix with eigenvalues 1 ,


. . . , n (counted according to multiplicity) and the corresponding normalized
eigenvectors u1 , . . . , un , where uj uk = 0 for k 6= j. Then K can be written as
n
X
K= j uj uj
j=1

and uj uj uk uk = 0 for k 6= j and j, k = 1, 2, . . . , n. Note that the matrices uj uj


are projection matrices and
Xn
uj uj = In .
j=1

(i) Calculate exp(K).


90 Problems and Solutions

(ii) Every n n unitary matrix can be written as U = exp(K), where K is a


skew-hermitian matrix. Find U from a given K.
(iii) Use the result from (ii) to find for a given U a possible K.
(iv) Apply the result from (ii) and (iii) to the unitary 2 2 matrix
 
cos sin
U () = .
sin cos

(v) Apply the result from (ii) and (iii) to the 2 2 unitary matrix
 
cos ei sin
V (, ) = .
ei sin cos

(vi) Every hermitian matrix H can be written as H = iK, where K is a skew-


hermitian matrix. Find H for the examples given above.

Problem 114. Consider a symmetric matrix over R. We impose the following


conditions. The diagonal elements are all zero. The non-diagonal elements can
only be +1 or 1. Show that such a matrix can only have integer values as
eigenvalues. An example would be

0 1 1 1

1 0 1 1
1 1 0 1

1 1 1 0

with eigenvalues 3 and 1 (three times).

Problem 115. Let A be an nn normal matrix over C. How would one apply
genetic algorithms to find the eigenvalues of A. This means we have to construct
a fitness function f with the minima as the eigenvalues. The eigenvalue equation
is given by Ax = zx (z C and x Cn with x 6= 0). The characteristic equation
is
p(z) det(A zIn ) = 0.
What would be a fitness function? Apply it to the matrices

    0 0 1
0 i 0 i
B= , C= , D = 0 1 0.
i 0 i 0
1 0 0

Problem 116. Let A, B be hermitian matrices over C and eigenvalues 1 ,


. . . , n and 1 , . . . , n , respectively. Assume that tr(AB) = 0 (scalar product).
What can be said about the eigenvalues of A + B?
Eigenvalues and Eigenvectors 91

Problem 117. Consider the skew-symmetric matrix over R



0 a3 a2
A = a3 0 a1
a2 a1 0

where a1 , a2 , a3 R. Find the eigenvalues. Let 03 be the 3 3 zero matrix. Let


A1 , A2 , A3 be skew-symmetric 3 3 matrices over R. Find the eigenvalues of
the 9 9 matrix
03 A3 A2
B = A3 03 A1 .
A2 A1 03

Problem 118. Consider the 4 4 Haar matrix


1 1 1 1

1 1 1 1 1
K = .
2 2 2 0 0

0 0 2 2

Find all 4 4 hermitian matrices H such that KHK T = H.

Problem 119. Consider the reverse-diagonal n n matrix

0 0 ... 0 ei1

0 0 . . . ei2 0
. ..
.

A(1 , . . . , n ) =
. .

0 ein1 ... 0 0
ein 0 ... 0 0

where j R (j = 1, . . . , n). Find the eigenvalues and eigenvectors. Is the


matrix unitary?

Problem 120. Let a11 , a22 R and a12 C. Consider the hermitian matrix
 
a11 a12
H=
a12 a22

with the real eigenvalues 1 and 2 . What conditions are impost on the matrix
elements of H if 1 = 2 ?

Problem 121. (i) Consider the spin matrices for describing a spin- 12 system
     
1 0 1 1 0 i 1 1 0
s1 = , s2 = , s3 =
2 1 0 2 i 0 2 0 1
92 Problems and Solutions

and the spin matrices for describing a spin-1 system



0 1 0 0 i 0 1 0 0
1 1
p1 = 1 0 1 , p2 = i 0 i , p3 = 0 0 0 .
2 0 1 0 2 0 i 0 0 0 1
Find the spectrum (eigenvalues and eigenvector) of the hermitian matrix
H
K = = s1 p1 s1 + s2 p2 s2 + s3 p3 s3 .
~
Thus K is a 12 12 matrix with tr(K) = 0.

Problem 122. s`(3, R) is the rank 2 Lie algebra with Cartan matrix
 
2 1
C= .
1 2
Find the eigenvalues and normalized eigenvectors of C.

Problem 123. Find the eigenvalues and normalized eigenvectors of the 4 4


matrix
a11 a12 1 0

a a22 0 1
A = 21 .
1 0 0 0
0 1 0 0

Problem 124. Let k R. Consider the matrices


0 ei1 0 0

i2
e 0 0 0
A(1 , 2 , 3 , 4 ) = ,
0 0 0 ei3
i4
0 0 e 0
0 0 ei5 0

0 0 0 ei6
B(5 , 6 , 7 , 8 ) = i7
e 0 0 0

0 ei8 0 0
and A(1 , 2 , 3 , 4 )B(5 , 6 , 7 , 8 ). Find the eigenvalues of these matrices.

Problem 125. Let U be an n n unitary matrix, i.e. U U = In . Assume


that U = U T . What can be said about the eigenvalues of such a matrix?

Problem 126. Let In be the n n identity matrix. Find the eigenvalues of


the 2n 2n matrix  
1 In In
R= .
2 In In
Eigenvalues and Eigenvectors 93

Problem 127. Consider the unitary matrix


0 0 0 1

   
0 0 1 0 0 1 0 1
U = = .
0 1 0 0 1 0 1 0
1 0 0 0
Find the skew-hermitian matrix K such that U = exp(K).

Problem 128. Given the matrix


1 2 3 4

5 6 7 8
A= .
9 10 11 12
13 14 15 16
Prove or disprove that exactly two eigenvalues are 0.

Problem 129. Let A be an n n matrix over C. Show that the eigenvectors


corresponding to distinct eigenvalues are linearly independent.

Problem 130. We know that any n n unitary matrix has only eigenvalues
with || = 1. Assume that a given n n matrix has only eigenvalues with
|| = 1. Can we conclude that the matrix is unitary?

Problem 131. Find the eigenvalues and eigenvectors of


0 a12 a13 a14

a 0 0 0
A = 12 .
a13 0 0 0
a14 0 0 0

Problem 132. (i) Let tj R for j = 1, 2, 3, 4. Find the eigenvalues and


eigenvectors of
0 t1 0 t4 ei

t1 0 t2 0
H = .
0 t2 0 t3
t4 ei 0 t3 0
(ii) Let tj R for j = 1, . . . , 5. Find the eigenvalues and eigenvectors of
0 t1 0 0 t5 ei

t1 0 t2 0 0
H = 0 t2 0 t3 0 .

0 0 t3 0 t4

t5 ei 0 0 t4 0
94 Problems and Solutions

Problem 133. Let v be a nonzero column vector in Rn . Matrix multiplication


is associative. Then we have

(vvT )v = v(vT v).

Discuss.

Problem 134. (i) Find the eigenvalues and eigenvectors of the matrix

a11 0 0 0

a21 a22 0 0
A= .
0 a32 a33 0
0 0 a43 a44

(ii) Find the eigenvalues and eigenvectors of the matrix

b11 0 0 0 0 0

b21 b22 0 0 0 0
0 b32 b33 0 0 0

B= .
0 0 b43 b44 0 0
0 0 0 b54 b55 0

0 0 0 0 b65 b66

These matrices are the so-called staircase matrices. Extend the results to the
n n case.

Problem 135. (i) Let A be an invertible nn matrix over C. Assume we know


the eigenvalues and eigenvectors of A. What can be said about the eigenvalues
and eigenvectors of A + A1 ?
(ii) Apply the result from (i) to the permutation matrix

0 0 0 0 0 1

1 0 0 0 0 0
0 1 0 0 0 0

A= .
0 0 1 0 0 0
0 0 0 1 0 0

0 0 0 0 1 0

Problem 136. (i) Find the eigenvalues and eigenvectors of the 4 4 matrix

a11 0 0 a14

0 a22 0 0
A= .
0 0 a33 0
a41 0 0 a44
Eigenvalues and Eigenvectors 95

(ii) Find the eigenvalues and eigenvectors of the 6 6 matrix

b11 0 0 0 0 a16

0 b22 0 0 0 0
0 0 b33 0 0 0

B= .
0 0 0 b44 0 0
0 0 0 0 b55 0

b61 0 0 0 0 b66

Problem 137. Find the eigenvalues and eigenvectors of the matrices



  1 1/2 1/3
1 1/2
, 1/2 1/3 1/4 .
1/2 1/3
1/3 1/4 1/5

Extend to the n n case.

Problem 138. Find the eigenvalues of the 6 6 matrix

1 1/2 1/3 1/4 1/5 1/6



1/2 1/3 1/4 1/5 1/6 1/7
1/3 1/4 1/5 1/6 1/7 1/8

A= .
1/4 1/5 1/6 1/7 1/8 1/9
1/5 1/6 1/7 1/8 1/9 1/10

1/6 1/7 1/8 1/9 1/10 1/11

Problem 139. (i) Let R. Consider the matrices


   
cos sin cos sin
A() = , B() = .
sin cos sin cos

Find the trace and determinant of these matrices. Show that for the matrix
A() the eigenvalues depend on but the eigenvectors do not. Show that for
the matrix B() the eigenvalues do not depend on but the eigenvectors do.
(ii) Let R. Consider the matrices
   
cosh sinh cosh sinh
C() = , D() = .
sinh cosh sinh cosh

Find the trace and determinant of these matrices. Show that for the matrix
C() the eigenvalues depend on but the eigenvectors do not. Show that for
the matrix D() the eigenvalues do not depend on but the eigenvectors do.
96 Problems and Solutions

Problem 140. Find the lowest eigenvalue of the 4 4 symmetric matrix


(x R)
0 x 5 0 0

x 5 4 2x 2x
.
0 2x 4 2x x

0 2x x 8 2x

Problem 141. Let m > 0 and R. Consider the three 3 3 matrices



0 sin 0 0 0 sin
M1 = m sin 0 cos , M2 = m 0 0 cos ,
0 cos 0 sin cos 0

0 sin cos
M3 = m sin 0 0 .
cos 0 0
Find the eigenvalues and eigenvectors of the matrices. These matrices play a
role for the Majorana neutrino.

Problem 142. Let A, B be real symmetric and block tridiagonal 44 matrices

a11 a12 0 0 b11 b12 0 0



a a22 a23 0 b b22 b23 0
A = 12 , B = 12 .

0 a23 a33 a34 0 b23 b33 b34
0 0 a34 a44 0 0 b34 b44

Assume that B is positive definite. Solve the eigenvalue problem

Av = Bv.

Problem 143. Let A be an n n matrix over C. What is the condition on A


such that all eigenvalues are 0 and A admits only one eigenvector.

Problem 144. Let A be a 22 matrix. Assume that det(A) = 0 and tr(A) = 0.


What can be said about the eigenvalues of A. Is such a matrix normal?

Problem 145. Let a, b, c R. Consider the symmetric matrix



0 a b
a 0 c.
b c 0

Find the eigenvalues and eigenvectors.


Eigenvalues and Eigenvectors 97

Problem 146. Let In be the n n unit matrix and I2 the 2 2 unit matrix.
Consider the n matrix
0 1 0 ... 0

0 0 1 0 ... 0

Jn = .. ..
.
. .
1
0
Hence an arbitrary Jordan block is given by zIn + Jn , where z C. Find the
eigenvalues of  
a b
In + I2 Jn .
b a

Problem 147. (i) Let I2 be the 2 2 identity matrix and 3 be the Pauli spin
matrix. Find the eigenvalues and normalized eigenvectors of the 4 4 matrix
 
cosh(2t)I2 sinh(2t)3
A(t) = .
sinh(2t)3 cosh(2t)I2

(ii) Let I2 be the 2 2 identity matrix, 02 be the 2 2 zero matrix and 3 be


the Pauli spin matrix. Find the eigenvalues and normalized eigenvectors of the
6 6 matrix
cosh(2t)I2 02 sinh(2t)3
B(t) = 02 I2 02 .
sinh(2t)3 02 cosh(2t)I2
Can the results from (i) be utilized here?

Problem 148. Find the eigenvalues of the 6 6 matrix


1
I2 02 1 I2

2 2
02 I2 02 .
1 I2 02 1 I2
2 2

Problem 149. Find all 2 2 matrices A over C which admit the normalized
eigenvectors    
1 1 1 1
v1 = , v2 =
2 1 2 1
with the corresponding eigenvalues 1 and 2 .

Problem 150. Consider the quadratic form

7x21 + 6x22 + 5x23 4x1 x2 4x2 x3 + 14x1 8x2 + 10x3 + 6 = 0.


98 Problems and Solutions

Write this equation in matrix form and find the eigenvalues and normalized
eigenvectors of the 3 3 matrix.

Problem 151. Let M be an n n invertible matrix with eigenvalues 1 , . . .,


n (counting degeneracy). Find the eigenvalues of
1 1
M+ = (M + M 1 ) and M = (M M 1 ).
2 2

Problem 152. (i) Let x1 , x2 , x3 R. What is the condition such that the
3 3 matrix
0 x1 0
A(x1 , x2 , x3 ) = 0 0 x2 .
x3 0 0
is normal?
(ii) Find the eigenvalues and normalized eigenvectors of A.

Problem 153. Let a, b, c, d R. Find the eigenvalues and eigenvectors of the


matrices
0 a 0 0

0 a 0
a 0 b 0
M1 = a 0 b, M2 =
0 b 0 c

0 b 0
0 0 c 0
0 a 0 0 0

a 0 b 0 0
M3 = 0 b 0 c 0.

0 0 c 0 d

0 0 0 d 0

Problem 154. (i) Study the eigenvalue problem for the symmetric matrices
over R
2 1 0 1

2 1 1
1 2 1 0
A3 = 1 2 1 , A4 = .
0 1 2 1
1 1 2
1 0 1 2
Extend the n dimensions
2 1 0 ... 0 1

1 2 1 ... 0 0
0 1 2 ... 0 0

A=
... .. .. .. .. .. .
. . . . .

0 0 0 ... 2 1
1 0 0 ... 1 2
Eigenvalues and Eigenvectors 99

Problem 155. Study the eigenvalue problem for 3 3, 4 4, 5 5 matrices


where the entries on the diagonal are 0 and the entries on the counter-diagonal
are 0, i.e.

0 b12 b13 0

0 a12 0
b21 0 0 b24
A = a21 0 a23 , B=
b31 0 0 b34

0 a32 0
0 b42 b43 0

0 c12 c13 c14 0



c21 0 c23 0 c25
C = c31 c32 0 c34 c35 .

c41 0 c43 0 c45

0 c52 c53 c54 0
Extend to arbitrary n. Consider the case n odd and n even separately.

Problem 156. Let In be the n n identity matrix and 0n be the n n zero


matrix. Find the eigenvalues and eigenvectors of the 2n 2n matrices
   
0n In 0 n In
A= , B= .
In 0n In 0 n

Problem 157. Let A be an normal 2 2 matrix with eigenvalues 1 and 2 .


What can be said about the eigenvalues of the 3 3 matrix

a11 0 a12
B = 0 0 0 .
a21 0 a22

Problem 158. Let  [0, 1]. Consider the matrix


 
1 1 
U () = .
1 + 2  1
Find the eigenvalues and eigenvectors. The matrix interpolates between the
Pauli matrix 3 ( = 0) and the Hadamard matrix ( = 1).

Problem 159. Let = 2 cos(/5). Find the eigenvalues and eigenvectors of


the three Cartan matrices

  2 1 0
2
A2 = , A3 = 1 2 ,
2
0 2
100 Problems and Solutions

2 1 0 0

1 2 1 0
A4 = .
0 1 2
0 0 2

Problem 160. Find the eigenvalues and eigenvectors of the 4 4 matrix

0 a12 a13 a14



a12 0 0 0
A= .
a13 0 0 0
a14 0 0 0

Problem 161. Find the eigenvalues and eigenvectors of 4 4 matrix

1 1 1 z

1 1 1 z
A(z) = .
1 1 1 z
z z z 1

Problem 162. Find all 2 2 matrices A over C which admit the normalized
eigenvectors    
1 1 1 1
v1 = , v2 =
2 1 2 1
with the corresponding eigenvalues 1 and 2 and 1 6= 2 .

Problem 163. Consider the Hadamard matrix


 
1 1 1 1
U= = (3 + 1 ).
2 1 1 2

The eigenvalues of the Hadamard matrix are given by +1 and 1 with the
corresponding normalized eigenvectors
p   p 
1 p4 + 22 , 1 p4 2 2 .

8 42 2 8 4+2 2

How can this information be used to find the eigenvalues and eigenvectors of the
Bell matrix
1 0 0 1

1 0 1 1 0
B= .
2 0 1 1 0
1 0 0 1
Eigenvalues and Eigenvectors 101

Problem 164. Let U be an nn unitary matrix. Assume also that U = U ,


i.e. the matrix is also skew-hermitian. Find the eigenvalues of such a matrix.

Problem 165. Let [0, 1]. Find the inverse, eigenvalues and eigenvectors
of the 4 4 matrix
1 0 0 0

0 1 0 0
A() = .
0 0 1 0
i 0 0 1

Problem 166. Let A = (ajk ) be an n n semi-positive definite matrix with


P n
k=1 ajk < 1 for j = 1, . . . , n. Show that 0 < 1 for all eigenvalues of A.

Problem 167. Let A be an n n stochastic matrix. Show that it has an


eigenvalue equal to 1.

Problem 168. Given an n n permutation matrix P .


(i) Assume that n is even and tr(P ) = 0. Can we conclude that half of the
eigenvalues of such a matrix are +1 and the other half are 1?
(ii) Assume that n is odd and tr(P ) = 0. Can we conclude that the eigenvalues
are given by the n solutions of n = 1?

Problem 169. Let A be an nn matrix over C. The characteristic polynomial


of A is defined by
n
X
() := det(In A) = n + (1)k (k)nk .
k=1

The eigenvalues 1 , . . . , n of A are the solutions of the characteristic equation


() = 0. The coefficients (j) (j = 1, . . . , n) are given by
n
X
(1) = j = tr(A)
j=1
X
(2) = j k
j<k
..
.
n
Y
(n) = j = det(A).
j=1

Another set of symmetric polynomials is given by the traces of powers of the


102 Problems and Solutions

matrix A, namely
n
X
s(j) = (k )j = tr(Aj ) j = 1, . . . , n.
k=1

One has (so-called Newton relation)

j(j) s(1)(j 1) + + (1)j1 s(j 1)(1) + (1)j s(j) = 0, j = 1, . . . , n.

Consider the 3 3 matrix



1/ 2 0 1/ 2
A = 0 1 0 .
1/ 2 0 1/ 2

Calculate s(1), s(2), s(3) from the traces of the powers of A. Then apply the
Newton relation to find (1), (2), (3).

Problem 170. (i) Find the eigenvalues and eigenvectors of the 4 4 matrix

a11 0 0 a14

0 a22 0 0
A= .
0 0 a33 0
a41 0 0 a44

(ii) Find the eigenvalues and eigenvectors of the 6 6 matrix

b11 0 0 0 0 b16

0 b22 0 0 0 0
B= 0 0 b33 0 0 0 .

0 0 0 0 b55 0

b61 0 0 0 0 b66

Problem 171. Consider the matrix


1 0 0 0 1

0 1 1 1 0
A = 0 1 1 1 0.

0 1 1 1 0

1 0 0 0 1

(i) Find the rank of the matrix. Explain.


(ii) Find the determinant and trace of the matrix.
(iii) Find all eigenvalues of the matrix.
(iv) Find one eigenvector.
Eigenvalues and Eigenvectors 103

(v) Is the matrix positive semidefinite?

Problem 172. Let R. Find the eigenvalues and eigenvectors of the


symmetric 3 3 and 4 4 matrices, respectively
1 0 0

1 0
1 1 0
A3 () = 1 1 , A4 () = .

0 1 1
0 1
0 0 1
Extend to n dimensions.

Problem 173. Let R. Find the eigenvalues and eigenvectors of the


symmetric 4 4 matrix, respectively
1 0 1

1 1 0
A4 () = .
0 1 1
1 0 1
Extend to n dimensions.

Problem 174. Let n 2 and even. Consider an n n hermitian matrix A.


Thus the eigenvalues are real. Assume we have the information that if is an
eigenvalue then is also an eigenvalue of A. How can the calculation of the
eigenvalues be simplified with this information?

Problem 175. Study the properties of the matrices


 
1/2 3/2
A =
1/2 1/2

i.e. find the determinant, trace, inverse, A3 and the eigenvalues.

Problem 176. Find the eigenvalues and normalized eigenvectors of the 4 4


matrix
1 0 1 0

1 0 1 0 1
A= .
2 1 0 1 0
0 1 0 1
Are the four column vectors
1 0

1 0
1 0 1 1 1 1
, , 0 , 1cr0
2 1 2 0 2 1cr 0 2 1
0 1
104 Problems and Solutions

in the matrix A entangled?

Problem 177. (i) Find the eigenvalues and eigenvectors of the stochastic
matrix
1/2 1/2 0
S = 0 1/2 1/2 .
1 0 0
(ii) Find the eigenvalues and eigenvectors of the stochastic matrix
pAA pAB pAC 0

0 0 0 1
S=
0 0 0 1

pDA pDB pDC 0

where pAA = pDA = 2 3, pAB = pDB = 3 2, pAC = pDC = 2 1.

Problem 178. Find the eigenvalues of the 4 4 matrix


2 2 0 0

1 4 1 0
A= .
0 2 2 2
0 0 1 4

Problem 179. Let n 2. Consider the tridiagonal matrix


a1 b1 0

c1 a2 b2
.. ..
An = 0 c2
. . .


. .. . .. b


n1
cn1 an
We set D0 = 1 and D1 = a1 . For n 2 we set Dn = det(An ), i.e. the
determinant of An . Then the determinant Dn (n 2) satisfies the recurrence
relation
Dn = an Dn1 cn1 bn1 Dn2 , n = 2, 3, . . .
If we set a1 = a2 = = an = ( will be the eigenvalue) then we obtain the
characteristic polynomial for the matrix
0 b1 0

c1 0 b2
.. ..
M n = 0 c2
. . .

.. ..
. . bn1
cn1 0
Eigenvalues and Eigenvectors 105

Apply it to the matrix



0 1 0
1 0 1
.. ..
H = 0

1 . .

.. ..
. . 1
1 0

and then solve the characteristic equation to find the eigenvalues.

Problem 180. Let 1 , 2 , 1 , 2 C. Let u1 , u2 be an orthonormal basis in


C2 . We define the matrices

A = 1 u1 u1 + 2 u2 u2 , B = 1 u1 u1 + 2 u2 u2 .

Find the commutator [A, B]. Find the conditions on 1 , 2 , 1 , 2 such that
[A, B] = 02 .

Problem 181. Find the eigenvalues and normalized eigenvectors of the 2 2


matrix  
cos ei sin
.
ei sin cos

Problem 182. Consider the 4 4 orthogonal matrices

cos sin 0 0 1 0 0 0

sin cos 0 0 0 cos sin 0
A12 () = , A23 () =
0 0 1 0 0 sin cos 0

0 0 0 1 0 0 0 1

1 0 0 0

0 1 0 0
A34 () = .
0 0 cos sin
0 0 sin cos
Find the eigenvalues of V () = V12 ()V23 ()V34 ().

Problem 183. Prove or disprove: For any permutation matrix their eigenval-
ues form a group under multiplication. Hint. Each permutation matrix admits
the eigenvalue +1. Why?

Problem 184. Consider an n n permutation matrix P . Obviously +1 is


always an eigenvalue since the column vector with all n entries equal to +1
is an eigenvector. Now want to apply a brute force method and give a C++
106 Problems and Solutions

implementation to figure out whether 1 is an eigenvalue. We run over all


column vectors v of length n, where the entries can only be +1 or 1, where of
course the cases with all entries +1 or all entries 1 can be omitted. Thus the
number of column vectors we have to run through are 2n 2. The condition
then to be checked is
P v = v.
If true we have an eigenvalues 1 with the corresponding eigenvector v.

Problem 185. Find all 2 2 matrices over C which admit the normalized
eigenvectors    
1 1 1 1
v1 = , v2 =
2 1 2 1
with the corresponding eigenvalues 1 and 2 .

Problem 186. Consider the 3 3 matrices



1 0 0 1 1 0
A = 1 1 0, B = 0 1 1.
0 1 1 0 0 1

Find the eigenvalues and eigenvectors of A and B. Are the matrices similar?

Problem 187. Consider the 3 3 matrices



0 1 1 0 0 1
A = 1 0 0, B = 0 0 1.
1 0 0 1 1 0

Find the eigenvalues of A and B and the eigenvalues of the commutator [A, B].

Problem 188. (i) What can be said about the eigenvalues and eigenvectors
of a nonzero hermitian n n matrix A with det(A) = 0 and tr(A) = 0.
(ii) Give one eigenvalue of A. Give one eigenvalue of A A.
(iii) Consider the case n = 3 for the matrix A. Find all the eigenvalues and
eigenvectors.

Problem 189. Let n 2 and j, k = 0, 1, . . . , n 1. Consider



1 for j < k
M (j, k) = 0 for j = k
1 for j > k

(i) Write down the matrix M for n = 2 and find the eigenvalues and normalized
eigenvectors.
Eigenvalues and Eigenvectors 107

(ii) Write down the matrix M for n = 3 and find the eigenvalues and normalized
eigenvectors.

Problem 190. Find the eigenvalues and eigenvectors of the 3 3 matrix


A = (ajk ) (j, k = 1, 2, 3)
p
ajk = j(k 1).

Problem 191. Let R. Find the eigenvalues and eigenvectors of the 3 3


matrices
0 0 ei 0 1 0
A() = 1 0 0 , B() = 0 0 1.
0 1 0 ei 0 0

Problem 192. Let n 2 and n even. Find the eigenvalues of the n n


matrices
2 1 0 ... 0 1

..
1
2 1 .
0
.. .. .. .. ..
0 . . . . .
A = .. .

.. .. .. ..
. . . . . 0
.. .. ..

0 . . . 1

..
1 0 . 0 1 2

Problem 193. Let A be an nn diagonalizable matrix over C with eigenvalues


1 , . . . , n and diagonalization A = T DT 1 , where D is a diagonal matrix such
that Djj = j for all j = 1, . . . , n. Show that

a11 1

a22
. = (T (T 1 )T ) .2

.. ..
ann n

where is the Hadamard product.

Problem 194. (i) Find the eigenvalues and eigenvectors of the 4 4 matrix

a11 a12 0 0

0 0 a23 a24
A= .
0 0 a33 a34
a41 a42 0 0
108 Problems and Solutions

(ii) Find the eigenvalues and eigenvectors of the permutation matrix

0 1 0 0

0 0 0 1
A= .
0 0 1 0
1 0 0 0

Problem 195. Let j, k = 0, 1, 2, 3. Write down the 4 4 matrix A with the


entries
 
j k
Ajk = cos
2

and find the eigenvalues.

Problem 196. Consider the 4 4 matrices over R

a11 a12 0 0

a12 a11 a12 0
A1 =
0 a12 a11 a12

0 0 a12 a11

and
a11 a12 0 a14

a a11 a12 0
A2 = 12 .
0 a12 a11 a12
a14 0 a12 a11

Find the eigenvalues of A1 and A2 .

Problem 197. Find the eigenvalues and eigenvectors of the 3 3 matrix



0 a12 0
A = a21 0 a23 .
0 a32 a33

Problem 198. (i) Find the eigenvalues and eigenvectors of the 4 4 matrix

a11 0 0 a14

0 a22 0 0
A= .
0 0 a33 0
a41 0 0 a44
Eigenvalues and Eigenvectors 109

(ii) Find the eigenvalues and eigenvectors of the 6 6 matrix


b11 0 0 0 0 b16

0 b22 0 0 0 0
0 0 b33 0 0 0

B= .
0 0 0 b44 0 0
0 0 0 0 b55 0

b61 0 0 0 0 b66

Problem 199. Find the eigenvalues of the matrices



  1 2/3 1/3
1 1/2
, 2/3 2/3 1/3
1/2 1/2
1/3 1/3 1/3
1 3/4 2/4 1/4

3/4 3/4 2/4 1/4
.

2/4 2/4 2/4 1/4

1/4 1/4 1/4 1/4
Extend to n-dimensions.

Problem 200. Let A be an n n matrix over C. Show that the matrix A


admits the inverse matrix A1 iff Av = 0 implies v = 0.

Problem 201. The permanent of an n n matrix A is defined as


n
X Y
Per(A) = aj,(j)
Sn j=1

where Sn is the set of all permutation of n elements. Find the permanent of a


2 2 matrix  
a11 a12
A= .
a21 a22

Problem 202. Consider the 3 3 symmetric matrix over R



2 1 0
A = 1 2 1 .
0 1 2
Find the largest eigenvalue and the corresponding eigenvector using the power
method. Start from the vector

1
v = 1.
1
110 Problems and Solutions

Problem 203. Let I3 be the 3 3 unit matrix and 03 be the 3 3 zero matrix.
Find the eigenvalues of the 6 6 matrix
 
03 I3
.
I3 03

Problem 204. Consider the tridiagonal n n matrix


a1 b2 0 ... 0 0

c2 a2 b3 ... 0 0
0 c3 a3 ... 0 0


A= .. .. .. .. .. ..
. . . . . .


0 0 0 ... an1 bn
0 0 0 ... cn an
with a1 , aj , bj , cj C (j = 1, 2, . . . , n). It has in general n complex eigenvalues
the n roots of the characteristic polynomial p(). Show that this polynomial
can be evaluated by the recursive formula
pk () = ( ak )pk1 bk ck pk2 (), k = 2, 3, . . . , n
p1 () = a1
p0 () = 1.

Problem 205. Let R and 2 < 1. Consider the 2 2 matrix


K = 1 i3 .
p
Show that the eigenvalues of K and K are given by 1 2 . Find the
normalized eigenvecors.

Problem 206. (i) Find the eigenvalues of the double 2 2 stochastic matrix
 2 
sin () cos2 ()
.
cos2 () sin2 ()
(ii) Find the eigenvalues of the double 3 3 stochastic matrix
2
sin () 0 cos2 ()
cos2 () sin2 () 0 .
0 cos2 () sin2 ()
(iii) Find the eigenvalues of the double stochastic 4 4 matrix
2
sin () 0 0 cos2 ()

2 2
cos () sin () 0 0
.
0 cos2 () sin2 () 0

2 2
0 0 cos () sin ()
Eigenvalues and Eigenvectors 111

Problem 207. Find the eigenvalues and eigenvectors of the 4 4 matrix

a11 a12 a13 a14



0 a22 0 0
A= .
0 0 a33 0
a41 0 0 a44

Problem 208. (i) Find the eigenvalues of the 2 2 matrices


   
a11 a12 a11 a12
, .
0 a22 a21 0

(ii) Find the eigenvalues of the 3 3 matrices



a11 a12 a13 a11 a12 a13
0 a22 a23 , a21 a22 0 .
0 0 a33 a31 0 0

(iii) Find the eigenvalues of the 4 4 matrices

a11 a12 a13 a14 a11 a12 a13 a14



0 a22 a23 a24 a21 a22 a23 0
, .
0 0 a33 a34 a31 a32 0 0

0 0 0 a44 a41 0 0 0

Problem 209. Consider the permutation matrix

0 1 0 0

0 0 0 1
P = .
1 0 0 0
0 0 1 0

(i) Find the eigenvalues and normalized eigenvectors.


(ii) Find the smallest positive integer k such that P k = I4 .

Problem 210. Let A, B be two nonzero nn matrices over C. Let Av = v be


the eigenvalue equation for A. Assume that [A, B] = 0n . Then from [A, B]v = 0
it follows that

[A, B]v = (AB BA)v = A(Bv) B(Av) = 0.

Therefore A(Bv) = (Bv. If Bv 6= 0 we find that Bv is an eigenvector of A


with eigenvalue . Apply it to A = 1 2 and B = 3 3 .
112 Problems and Solutions

Problem 211. Let A, B be two nonzero n n matrices over C. Let Av =


v be the eigenvalue equation for A. Assume that [A, B]+ = 0n . Then from
[A, B]+ v = 0 it follows that

[A, B]+ v = (AB + BA)v = A(Bv) + B(Av) = 0.

Therefore A(Bv) = (Bv. If Bv 6= 0 we have an eigenvalue equation with


eigenvalue . Apply it to A = 1 and B = 2 , where [1 , 2 ]+ = 02 .

Problem 212. Find the eigenvalues of the 7 7 matrix

0 1 1 0 0 0 0

1 0 0 1 0 0 0
1 0 0 1 0 0 0

A = 0 1 1 0 1 1 0.

0 0 0 1 0 0 1

0 0 0 1 0 0 1

0 0 0 0 1 1 0

Problem 213. Consider the spin-1 matrices



0 1 0 0 i 0 1 0 0
1 1
S1 = 1 0 1 , S2 = i 0 i , S3 = 0 0 0
2 0 1 0 2 0 i 0 0 0 1

and

S,1 = S I3 I3 , S,2 = I3 S I3 , S,3 = I3 I3 S

with = 1, 2, 3. Let

S1 I3 I3 I3 S1 I3 I3 I3 S1
S1 = S2 I3 I3 , S2 = I3 S2 I3 , S3 = I3 I3 S2
S3 I3 I3 I3 S3 I3 I3 I3 S3

and
I3 S2 S3 I3 S3 S2
S2 S3 = I3 S3 S1 I3 S1 S3
I3 S1 S2 I3 S2 S1
Thus

S1 (S2 S3 ) = S1 S2 S3 S1 S3 S2 +S2 S3 S1 S2 S1 S3 +S3 S1 S2 S3 S2 S1 .

Find the eigenvalues of S1 (S2 S3 ).


Eigenvalues and Eigenvectors 113

Problem 214. Find the eigenvalues and normalized eigenvectors of the 4 4


matrix  
02 I2
.
I2 02

Problem 215. Let n 2. Consider the (n 1) n matrix A over R. Then


AAT is an (n 1) (n 1) matrix over R and AT A is an n n matrix over R.
Show that the (n 1) eigenvalues of AAT are also eigenvalues of AT A and AT A
additionally admits the eigenvalue 0.

Problem 216. Study the eigenvalue problem for the symmetric matrices over
R
1 1/2 1/3 1/4

  1 1/2 1/3
1 1/2 1/2 1/2 0 0
, 1/2 1/2 0 , .

1/2 1/2 1/3 0 1/3 0
1/3 0 1/3
1/4 0 0 1/4
Extend to infinity.

Problem 217. Let H be an hermitian n n matrix and U an n n unitary


matrix such that U HU = H. We call H invariant under U . From U HU = H
it follows that [H, U ] = 0n . If v is an eigenvector of H, i.e. Hv = v, then U v
is also an eigenvector of H since

H(U v) = (HU )v = U (Hv) = (U v).

The set of all unitary matrices Uj (j = 1, . . . , m) that leave a given hermitian


matrix invariant, i.e. Uj HUj = H (j = 1, . . . , m) form a group under matrix
multiplication.
(i) Find all 2 2 hermitian matrices H such that [H, U ] = 02 , where U = 1 .
(ii) Find all 2 2 hermitian matrices H such that [H, U ] = 02 , where U = 2 .
(iii) Find all 44 hermitian matrices H such that [H, U ] = 04 , where U = 1 1 .
(ii) Find all 44 hermitian matrices H such that [H, U ] = 04 , where U = 2 2 .

Problem 218. Find the eigenvalues and normalized eigenvectors of the 2 2


matrix  
sin() cos()
M () = .
cos() sin()

Problem 219. Consider the skew-symmetric matrix over R



0 a3 a2
A = a3 0 a1
a2 a1 0
114 Problems and Solutions

where a1 , a2 , a3 R. Find the eigenvalues. Let 03 be the 3 3 zero matrix. Let


A1 , A2 , A3 be skew-symmetric 3 3 matrices over R. Find the eigenvalues of
the 9 9 matrix
03 A3 A2
B = A3 03 A1 .
A2 A1 03

Problem 220. Let


   
1 1 0 1
A= , B= .
1 1 1 0
(i) Find A B, B A.
(ii) Find
tr(A B), tr(B A).
Find
det(A B), det(B A).
(iii) Find the eigenvalues of A and B.
(iv) Find the eigenvalues of A B and B A.
(v) Find rank(A), rank(B) and rank(A B).

Problem 221. Let 1 , 2 and 3 be the eigenvalues of the matrix



1 0 1
A = 0 1 0
1 0 1

Find 21 + 22 + 23 without calculating the eigenvalues of A or A2 .

Problem 222. Consider the semi-simple Lie group SL(2, R). Let A SL(2, R).
Then A1 SL(2, R). Explain why. Show that A and A1 have the same eigen-
values. Is this still true for A SL(3, R)?

Problem 223. Let An be the n n matrices of the form


0 0 0 t

  0 0 t
0 t 0 0 t 0
A1 = 1, A2 = , A3 = 0 1 0, A4 =
t r 0 t r 0

t 0 r
t 0 0 r
0 0 0 0 t

0 0 0 t 0
A5 = 0 0 1 0 0.

0 t 0 r 0

t 0 0 0 r
Eigenvalues and Eigenvectors 115

Thus the even dimensional matrix A2n has t along the skew-diagonal and r along
the lower main diagonal. Otherwise the entries are 0. The odd dimensional
matrix A2n+1 has t along the skew-diagonal except 1 at the centre and r along
the lower main diagonal. Otherwise the entries are 0. Find the eigenvalues of
these matrices.

Problem 224. Let 1 , 2 R. Consider the 2 2 matrix

1 cos2 () + 22 sin2 () (22 21 ) cos() sin()


 2 
A(21 , 22 ) = .
(22 21 ) sin() cos() 21 sin2 () + 22 cos2 ()

(i) Find the trace and determinant of A(21 , 22 ).


(ii) Find the eigenvalues of A(21 , 22 ).
Remark. Any positive semidefinite 2 2 matrix over R can be written in this
form. Try the matrix  
1 1
M= .
1 1

Problem 225. Consider the 4 4 matrix


a1 a2 a3 a4

b1 0 0 0
M = .
0 b2 0 0
0 0 b3 0

Find the eigenvalues.

Problem 226. Let A be an n n hermitian matrix with n odd. Thus the


eigenvalues are real. Assume that exactly one eigenvalue is 0 and tr(A) = 0.
What are the conditions on A such that the remaining (real) eigenvalues are
symmetric around 0, i.e. for each eigenvalue 6= 0 one has the eigenvalue .
Consider first the 3 3 case. Can the spectral theorem be applied?

Problem 227. Let j, k {0, 1}. Find the eigenvalues of the 2 2 matrix
M = (Mjk )
Mjk = ei(jk) .

Problem 228. Consider a 2 2 matrix A = (ajk ) over R with


2
X
ajk = 1, tr(A) = 0.
j,k=1

What can be said about the eigenvalues of such a matrix?


116 Problems and Solutions

Problem 229. Let [0, /2). Find the trace, determinant, eigenvalues and
eigenvectors of the 2 2 matrices

1 tan2 () tan() 1 tan2 () tan() 1 tan2 () 0


     
M1 = , M2 = , M3 = .
2 tan() 1 2 tan() 1 0 0

Problem 230. Let , R. Find the eigenvalues and eigenvectors of


   
cosh() sinh() cos() sin()

sinh() cosh() sin() cos()

Problem 231. Compare the eigenvalues of the 6 6 matrices

0 1 0 0 0 0

1 0 1 0 0 0
0 1 0 1 0 0

0 0 1 0 1 0

0 0 0 1 0 1

0 0 0 0 1 0

0 1 0 0 0 1

1 0 1 0 0 0
0 1 0 1 0 0

.
0 0 1 0 1 0

0 0 0 1 0 1

1 0 0 0 1 0

Problem 232. Find the eigenvalues and normalized eigenvectors of the her-
mitian 3 3 matrix
 1 0 v1
H = 0  2 v2
v1 v2 3
with 1 , 2 , 3 R and v1 , v2 C.

Problem 233. Let j R with j = 1, 2, 3, 4. Consider the hermitian matrix

1 1 0 0

1 2 1 0
H(j ) = .
0 1 3 1
0 0 1 4

(i) Find the eigenvalues and eigenvectors.


Eigenvalues and Eigenvectors 117

(ii) Let
v1

v
v = 2
v3
v4
be an eigenvectors. Find v1 /v2 , v2 /v3 , v3 /v4 .

Problem 234. The Cartan matrix A of a rank r root system of a semi-simple


Lie algebra is an r r matrix whose entries are dervided from the simple roots.
The entries of the Cartan matrix are given by

(j , k )
ajk = 2
j , j )

where ( , ) is the Euclidean inner product and j are the simple roots. The entries
are independent of the choise of simple roots (up to odering). The Cartan matrix
of the Lie algebra e8 is given by the 8 8 matrix

2 1 0 0 0 0 0 0

1 2 1 0 0 0 0 0
0 1 2 1 0 0 0 1

0 0 1 2 1 0 0 0

.
0 0 0 1 2 1 0 0

0 0 0 0 1 2 1 0

0 0 0 0 0 1 2 0

0 0 1 0 0 0 0 2

(i) Find the trace and the determinant of the matrix.


(ii) Find the eigenvalues and eigenvectors of the matrix.

Problem 235. Is the 3 3 stochastic matrix



1/3 1/3 1/3
2/3 0 1/3
1/2 1/2 0

diagonalizable? First find the eigenvalues and normalized eigenvectors.

Problem 236. Let R. Find the eigenvalues + (), () and normalized


eigenvectors of the matrix
 
sinh() 1
A(s) = .
1 sinh()

Find the shortest distance between + () and ().


118 Problems and Solutions

Problem 237. Consider the symmetric 3 3 matrix



n1 1 0
A = 1 n2 1 .
0 1 n3
Find all postive integers n1 , n2 , n3 such that det(A) = 1 and A is positive
definite.

Problem 238. Find the eigenvalues of the staircase matrices


0 0 0 1

  0 0 1
0 1 0 0 1 1
, 0 1 1, .
1 1 0 1 1 1

1 1 1
1 1 1 1

Problem 239. Consider the real symmetric 3 3 matrices



0 0 1 0 1 1
A = 0 1 0, B = 1 0 1.
1 0 0 1 1 0
(i) Find the eigenvalues and normalized eigenvectors for the matrices A and B.
(ii) Find the commutator [A, B].
(iii) Discuss the results from (ii) and (i) with respect to the eigenvectors.

Problem 240. Let A be an n n matrix. Let J be the n n matrix with 1s


in the counter diagonal and 0s otherwise. Let
tr(A) = 0, tr(JA) = 0.
What can be said about the eigenvalues of A? Consider first the cases n = 2
and n = 3.

Problem 241. What is the condition on R such that the hermitian matrix
 
1 cos()
A() =
cos() 1
has positive eigenvalues?

Problem 242. Consider the 6 5 matrix


0 1 0 1 0

1 0 1 0 1
1 0 1 0 1

A= .
0 1 0 1 0
0 1 0 1 0

0 0 1 0 0
Eigenvalues and Eigenvectors 119

Find the eigenvalues of the matrices AT A and AAT .

Problem 243. Let A be an n n diagonalizable matrix with distinct eigen-


values j (j = 1, . . . , n). Show that the Vandermonde matrix
1 1 1
1 2 n
2
22 2n

V =
.1

. .. .. ..
. . . .

n1
1 n1
2 n1
n

is nonsingular.

Problem 244. Let say we want to calculate the eigenvalues of

1 0 1 0

0 1 0 1
A= .
1 0 1 0
0 1 0 1

How could we utilize the facts that


0 1  0 1

    
1 0 0 1 0 1 0 1 0 1 1 0 2 0
= A, =
0 1 1 0 1 0 1 0 1 0 0 1 0 2

1 0 1 0

and tr(A) = 4 and thus avoid solving det(A I4 ) = 0?


Chapter 5

Commutators and
Anticommutators

Problem 1. Let A, B be 2 2 symmetric matrices over R. Assume that


AAT = I2 and BB T = I2 . Is
[A, B] = 02 ?
Prove or disprove.

Problem 2. Let A, B, X, Y be n n matrices over C. Assume that

AX XB = Y.

(i) Let z C. Show that

(A zIn )X X(B zIn ) = Y.

(ii) Assume that A zIn and B zIn are invertible. Show that

X(B zIn )1 (A zIn )1 X = (A zIn )1 Y (B zIn )1 .

Problem 3. Let A, B be n n matrices over C. Assume that [A, B] = 0n .


Let U be a unitary matrix. Calculate [U AU, U BU ].

Problem 4. Can we find nonzero symmetric 2 2 matrices H and A over R


such that
[H, A] = A

120
Commutators and Anticommutators 121

where R and 6= 0?

Problem 5. Let A, B be n n hermitian matrices. Is i[A, B] hermitian?

Problem 6. A truncated Bose annihilation operator is defined as the n n


(n 2) matrix

0 1 0 0 0 ... 0

0 0 2 0 0 ... 0
0 0 0 3 0 ... 0


Bn =
.. .. .. .. .. .. .. .
. . . . . . .


0 0 0 0 ... 0 n 1
0 0 0 0 ... 0 0
(i) Calculate Bn Bn .
(ii) Calculate the commutator [Bn , Bn ].

Problem 7. Find nonzero 2 2 matrices A, B such that [A, B] 6= 02 , but

[A, [A, B]] = 02 , [B, [A, B]] = 02 .

Problem 8. Let A, B be symmetric n n matrices over R. Show that [A, B]


is skew-symmetric over R.

Problem 9. Let A, B be n n matrices. Show that


1
AB ([A, B] + [A, B]+ ).
2

Problem 10. Let A, B be n n matrices. Suppose that

[A, B] = 0n , [A, B]+ = 0n

and that A is invertible. Show that B must be the zero matrix.

Problem 11. Let  


0 1
C= .
1 0
Find all 22 matrices A such that [A, C] = 02 , where 02 is the 22 zero matrix.

Problem 12. Let A, B, C be n n matrices. Show that

tr([A, B]C) tr(A[B, C]).


122 Problems and Solutions

Problem 13. Find all nonzero 2 2 matrices A, B such that

[A, B] = A + B.

Problem 14. Find all nonzero 2 2 matrices J+ , J , Jz such that

[Jz , J+ ] = J+ , [Jz , J ] = J , [J+ , J ] = 2Jz

where (J+ ) = J .

Problem 15. Find all nonzero 2 2 matrices K+ , K , Kz such that

[Kz , K+ ] = K+ , [Kz , K ] = K , [K+ , K ] = 2Kz

where (K+ ) = K .

Problem 16. Find all nonzero 2 2 matrices A1 , A2 , A3 such that

[A1 , A2 ] = 0, [A1 , A3 ] = A1 , [A2 , A3 ] = A2 .

Problem 17. Let H be a nonzero n n hermitian matrix. Let E be a nonzero


n n matrix. Assume that
[H, E] = aE
where a R and a 6= 0. Show that E cannot be hermitian.

Problem 18. Let A and B be positive semi-definite matrices. Can we conclude


that [A, B]+ AB + BA is positive semi-definite.

Problem 19. Let A, B be n n matrices. Given the expression

A2 B + AB 2 + B 2 A + BA2 2ABA 2BAB.

Write the expression in a more compact form using commutators.

Problem 20. Let A, B be n n matrices over C. Assume that A2 = In and


B 2 = In .
(i) Find the commutators [AB + BA, A], [AB + BA, B].
(ii) Give an example of such matrices for n = 2 and A 6= B.

Problem 21. Consider the 2 2 matrices


   
cos sin cosh sinh
A() = , B() =
sin cos sinh cosh
Commutators and Anticommutators 123

where , R. Calculate the commutator [A(), B()]. What is the condition


on , such that [A(), B()] = 02 ?

Problem 22. Let A1 , A2 , A3 be n n matrices over C. The ternary commu-


tator [A1 , A2 , A3 ] (also called the ternutator) is defined as
X
[A1 , A2 , A3 ] := sgnA(1) A(2) A(3)
S3
A1 A2 A3 + A2 A3 A1 + A3 A1 A2 A1 A3 A2 A2 A1 A3 A3 A2 A1 .
(i) Let n = 2 and consider the Pauli spin matrices 1 , 2 , 3 . Calculate the
ternutator
[1 , 2 , 3 ].
(ii) Calculate
A1 A2 A3 +A2 A3 A1 +A3 A1 A2 A1 A3 A2 A2 A1 A3 A3 A2 A1 .

Problem 23. Let A, B, C be 2 2 matrices. Find the conditions such that


[A, B, C] = 0.

Problem 24. Let A, B, H be n n matrices such that


[H, A] = 0, [H, B] = 0.
Show that
[H In + In H, A B] = 0
where denotes the direct sum.

Problem 25. Show that any two 2 2 matrices which commute with the
matrix  
0 1
1 0
commute with each other.

Problem 26. Let A1 , A2 be m m matrices over C. Let B1 , B2 be n n


matrices over C. Show that
[A1 B1 , A2 B2 ] = ([A1 , A2 ]) ([B1 , B2 ])
where denotes the direct sum.

Problem 27. Let A, B be an n n matrix over C. Let u Cn considered as


column vector. Is
[u A, u B] = u [A, B] ?
124 Problems and Solutions

Here [ , ] denotes the commutator and



/x1
= ... .

/xn

Problem 28. Let c R and A be an 22 matrix over R. Find the commutator


of the 3 3 matrices
c A, Ac
where denotes the direct sum.

Problem 29. Consider the 3 3 matrices



a1 0 0 0 0 b1
A = 0 a2 0 , B = 0 b2 0 .
0 0 a3 b3 0 0

Can we find aj , bj (j = 1, 2, 3) such that the commutator [A, B] is invertible?

Problem 30. Let A, B be n n matrices over C. Can we conclude that

k[A, B]k kAk kBk ?

Problem 31. Consider (m + n) (m + n) matrices of the form


 
mm mn
.
nm nn
Let    
B1 0 B
e1 0
B= , B
e=
0 B2 0 B
e2
and    
0 F1 0 Fe1
F = , Fe = .
F2 0 Fe2 0
Find the commutators [B.B],
e [B, F ] and the anticommutator [F, Fe]+ .

Problem 32. Can one find non-invertible 2 2 matrices A and B such the
commutator [A, B] is invertible?

Problem 33. Let A, B be n n matrices over C. Assume that B is invertible.


Show that
[A, B 1 ] B 1 [A, B]B 1 .
Commutators and Anticommutators 125

Problem 34. Let A, B be n n matrices. Show that


1 1
AB [A, B] + [A, B]+ .
2 2

Problem 35. (i) Let A, B be 2 2 skew-symmetric matrices over R. Find


the commutator [A, B].
(ii) Let A, B be 3 3 skew-symmetric matrices over R. Find the commutator
[A, B].

Problem 36. Find two linearly independent 2 2 matrices A, B such that


A = [B, [B, A]], B = [A, [A, B]].

Problem 37. Let A be an n n matrix and 0n be the n n zero matrix. Find


the commutator    
0n A 0n A
,
A 0n A 0n
and the anticommutator

Problem 38. Find all 2 2 matrices over C such that the commutator is an
invertible diagonal matrix D, i.e. d11 6= 0 and d22 6= 0.

Problem 39. Let A, B be invertible n n matrices over C. Assume that


[A, B] = 0n . Can we conclude that [A1 , B 1 ] = 0n ?

Problem 40. Consider the 3 3 matrices over C



a11 0 0 0 b12 b13
A = 0 a22 0 , B = b21 0 b23 .
0 0 a33 b31 b32 0
(i) Calculate the commutator [A, B] and det([A, B]).
(ii) Set a11 = ei1 , a22 = ei2 , a33 = ei3 . Find the condition on 1 , 2 , 3 , b12 ,
b13 , b21 , b23 , b31 , b32 such that [A, B] is unitary.

Problem 41. Let A, B be n n matrices and T a (fixed) invertible n n


matrix. We define the bracket
[A, B]T := AT B BT A.
Let
       
0 1 0 1 0 0 1 0
T = , X= , Y = , H= .
1 0 0 0 1 0 0 1
126 Problems and Solutions

Find [X, Y ]T , [X, H]T , [Y, H]T .

Problem 42. Can one find a 2 2 matrix A over R such that


 
0 1
[AT , A] = .
1 0

Problem 43. Consider the set of 3 3 matrices



1 0 0 0 0 0 0 0 0
A1 = 0 0 0 , A2 = 0 1 0, A3 = 0 0 0
0 0 0 0 0 0 0 0 1

0 1 0 0 0 0 0 0 1
A12 = 1 0 0 , A23 = 0 0 1, A13 = 0 0 0 .
0 0 0 0 1 0 1 0 0
Calculate the anticommutator and thus show that we have a basis of a Jordan
algebra.

Problem 44. Consider the invertible matrices



1 0 0 0 1 0
A = 0 e2i/3 0 , B = 0 0 1.
0 0 e2i/3 1 0 0

Is the matrix [A, B] invertible?

Problem 45. A classical 3 3 matrix representation of the algebra iso(1, 1)


is given by

0 0 0 0 0 0 0 0 0
K = 0 0 2 , P+ = 1 0 0 , P = 1 0 0 .
0 2 0 1 0 0 1 0 0
Find the commutators and anticommutators.

Problem 46. Find the conditions on the two 2 2 hermitian matrices A, B


such that
[A B, P ] = 04
where P is the permutation matrix
1 0 0 0

0 0 1 0
P = .
0 1 0 0
0 0 0 1
Commutators and Anticommutators 127

Problem 47. Let A be an arbitrary n n matrix over C with tr(A) = 0.


Show that A can be written as commutator, i.e. there are n n matrices X and
Y such that
A = XY Y X.

Problem 48. Let A be an arbitrary nn matrix over C with tr(A) = 0. Show


that A can be written as commutator, i.e., there are n n matrices X and Y
such that A = [X, Y ].

Problem 49. Let A be an arbitrary n n matrix over C with trA = 0. Show


that A can be written as commutator, i.e., there are n n matrices X and Y
such that A = [X, Y ].

Problem 50. Let A, B be hermitian matrices, i.e. A = A and B = B.


Then in general A + iB is non-normal. What are the conditions on A and B
such that A + iB is normal?

Problem 51. Let A, B be n n matrices over C. Show that if A and B


commute and if A is normal, then A and B commute.

Problem 52. Let j (j = 0, 1, 2, 3) be the Pauli spin matrices, where 0 is the


2 2 identity matrix. Form the four 4 4 matrices
 
02 k
k = , k = 0, 1, 2, 3
k 02
where 02 is the 2 2 identity matrix.
(i) Are the matrices k linearly independent?
(ii) Find the eigenvalues and eigenvectors of the k s.
(iii) Are the matrices k invertible. Use the result from (ii). If so, find the
inverse.
(iv) Find the commutators [k , ` ] for k, ` = 0, 1, 2, 3. Find the anticommutators
[k , ` ]+ for k, ` = 0, 1, 2, 3.
(v) Can the matrices k be written as the Kronecker product of two 2 2
matrices?

Problem 53. Let A be an n n matrix over C. It can be written as A = HU ,


where H is a non-negative definite hermitian matrix and U is unitary. Show
that the matrices H and A commutate if and only if A is normal.

Problem 54. Let A, B be n n matrices over C such that A2 = In , B 2 = In ,


where In is the n n unit matrix. Now assume that
[A, B]+ AB + BA = 0n
128 Problems and Solutions

i.e. the anticommutator of A and B vanishes. Show that there is no solution for
A and B if n is odd.

Problem 55. Let T be an invertible matrix. Show that

T 1 AT A + T 1 [A, T ].

Problem 56. (i) Let 1 , 2 , 3 be the Pauli spin matrices. Consider the three
non-normal matrices

A = 1 + i2 , B = 2 + i3 , C = 3 + i1 .

Find the commutators and anti-commutators. Discuss.


(ii) Consider the three non-normal matrices

X = 1 1 + i2 2 , Y = 2 2 + i3 3 , Z = 3 3 + i1 1 .

Find the commutators and anti-commutators. Discuss.

Problem 57. (i) Let 1 , 2 , 3 be the Pauli spin matrices. Consider the
nonnormal matrices
     
1 1 1 3 1 1 3 i
A= , B= , C=
0 1 2 1 1 2 i 1

where det(A) = det(B) = det(C) = 1, i.e. A, B, C are elements of the Lie group
SL(2, C). Show that

[A, A ] = 3 , [B, B ] = 1 , [C, C ] = 2 .

(ii) Consider the unitary matrices


   
1 1 1 1 0
U= , V = .
2 1 1 0 i

Show that
B = U AU , C = V BV .
(iii) Consider the nonnormal and noninvertible matrices
     
0 1 1 1 1 1 1 i
X= , Y = , Z= .
0 0 2 1 1 2 i 1

All have trace zero and thus are elements of the Lie algebra s`(2, C). Show that

[X, X ] = 3 , [Y, Y ] = 1 , [Z, Z ] = 2 .


Commutators and Anticommutators 129

Hint. Obviously we have X = A I2 , Y = B I2 , Z = C I2 .


(iv) Show that
Y = U XU , Z = V Y V .
(v) Study the commutators

[X X, X X ], [X X , X X],

[Y Y, Y Y ], [Y Y , Y Y ],
[Z Z, Z Z ], [Z Z , Z Z].

Problem 58. (i) Can one find n n matrices A and B over C such that the
following conditions are satisfied

[A, B] = 0n , [A, B]+ = 0n

and both A and B are invertible?


(ii) Can one find nn matrices A and B over C such that the following conditions
are satisfied
[A, B] = 0n , [A, B]+ = In
and both A and B are invertible?

Problem 59. Consider the hermitian 3 3 matrices



0 0 0 0 0 i 0 i 0
L1 = 0 0 i , L2 = 0 0 0 , L3 = i 0 0.
0 i 0 i 0 0 0 0 0

Find the commutators [L1 , L2 ], [L2 , L3 ], [L3 , L1 ]. Find the 3 3 matrix

A = c23 [L2 , L3 ]+ + c13 [L3 , L1 ]+ + c12 [L1 , L2 ]+

where [, ]+ denotes the anti-commutator.

Problem 60. Let A, B, C be n n matrices. Show that

[A, [B, C]+ ] = [[A, B], C]+ + [B, [A, C]]+ .

Problem 61. Find all 22 matrices A, B, C such that [A, B] 6= 02 , [A, C] 6= 02 ,


[B, C] 6= 02 and
[A, [B, C]] = 02 .

Problem 62. Show that one can find a 3 3 matrix over R such that

A2 AT + AT A2 = 2A, AAT A = 2A, A3 = 03


130 Problems and Solutions

and
1 0 0
1 T
[A , A] = 0 0 0 .
2
0 0 1

Problem 63. Find all nonzero 2 2 matrices A, B such that

[A, B]+ = 02 , tr(AB ) = 0.

Problem 64. Consider the matrices



d1 0 0 0 a 0
D = 0 d2 0 , M = a 0 b.
0 0 d3 0 b 0

Find the conditions on d1 , d2 , d3 , a, b such that



0 a 0
[D, M ] = a 0 b .
0 b 0

Problem 65. (i) Find all 2 2 matrices A and B such that

[A, B] = A B.

(ii) Find all 2 2 matrices A and B such that

[A A, B B] = A A B B.

Problem 66. Given the 4 4 matrices


0 1 0 0 0 0 0 0

0 0 0 0 0 0 1 0
A= , B=
0 0 0 1 0 0 0 0

0 0 0 0 0 0 0

where R. Find A2 , B 2 and the anticommutator [A, B]+ .

Problem 67. (i) Let , C. Show that the 2 2 matrices


   
0 1 2
A= , B=
0 0 1
Commutators and Anticommutators 131

satisfy the conditions

[A, B]+ = I2 , [A, A]+ = 02 , [B, B]+ = 02 .

(ii) Let , C. Show that the 2 2 matrices


   
1 0 0
A= , B =
2 1 0

satisfy the conditions

[A, B]+ = I2 , [A, A]+ = 02 , [B, B]+ = 02 .

Problem 68. (i) Find all 2 2 matrices A over C such that

[A, A ]+ = I2

where [, ]+ denotes the anti-commutator.


(ii) Find all 2 2 matrices B over C such that

[B, B ]+ = I2 , [B, B ] = 3

where [, ] denotes the commutator and 3 is the third Pauli spin matrix.

Problem 69. Find 4 4 matrices C and 2 matrices A such that

[C, A I2 + I2 A] = 04 .

Problem 70. (i) Consider the two 2 2 matrices (counter diagonal matrices)
   
0 a12 0 b12
A= B= .
a21 0 b21 0

Find the condition on A and B such that the commutator [A, B] vanishes, i.e.
[A, B] = 02 .
(ii) Consider the two 3 3 matrices (counter diagonal matrices)

0 0 a13 0 0 b13
A = 0 a22 0 B = 0 b22 0 .
a31 0 0 b31 0 0

Find the condition on A and B such that the commutator [A, B] vanishes, i.e.
[A, B] = 03 .
(iii) Extend to n dimensions.
132 Problems and Solutions

Problem 71. Find all nonzero 2 2 matrices A and B such that

[A, [A, B]] = 02 .

Problem 72. Can we find 3 3 matrices A and B such that [A, B]+ = 03 and
A2 = B 2 = I 3 ?

Problem 73. Find all 2 2 matrices A, B over C such that

[A, B] = A + B.

Problem 74. Find all 2 2 matrices A over C such that


 
1 0
[A, A ]+ = I2 , [A, A ] = 3 = .
0 1

Problem 75. Let A be an arbitrary 2 2 matrix. Calculate the commutator

[A 3 , I2 A].

Problem 76. Let n 1 and A, B be n n matrices over C. Show that


n1
X
[A, B n ] = Aj [A, B]B n1j .
j=0

Problem 77. Consider the Pauli spin matrices 1 , 2 , 3 and the set

{ 2 3 , 3 1 , 1 2 }.

Find the commutators

[2 3 , 3 1 ], [2 3 , 1 2 ], [3 1 , 1 2 ]

and anticommutators

[2 3 , 3 1 ]+ , [2 3 , 1 2 ]+ , [3 1 , 1 2 ]+ .
Chapter 6

Decomposition of Matrices

Problem 1. We consider 3 3 matrices over R. An orthogonal matrix Q such


that det Q = 1 is called a rotation matrix. Let 1 p < r 3 and be a real
number. An orthogonal 3 3 matrix Qpr () = (qij )1i,j3 given by

qpp = qrr = cos


qii = 1 if i 6= p, r
qpr = qrp = sin
qip = qpi = qir = qri = 0 i 6= p, r
qij = 0 if i 6= p, r and j 6= p, r

will be called a plane rotation through in the plane span (ep , er ). Let Q =
(qij )1i,j3 be a rotation matrix. Show that there exist angles [0, ), ,
(, ] called the Euler angles of Q such that

Q = Q12 ()Q23 ()Q12 (). (1)

Problem 2. For any nn matrix A over C, there exists a positive semi-definite


matrix H and a unitary matrix such that A = HU (polar decomposition). If A
is nonsingular, then H is positive definite and U and H are unique. Find the
polar decomposition for

1 0 4
A = 0 5 4 .
4 4 3

133
134 Problems and Solutions

Problem 3. If A Rnn , then there exists an orthogonal Q Rnn such


that
R11 R12 ... R1m

0 R22 ... R2m
QT AQ =

... ..
.
..
.
..
.
0 0 ... Rmn
where each Rii is either a 11 matrix or a 22 matrix having complex conjugate
eigenvalues. Find Q for the matrix

0 1 0
A = 2 0 3.
0 4 0

Then calculate QT AQ.

Problem 4. Let n 2 and n = 2k. Let A be an n k matrix and

A A = I k

where Ik is the k k unit matrix. Find the n n matrix AA using the singular
value decomposition. Calculate tr(AA ).

Problem 5. Let n 2 and n = 2k. Let A be an n k matrix and

A A = I k

where Ik is the k k unit matrix. Let S be a positive definite n n matrix.


Show that
tr(A S 2 A)
1 .
tr((A SA)2 )

Problem 6. Consider the symmetric matrix over R



2 1 0
A = 1 2 1
0 1 2

and the orthogonal matrix



cos sin 0
O = sin cos 0 .
0 0 1

e = O1 AO. Can we find an angle such that e


Calculate A a12 = e
a21 = 0?
Decomposition of Matrices 135

Problem 7. Let A be an n n matrix over R. Assume that A1 exits. Given


the singular value decomposition of A, i.e. A = U W V T . Find the singular value
decomposition for A1 .

Problem 8. Find the cosine-sine decomposition of the 4 4 unitary matrix


1 0 0 1

1 0 1 1 0
.
2 0 1 1 0
1 0 0 1

Problem 9. Find a cosine-sine decomposition of the Hadamard matrix


 
1 1 1
.
2 1 1

Problem 10. (i) Consider the 4 4 matrices


0 1 0 0 0 0 1 0

1 0 1 0 e = 0 0 0 1
= , .

0 1 0 1 1 0 0 0
0 0 1 0 0 1 0 0
Can one find 4 4 permutation matrices P , Q such that
= P Q
e ?

(ii) Consider the 2n 2n matrices


0 1 0 ... 0 0 0

1 0 1 ... 0 0
0 1 0 ... 0 0 0

= ..

.

0 0 0 ... 1 0 1
0 0 0 ... 0 1 0
and  
0n In

e=
In 0n
where 0n is the n n zero matrix and In is the n n identity matrix. Can one
find 2n 2n permutation matrices P , Q such that = P Q?
e

Problem 11. Let A be an m m matrix. Let B be an n n matrix. Let X


be an m n matrix such that
AX = XB. (1)
136 Problems and Solutions

We can find non-singular matrices V and W such that

V 1 AV = JA , W 1 BW = JB

where JA , JB are the Jordan canonical form of A and B, respectively. Show


that from (1) it follows that
JA Y = Y JB
where Y := V 1 XW .

Problem 12. Find the Cosine-Sine decomposition of

1 0 0 1

1 0 1 1 0
.
2 0 1 1 0
1 0 0 1

Problem 13. Given the 3 2 matrix



0 1
A = 1 1.
1 0

Find the singular value decomposition of A.

Problem 14. Let A be an invertible n n matrix. Note that A need not be


normal. Then there is a unique n n unitary matrix U and an n n positive
definite matrix P such that
A = U P.
This is called the polar decomposition The positive definite matrix P is the
unique square root of the positive definite matrix A A and then U is defined by
U = AP 1 . Apply the polar decomposition to the nonnormal matrix
   
1 1 1 0
A= A =
0 1 1 1

which is an element of the Lie group SL(2, R).


Chapter 7

Functions of Matrices

Let A be an n n matrix over C. The function exp(A) can be calculated from



X Aj
exp(A) =
j=0
j!

and  m
A
exp(A) = lim In + .
m m
For sinh(A) and cosh(A) we have

X A2j+1
sinh(A) =
j=0
(2j + 1)!


X A2j
cosh(A) = .
j=0
(2j)!

Let M be an n n matrix over C. If there exists a matrix B such that B 2 = M ,


then B is called a square root of M .

Problem 1. Consider the matrix ( R)

cosh() 0 0 sinh()

0 cosh() sinh() 0
S() = .

0 sinh() cosh() 0
sinh() 0 0 cosh()

137
138 Problems and Solutions

(i) Show that the matrix is invertible, i.e. find the determinant.
(ii) Calculate the inverse of S().
(iii) Calculate
d
A := S()
d =0

and then calculate exp(A).


(iv) Do the matrices form a group under matrix multiplication?

Problem 2. Let A be an n n positive definite matrix over R. Let q and J


be column vectors in Rn . Calculate
Z Z  
1 T T
Z(J) = dq1 dqn exp q Aq + J q .
2

Note that

Z r
(aq 2 +bq+c) (b2 4ac)/(4a)
dqe = e . (1)
a

Problem 3. Let A be an n n positive definite matrix over R, i.e. xT Ax > 0


for all x Rn . Calculate Z
exp(xT Ax)dx.
Rn

Problem 4. Consider the 2 2 matrix


 
3 4
A= .
1 1

Calculate An , where n N.

Problem 5. Let A be an n n matrix over C. The n n matrix B over C is


a square root of A iff B 2 = A. The number of square roots of a given matrix A
may be zero, finite or infinite. Does the matrix
 
0 1
A=
0 0

admit a square root?

Problem 6. Show that for any Pauli spin matrix 1 , 2 , 3 we have

sin(j ) = sin()j .
Functions of Matrices 139

Problem 7. Let M be an n n matrix with mjk = 1 for all j, k = 1, 2, . . . , n.


Let s C. Find exp(sM ). Then consider the special case sn = i

Problem 8. Let X, Y be n n matrices. Show that



X [X k , Y ]
[eX , Y ] = .
k!
k=1

Problem 9. Let A, B be n n matrices. Show that


Z 1
eA+B eA e(1t)A Bet(A+B) dt.
0

Problem 10. Let A be an n n matrix. Then exp(A) can also be calculated


as  m
A A
e = lim In + .
m m
Use this definition to show that

det(eA ) etr(A) .

Problem 11. Let A1 , A2 , . . . , Ap be n n matrices over C. The generalized


Trotter formula is given by

Xn
exp Aj = lim fn ({ Aj }) (1)
n
j=1

where the n-th approximant fn ({ Aj }) is defined by


      n
1 1 1
fn ({ Aj }) := exp A1 exp A2 exp Ap .
n n n
Let p = 2 and    
1 0 0 1
A1 = , A2 = .
0 1 1 0
Calculate the left and right-hand side of (1).

Problem 12. Let , R. Calculate


 

exp .
0 0
140 Problems and Solutions

Problem 13. Consider the 2 2 matrix


 
1 1
A= .
0 1

Let t R. Find exp(tA).

Problem 14. Let A, B be n n matrices over C and C. The Baker-


Campbell-Hausdorff formula states that

2 X j
eA BeA = B + [A, B] + [A, [A, B]] + = {Aj , B} = B()
e
2! j=0
j!

where [A, B] := AB BA and

{Aj , B} := [A, {Aj1 , B}]

is the repeated commutator.


(i) Extend the formula to
eA B k eA
where k 1.
(ii) Extend the formula to
eA eB eA .

Problem 15. Consider the n n matrix (n 2)

0 1 0 ... 0

0 0 1 ... 0
. . . .. ..
A= . . .
. . . . ..
0 0 0 ... 1
0 0 0 ... 0

Let f : R R be an analytic function. Calculate

f 0 (0) f 00 (0) 2 f n1
f (0)In + A+ A + + An1
1! 2! (n 1)!

where 0 denotes differentiation. Discuss.

Problem 16. Let A, B be n n matrices. Show that

cos(A + B) = cos(A) cos(B) sin(A) sin(B)

if AB = BA.
Functions of Matrices 141

Problem 17. Consider the 3 3 matrix



0 0
A() = 0 0 0 , R.
0 0
Find exp(A).

Problem 18. Let A, B be n n matrices with A2 = In and B 2 = In . Assume


that the anticommutator of A and B vanishes, i.e.
[A, B]+ = AB + BA = 0n .
Let a, b C. Calculate eaA+bB .

Problem 19. Let A, B be n n matrices with A2 = In and B 2 = In . Assume


that the commutator of A and B vanishes, i.e.
[A, B] = AB BA = 0n .
Let a, b C. Calculate eaA+bB .

Problem 20. Consider the 2 2 matrix


 
a11 a12
A= .
0 a22
Find exp(tA).

Problem 21. Can one find n n matrices A such that ( R)


exp(iA) = In + (cos() 1)A2 + i sin()A ?

Problem 22. Let , C. Let


 

M (, ) = .
0
(i) Calculate exp(M (, )).
(ii) For which values of , C is the matrix nonnormal? Simplify the result
for = i and arbitrary. Is the matrix M ( = i, ) nonnormal? Is the
matrix exp(M ( = i, ) nonnormal?

Problem 23. Consider the two-dimensional rotation matrix


 
cos() sin()
R=
sin() cos()
142 Problems and Solutions

with 0 . Find a square root R1/2 of R, i.e a matrix S such that S 2 = R.

Problem 24. Let f : R R be an analytic function. Let R, n a


normalized vector in R3 and 1 , 2 , 3 the Pauli spin matrices. We define

n := n1 1 + n2 2 + n3 3 .

Then
1 1
f (n ) (f () + f ())I2 + (f () f ())(n ).
2 2
Apply this identity to f (x) = sin(x).

Problem 25. Let a, b R and


 
a b
M= .
b a

Calculate exp(M ).

Problem 26. Let A be an n n matrix over C. Assume that all eigenvalues


1 , 2 , . . . , n are pairwise distinct. Then etA can be calculated as follows
(Lagrange interpolation)
n n
X Y (A k In )
etA = e j t .
j=1
(j k )
k=1
k6=j

Let  
0 1
A= .
1 0
Calculate etA using this method.

Problem 27. Let A be an n n matrix over C. Assume that all eigenvalues


1 , 2 , . . . , n are pairwise distinct. Then etA can be calculated as follows
(Newton interpolation)
n
X j1
Y
tA 1 t
e =e In + [1 , . . . , j ] (A k In ).
j=2 k=1

The divided differences [1 , . . . , j ] depend on t and are defined recursively by

e1 t e2 t
[1 , 2 ] :=
1 2
[1 , . . . , k ] [2 , . . . , k+1 ]
[1 , . . . , k+1 ] := , k 2.
1 k+1
Functions of Matrices 143

Let  
0 1
A= .
1 0
Calculate etA using this method.

Problem 28. Let A, B, C be n n matrices over C such that A2 = In ,


B 2 = In and C 2 = In . Furthermore assume that

[A, B]+ AB+BA = 0n , [B, C]+ BC+CB = 0n , [C, A]+ CA+AC = 0n

i.e. the anticommutators vanish. Let , , C. Calculate eA+B+C using



X (A + B + C)j
eA+B+C = .
j=0
j!

Problem 29. Let A, B be n n matrices. Then we have the identity

det(eA eB eA eB ) exp(tr([A, B]))

where [A, B] := AB BA defines the commutator. Show that

det(eA eB eA eB ) = 1.

Problem 30. Let


     
0 1 0 0 1 0
e= , f= , h= .
0 0 1 0 0 1

(i) Show that


   
1 1 1 0
exp(e) = , exp(f ) = .
0 1 1 1

(ii) Show that  


0 1
exp(e) exp(f ) exp(e) = .
1 0

Problem 31. Let A be an n n matrix. The characteristic polynomial

det(In A) = n + a1 n1 + + an = p()

is closely related the the resolvent (In A)1 trough the formula
N1 n1 + N2 n2 + + Nn N ()
(In A)1 = =
n + a1 n1 + + an p()
144 Problems and Solutions

where the adjugate matrix N () is a polynomial in of degree n 1 with


constant n n coefficient matrices N1 , . . . , Nn . The Laplace transform of the
matrix exponential is the resolvent
L(etA ) = (In A)1 .
The Nk matrices and ak coefficients may be computed recursively as follows
1
N1 = In , a1 = tr(AN1 )
1
1
N2 = AN1 + a1 In , a2 = tr(AN2 )
2
..
.
1
Nn = ANn1 + an1 In , an = tr(ANn )
n
0 = ANn + an In .
Let
1 0 0 1

0 1 1 0
A= .
0 1 1 0
1 0 0 1
Find the Nk matrices and the coefficients ak and thus calculate the resolvent.

Problem 32. Let A be an n n matrix. The characteristic polynomial


det(In A) = n + a1 n1 + + an = p()
is closely related the the resolvent (In A)1 trough the formula
N1 n1 + N2 n2 + + Nn N ()
(In A)1 = n n1
=
+ a1 + + an p()
where the adjugate matrix N () is a polynomial in of degree n 1 with
constant n n coefficient matrices N1 , . . . , Nn . The Laplace transform of the
matrix exponential is the resolvent
L(etA ) = (In A)1 .
The Nk matrices and ak coefficients may be computed recursively as follows
1
N1 = In , a1 = tr(AN1 )
1
1
N2 = AN1 + a1 In , a2 = tr(AN2 )
2
..
.
1
Nn = ANn1 + an1 In , an = tr(ANn )
n
0 = ANn + an In .
Functions of Matrices 145

Show that
1 dp()
tr(L(etA )) = .
p() d

Problem 33. Let A be an n n positive definite matrix over R, i.e. all the
eigenvalues, which are real, are positive. We also have AT = A. Consider the
analytic function f : Rn R
 
1 T 1
f (x) = exp x A x .
2
Calculate the Fourier transform of f . The Fourier transform is defined by
Z

f (k) := f (x)eikx dx
Rn

where kx kT x k1 x1 + +kn xn and dx = dx1 . . . dxn . The inverse Fourier


transform is given by
Z
1
f (x) = f(k)eikx dk
(2)n Rn
where dk = dk1 . . . dkn . Note that we have with a > 0
Z r
2 (b2 4ac)/(4a)
e(ax +bx+c) dx = e .
R a

Problem 34. Let A be an n n matrix. Suppose f is an analytic func-


tion inside on a closed contour which encircles (A), where (A) denotes the
eigenvalues of A. We define f (A) to be the n n matrix
I
1
f (A) = f (z)(zIn A)1 dz.
2i
This is a matrix version of the Cauchy integral theorem. The integral is defined
on an element-by-element basis f (A) = (fjk ), where
I
1
fjk = f (z)eTj (zIn A)1 ek dz
2i

where ej (j = 1, 2, . . . , n) is the standard basis in Cn . Let f (z) = z 2 and


 
0 1
A= .
1 0

Calculate f (A).
146 Problems and Solutions

Problem 35. Let A, B be n n matrices over C. Calculate

eA BeA .

Set f () = eA BeA , where  is a real parameter. Then differentiate with respect
to . For  = 1 we have eA BeA .

Problem 36. Let A, B be positive definite matrices. Then we have the


integral representation (x 0)
Z
ln(A + xB) ln(A) (A + uIn )1 xB(A + xB + uIn )1 du.
0

Let    
2 1 1 1
A= , B= .
1 1 1 2
Calculate the left and right-hand side of the integral representation.

Problem 37. Let  R. Calculate

f () = e2 3 e2 .

Hint. Differentiate the matrix-valued function f with respect to  and solve the
initial value problem of the resulting ordinary differential equation.

Problem 38. Find the square root of the matrix



0 1 0
A = 0 0 0
0 0 0

i.e. find the matrices X such that X 2 = A.

Problem 39. Let A be an arbitrary n n matrix. Can we conclude that

exp(A ) = (exp(A)) ?

Problem 40. Let A be an invertible n n matrix over R. Consider the


functions
1
Ej = (Acj ej )T (Acj ej )
2
where j = 1, . . . , n, cj is the j-th column of the inverse matrix of A, ej is the
j-th column of the n n identity matrix. This means e1 , . . . , en is the standard
Functions of Matrices 147

basis (as column vectors) in Rn . The cj are determined by minimizing the Ej


with respect to the cj . Apply this method to find the inverse of the 3 3 matrix

1 0 1
A = 0 1 0 .
1 0 1

Problem 41. Let A be an n n matrix over C. Assume that A is hermitian,


i.e. A = A. Thus A has only real eigenvalues. Assume that

A5 + A3 + A = 3In .

Show that A = In .

Problem 42. Let f be a function from U , an open subset of Rm , to Rn .


Assume that the component function fj (j = 1, . . . , n) possess first order partial
derivatives. Then we can associate the n m matrix
 !
fj
, j = 1, . . . , n k = 1, . . . , m
xk p

where p U . The matrix is is called the Jacobian matrix of f at the point p.


When m = n the determinant of the square matrix f is called the Jacobian of f .
Let
A = { r R : r > 0 }, B = { R : 0 < 2 }
and f : A B R2 with f1 (r, ) = r cos , f2 (r, ) = r sin . Find the Jacobian
matrix and the Jacobian.

Problem 43. Let  


0 1
J= .
1 0
Find all nonzero 2 2 matrices A such that

AJ = JA.

Problem 44. Let A be an n n matrix over C. Let T be a nilpotent matrix


over C satisfying
T A + AT = 0.
Show that
(eT ) AeT = A.
148 Problems and Solutions

Problem 45. Let A, B be n n matrices over C. Let ,  R. Show that


Z !
A (A+B)
exp((A + B)) exp(A) In + de Be .
0

Problem 46. Consider the matrix



0 1 0
A = 1 0 1.
0 1 0
Let R. Find exp(A).

Problem 47. Let  R. Let


In A
be a positive definite matrix. Calculate
exp(tr(ln(In A))
using the identity det eM exp(tr(M )).

Problem 48. Consider the Pauli spin matrices 1 , 2 and 3 . Can one find
an R such that
exp(i3 )1 exp(i3 ) = 2 ?

Problem 49. (i) Let a, b R. Let


 
a b
K= .
b a
Find exp(iK).
(ii) Use the result to find a, b such that
 
0 1
exp(iK) = .
1 0

Problem 50. Let P be an n n projection matrix. Let  R. Calculate


exp(P ).

Problem 51. (i) Let P1 , P2 , . . . , Pn be an n n projection matrices. Assume


that Pj Pk = 0 (j 6= k) for all j, k = 1, 2, . . . , n. Let j R with j = 1, 2, . . . , n.
Calculate
exp(1 P1 + 2 P2 + + n Pn ).
Functions of Matrices 149

(ii) Assume additionally that

P1 + P2 + + Pn = I n .

Simplify the result from (i) using this condition.

Problem 52. Let A, B be nn hermitian matrices. There exists nn unitary


matrices U and V (depending on A and B) such that

exp(iA) exp(iB) = exp(iU AU 1 + iV BV 1 ).

Consider n = 2 and
   
0 1 1 1 1
A= , B= .
1 0 2 1 1

Find U and V . Note that A and B are also unitary and represend the NOT-gate
and Hadamard gate, respectively. Furthermore

 
0 1
[A, B] = 2 .
1 0

Problem 53. Let a, b C and


 
a b
M (a, b) = .
0 a

Calculate exp(M (a, b)).

Problem 54. Let A, B be n n matrices over C. Assume that A and B


commute with the commutator [A, B]. Then
 
1
exp(A + B) = exp(A) exp(B) exp [A, B] .
2

Can this formula be applied to the matrices


   
1 0 0 1
C= , D= .
0 0 0 0

Problem 55. Let  R. Let A, B be n n matrices over C. Expand

eA eB eA eB

up to second order in .
150 Problems and Solutions

Problem 56. Let , R. Consider the 2 2 matrix


 
i
B= .
i

Find exp(tB), where t R and thus solve the initial value problem of the matrix
differential equation
dA
= BA(t).
dt

Problem 57. Let A be an nn matrix over C. Assume that for all eigenvalues
we have <() < 0. Let B be an arbitrary n n matrix over C. Let
Z

R := etA BetA .
0

Show that the matrix R satisfies the matrix equation

RA + A R = B.

Problem 58. Let A, B be n n matrices over C such that [A, B] = A. What


can be said about the commutator

[eA , eB ] ?

Problem 59. Consider the positive semidefinite matrix


 
2 1
A= .
1 2
Use the right-hand side of the identity

det(A) exp(tr(ln(A)))

to calculate det(A).

Problem 60. Let A, B be n n matrices over C and A2 = In , B 2 = In .


Calculate
exp(z1 A + z2 B)
where z1 , z2 C.

Problem 61. Let z C. Let A, B be n n matrices over C. We say that B


is invariant with respect to A if

ezA BezA = B.
Functions of Matrices 151

Obviously ezA is the inverse of ezA . Show that, if this condition is satisfied,
one has [A, B] = 0n , where 0n is the n n zero matrix. If ezA would be unitary
we have U BU = B.

Problem 62. Let z C and


   
0 1 b11 b12
A= , B= .
1 0 b12 b11

(i) Calculate exp(zA), exp(zA) and exp(zA)B exp(zA).


(ii) Calculate the commutator [A, B].

Problem 63. Let z C and


   
0 i b11 b12
A= , B= .
i 0 b12 b11

(i) Calculate exp(zA), exp(zA) and exp(zA)B exp(zA).


(ii) Calculate the commutator [A, B].

Problem 64. Let z C and


   
0 1 b11 b12
A= , B= .
0 0 0 b11

(i) Calculate exp(zA), exp(zA) and exp(zA)B exp(zA).


(ii) Calculate the commutator [A, B].

Problem 65. Consider the Pauli spin matrices 1 , 2 , 3 . Find the skew-
hermitian matrices 1 , 2 , 3 such that

1 = exp(1 ), 2 = exp(2 ), 3 = exp(3 ).

Find the commutators [1 , 2 ], [2 , 3 ], [3 , 1 ] and compare with the commu-


tators [1 , 2 ], [2 , 3 ], [3 , 1 ].

Problem 66. Let R. Consider the matrix


 
cos sin
A() = .
sin cos

(i) Show that the matrix is orthogonal.


(ii) Find the determinant of A(). Is the matrix an element of SO(2, R)?
(iii) Do these matrices form a group under matrix multiplication?
(iv) Calculate
d
X= A() .
d =0
152 Problems and Solutions

Calculate exp(X) and compare this matrix with A(). Discuss.


(v) Let R and  
cos sin
B() = .
sin cos
Is the matrix A() B() orthogonal? Find the determinant of A() B().
Is this matrix an element of SO(4, R)?

Problem 67. We know that for any n n matrix A over C the matrix exp(A)
is invertible with the inverse exp(A). What about cos(A) and cosh(A)?

Problem 68. (i) Let  R. Let A be an n n matrix over C. Find

sinh(2A)
lim .
0 sinh()

(ii) Assume that A2 = In . Calculate

sinh(2A)
.
sinh()

(iii) Assume that A2 = 0n . Calculate

sinh(2A)
.
sinh()

Problem 69. Consider the matrices


   
0 1 0 0
A= , B= .
0 0 1 0

Is sin(A) invertible? Is cos(A) invertible? Is sin(B) invertible? Is cos(B) invert-


ible?

Problem 70. Is cos(A) invertible for all n n matrices A over C?

Problem 71. Let A be a nilpotent matrix. Is the matrix cos(A) invertible?

Problem 72. Consider the 2 2 matrices


   
0 1 0 0
A= , B= .
0 0 1 0

Find cosh(A), sinh(A), cosh(B), sinh(B). Which of these matrices are invertible?
Functions of Matrices 153

Problem 73. Let A be an n n matrix over C. Find


sinh(A)
lim .
0 sinh()

Problem 74. Consider the matrix



1 0 1
A = 0 1 0.
1 0 1
Write the matrix A in the form

0 0 1
A = I3 + B, B = 0 0 0
1 0 0
and calculate eA using eA = eI3 eB .

Problem 75. Consider the 3 3 matrix



0 i 0
A = 0 0 i .
0 i 0
Calculate exp(iA), where R.

Problem 76. Let A be an n n matrix. The characteristic polynomial


det(In A) = n + a1 n1 + + an = p()
is closely related the the resolvent (In A)1 trough the formula
N1 n1 + N2 n2 + + Nn N ()
(In A)1 = =
n + a1 n1 + + an p()
where the adjugate matrix N () is a polynomial in of degree n1 with constant
n n matrices N1 , . . . , Nn . The Laplace transform of the matrix exponential is
the resolvent
L(etA ) = (In A)1 .
The Nk matrices and ak coefficients may be computed recursively as follows
1
N1 = In , a1 = tr(AN1 )
1
1
N2 = AN1 + a1 In , a2 = tr(AN2 )
2
..
.
1
Nn = ANn1 + an1 In , an = tr(ANn )
n
0 = ANn + an In .
154 Problems and Solutions

Let
1 0 0 1

0 1 1 0
A= .
0 1 1 0
1 0 0 1
Find the Nk matrices and the coefficients ak and thus calculate the resolvent.

Problem 77. Let A be a normal matrix with eigenvalues 1 , . . . , n and corre-


sponding normalized pairwise orthogonal eigenvectors u1 , . . . , un . Let w, v Cn
(column vectors). Find
w eA v
by expanding w and v with respect to the basis uj (j = 1, . . . , n).

Problem 78. Consider the matrices


   
0 1 0 0
A= , B= .
0 0 1 0

Find cos(A) and the inverse of this matrix. Find cos(B) and the inverse of this
matrix. Find the commutators [A, B] and [cos(A), cos(B)]. Discuss.

Problem 79. Let V be the 2 2 matrix

V = v0 I2 + v1 1 + v2 2 + v3 3

where v0 , v1 , v2 , v3 R. Consider the equation

exp(iV ) = (I2 iW )(I2 + iW )1

where  is real. Find W as a function of V .

Problem 80. Consider the rotation matrix


 
cos(t) sin(t)
R(t) =
sin(t) cos(t)

where is the fixed frequency. Find the matrix


dR(t) T
H(t) = i~ R (t)
dt
and show it is hermitian.

Problem 81. (i) Let 1 be the Pauli matrix


 
0 1
1 = .
1 0
Functions of Matrices 155

Calculate  
1
exp i(1 I2 ) .
2
(ii) Find all 2 2 matrices A and c C such that

exp(c(A I2 )) = A.

Problem 82. Let B be an n n matrix with B 2 = In . Show that


 
1
exp i(B In ) B.
2

Problem 83. Let A, B be n n matrices over C and

exp(A) exp(B) = exp(C).

Then the matrix C can be given as an infinite series of commutators of A and


B. Let z C. We write

exp(zA) exp(zB) = exp(C(zA, zB))

where

X
C(zA, zB) = cj (A, B)z j .
j=1

Show that the expansion up to fourth order is given by

c1 (A, B) = A + B
1
c2 (A, B) = [A, B]
2
1 1
c3 (A, B) = [A, [A, B]] [B, [A, B]
12 12
1
c4 (A, B) = [A, [B, [A, B].
24

Problem 84. The 2 2 matrices


   
0 1 0 0
A= , B=
0 0 1 0

are nonnormal, i.e. AA 6= A A and BB 6= B B. Note that A = B. Are the


matrices
exp(A), exp(B)
156 Problems and Solutions

normal? Are the matrices sin(A), sin(B), cos(A), cos(B) normal?

Problem 85. Let a, b C. Find


 
a b
exp .
0 a

Problem 86. Find the unitary matrix

U (t) = ei sin(t)1

with the Pauli spin matrix 1 .

Problem 87. Let N be a nilpotent n n matrix with N j = 0n with j 1. Is


 
1 1
ln(In N ) = N + N + + N j1
2 j1
and exp(ln(In N ) = In N .

Problem 88. Let H = H1 + H2 + H3 and H1 , H2 , H3 be n n hermitian


matrices. Show that
 3
H H1 /2 H2 /2 H3 H2 /2 H1 /2 1
e =e e e e e + S
2
where
1
S= ([[H2 + H3 , H1 ], H1 + 2H2 + 2H3 ] + [[H3 , H2 ], H2 + 2H3 ]).
6

Problem 89. Consider the matrices


   
0 1 1 0
1 = , 3 = .
1 0 0 1
Calculate the matrix  
1
U = exp i (1 + 3 ) .
2
Is the matrix U unitary? Prove or disprove. If so find the group generated by
U.

Problem 90. Consider the 2 2 matrix


 
a11 a12
A=
a21 a22
Functions of Matrices 157

over the complex numbers. Find exp(A).

Problem 91. Let z C. Let A be an n n matrix over C. Assume that


A3 = zA. Find exp(A).

Problem 92. Let a12 , a13 , a23 R. Consider the skew-symmetric matrix

0 a12 a13
A(a12 , a13 , a23 ) = a12 0 a23 .
a13 a23 0

Find exp(A).

Problem 93. Consider the matrix


 
cos() sin()
A() = .
sin() cos()

Is
dA() dA()
A() = A() ?
d d

Problem 94. Let A, B be n n matrices over C. Assume that the inverse of


A and (A + B) exist. Show that

(A + B)1 = A1 A1 B(A + B)1 .

Apply the identity to A = 3 and B = 1 .

Problem 95. (i) Let 1 , 2 , 3 be the Pauli spin matrices. Let z C.


Calculate
cosh(zj ), sinh(zj ) j = 1, 2, 3.
(ii) Let R. Let f : R R be an analytic function. Calculate
   
d d
cosh f (x), sinh f (x).
dx dx

Problem 96. Let A, B be n n matrices. We know that


1 1
eA BeA = B + [A, B] + [A, [A, B]] + [A, [A, [A, B]]] +
2! 3!
Find
eA eB eA .
158 Problems and Solutions

Problem 97. Consider the matrix


 
0 1
A(z1 , z2 ) = .
z1 z2

Find
exp(zA(z1 , z2 )).

Problem 98. Consider the 2 2 matrix


 
cosh() sinh()
A() = .
sinh() cosh()

Find the maxima and minima of the function

f () = tr(A2 ()) (tr(A()))2 .

Problem 99. Consider the 3 3 matrix



0 1 0
i
S2 = 1 0 1 .
2 0 1 0

Is the matrix hermitian? Find the eigenvalues and eigenvectors of S2 .


(ii) Is S 3 = S? Prove or disprove.
(iii) Let R. Find
exp(iS2 ).

Problem 100. Let x, y R. We know that

sin(x + y) = sin(x) cos(y) + cos(x) sin(y).

Let    
1 1 0 1
A= , B= .
1 1 1 0
Is
sin(A + B) = sin(A) cos(B) + cos(B) sin(B).
Prove or disprove.

Problem 101. Let A1 , . . . , Am be n n matrices. Show that


m
1X
eA1 eAm = exp( [Aj , Ak ]) exp(A1 + + Am )
2
j<k
Functions of Matrices 159

if the matrices Aj (j = 1, . . . , m) satisfy

[[Aj , Ak ], A` ] = 0 for all j, k, ` {1, . . . , m}.

Problem 102. (i) Consider the three (hermitian) spin-1 matrices



0 1 0 0 1 0 1 0 0
1 i
S1 = 1 0 1 , S2 = 1 0 1 , S3 = 0 0 0
2 0 1 0 2 0 1 0 0 0 1

all with the eigenvalues +1, 0 and 1. Show that Sj3 = Sj .


(ii) Let R. Show that

exp(iSj ) = I3 + i sin()Sj (1 cos())Sj2

which is a unitary matrix.

Problem 103. Let A, B be n n matrices over C with A2 = B 2 = In and


[A, B]+ = 0n , i.e. the anticommutator vanishes. Let z C. The Lie-Trotter
formula is given by
 p
exp(z(A + B)) = lim ezA/p ezB/p .
p

Calculate ez(A+B) using the right-hand side.

Problem 104. The exponential of an n n matrix A is defined as



X Ak
exp(A) = .
k!
k=0

Calculate     
0 1 1
exp
0 0 0
in two different ways. Compare and discuss.

Problem 105. Let v1 , v2 be an orthonormal set in C2 . Consider the 2 2


matrix
A = iv1 v1 + iv2 v2 .
Thus I2 = v1 v1 + v2 v2 . Find K such that exp(K) = A.

Problem 106. Let z1 , z2 , z3 C. Consider the 2 2 matrix


 
z1 z2
A(z1 , z2 , z3 ) = .
z3 0
160 Problems and Solutions

Calculate
exp(A(z1 , z2 , z3 ))
using

X Aj (z1 , z2 , z3 )
exp(A(z1 , z2 , z3 )) = .
j=0
j!

Problem 107. Any 2 2 matrix can be written as a linear combination of the


Pauli spin matrices and the 2 2 identity matrix

A = aI2 + b1 + c2 + d3

where a, b, c, d C.
(i) Find A2 and A3 .
(ii) Use the result from (i) to find all matrices A such that A3 = 1 .

Problem 108. Let A, B be n n hermitian matrices. Then there exit n n


unitary matrices U and V such that

exp(iA) exp(iB) = exp(iU AU 1 + iV BV 1 ).

Let n = 2, A = 1 , B = 3 . Find U and V .

Problem 109. Let A, B be n n matrices with kBk  kAk. Then we have


the expansion  Z 1 
A+B A tA tA
e =e In + e Be dt +
0
Apply the equation to
   
0 1 1/4 0
A= , B= .
1 0 0 1/4

Problem 110. Consider the Bell matrix


1 0 0 1

1 0 1 1 0
B=
2 0 1 1 0

1 0 0 1
which is a unitary matrix.
(i) Apply the spectral theorem to find the skew-hermitian matrix K such that
B = eK .
(ii) Apply the Cayley-Hamilton theorem to find the skew-hermitian matrix K
such that B = eK .
Functions of Matrices 161

Problem 111. Find the square root of


0 0 0 1

   
0 0 1 0 0 1 0 1
U = =
0 1 0 0 1 0 1 0
1 0 0 0
applying the spectral theorem.

Problem 112. (i) Let R. Find the eigenvalues and eigenvectors of the
2 2 matrix  
1 1
A() = .
1 + 2 1
For = 0 we obtain the Pauli spin matrix 1 , for = 1 we have the Hadamard
matrix and for we obtain the Pauli spin matrix 3 .
(ii) Let R. Find the eigenvalues and eigenvectors of the 2 2 matrix
 
tanh() 1/ cosh()
B() =
1/ cosh() tanh()

The matrices A() and B() are connected via the invertible transformation
sinh().

Problem 113. (i) Find the square roots of


 
0 i
i1 = .
i 0

(ii) Find the square roots of


0 0 0 1

0 0 1 0
(i1 ) (i1 ) = .
0 1 0 0
1 0 0 0

Problem 114. Let R and P be an n n projection matrix. Let Q = P 1/2


be a square root of P , i.e. Q2 = P . Find

U () = exp(iP 1/2 ).

Problem 115. Calculating exp(A) we can also use the Cayley-Hamilton the-
orem and the Putzer method. Using the Cayley-Hamilton theorem we can write

f (A) = an1 An1 + an2 An2 + + a2 A2 + a1 A + a0 In (1)


162 Problems and Solutions

where the complex numbers a0 , a1 , . . . , an1 are determined as follows: Let

r() := an1 n1 + an2 n2 + + a2 2 + a1 + a0

which is the right-hand side of (1) with Aj replaced by j (j = 0, 1, . . . , n 1).


For each distinct eigenvalue j of the matrix A, we consider the equation

f (j ) = r(j ). (2)

If j is an eigenvalue of multiplicity k, for k > 1, then we consider also the


following equations

f 0 ()|=j = r0 ()|=j , , f (k1) () = r(k1) () .

=j =j

Any unitary matrix U can be written as U = exp(iK), where K is hermitian.


Apply this method to find K for the Hadamard gate
 
1 1 1
UH = .
2 1 1

Problem 116. Let z C. Construct all 2 2 matrices A and B over C such


that
exp(zA)B exp(zA) = ez B.

Problem 117. Let A be an n n matrix over C. Consider the Taylor series


1 1 2 13 3
(In + A)1/2 = In + A A + A
2 24 246
and
1 13 2 135 3
(In + A)1/2 = In A + A A +
2 24 246
What is the condition (the norm) on A such that the Taylor series exist? Can
it be applied to the matrix  
1 1
A= ?
1 1
Note that for n = 1 we have the condition 1 < A +1.

Problem 118. Let U be an n n unitary matrix. Let H = U + U . Calculate


exp(zH).

Problem 119. Let A be an n n matrix. The characteristic polynomial

det(In A) = n + a1 n1 + + an = p()
Functions of Matrices 163

is closely related the the resolvent (In A)1 trough the formula

N1 n1 + N2 n2 + + Nn N ()
(In A)1 = =
n + a1 n1 + + an p()

where the adjuagte matrix N () is a polynomial in of degree n1 with constant


n n matrices N1 , cdots, Nn . The Laplace transform of the matrix exponential
is the resolvent
L(etA ) = (In A)1 .
The Nk matrices and ak coefficients may be computed recursively as follows
1
N1 = In , a1 = tr(AN1 )
1
1
N2 = AN1 + a1 In , a2 = tr(AN2 )
2
..
.
1
Nn = ANn1 + an1 In , an = tr(ANn )
n
0 = ANn + an In .

Let
1 0 0 1

0 1 1 0
A= .
0 1 1 0
1 0 0 1
Find the Nk matrices and the coefficients ak and thus calculate the resolvent.

Problem 120. Let A, B, C be n n positive semidefinite matrices. We define

A B := A1/2 BA1/2

where A1/2 denotes the unique positive square root of A. Is

A (B C) = (A B) C ?

Prove or disprove.

Problem 121. Let > 0 and


0 0 0 i/

0 0 i 0
A() = .
0 i 0 0
i 0 0 0

Find exp(A()).
164 Problems and Solutions


Problem 122. Calculate 3 1 . Proceed as follows: Start with

A = aI2 + b1 + c2 + d3

where a, b, c, d C. The Pauli matrices 1 , 2 , 3 together with I2 form a basis


of the 2 2 matrices over C. Calculate A2 and A3 and set A3 = 1 . Solve the
four equations for a, b, c, d.

Problem 123. Let A, B be n n matrices. Show that



X 1
[eA , eB ] = eA eB eB eA = [Aj , B k ].
j!k!
j,k=1
Chapter 8

Cayley-Hamilton Theorem

Consider an n n matrix A over C and the polynomial

p() = det(A In )

with the characteristic equation p() = 0. The Cayley-Hamilton theorem states


that substituting the matrix A in the characteristic polynomial results in the
n n zero matrix.

Problem 1. Apply the Cayley-Hamilton theorem to the 3 3 matrix A and


express the result using the trace and determinant of A.

Problem 2. Let A be an n n matrix. Let

n1
X
c(z) := det(zIn A) = z n ck z k
k=0

be the characteristic polynomial of A. Apply the Cayley-Hamilton theorem


c(A) = 0 to calculate exp(A).

Problem 3. (i) Let A be an n n matrix with A3 = In . Calculate exp(A)


using

X
exp(A) = Aj /(j!).
j=0

165
166 Problems and Solutions

(ii) Let

0 1 0
B = 0 0 1
1 0 0
with B 3 = I3 . Calculate exp(B) using the result from (i).
(iii) Calculate exp(B) applying the Cayley-Hamilton theorem.

Problem 4. The Cayley-Hamilton theorem can also be used to calculate


exp(A) and other entire functions for an n n matrix. Let A be an n n matrix
over C. Let f be an entire function, i.e., an analytic function on the whole
complex plane, for example exp(z), sin(z), cos(z). An infinite series expansion
for f (A) is not generally useful for computing f (A). Using the Cayley-Hamilton
theorem we can write

f (A) = an1 An1 + an2 An2 + + a2 A2 + a1 A + a0 In (1)

where the complex numbers a0 , a1 , . . . , an1 are determined as follows: Let

r() := an1 n1 + an2 n2 + + a2 2 + a1 + a0

which is the right-hand side of (1) with Aj replaced by j , where j = 0, 1, . . . , n


1 of each distinct eigenvalue j of the matrix A, we consider the equation

f (j ) = r(j ). (2)

If j is an eigenvalue of multiplicity k, for k > 1, then we consider also the


following equations

f 0 ()|=j = r0 ()|=j
f 00 ()|=j = r00 ()|=j
=
(k1)
f () = r(k1) () .

=j =j

(i) Apply this technique to find exp(A) with


 
c c
A= , c R, c 6= 0.
c c

(ii) Use the method given above to calculate exp(iK), where the hermitian 2 2
matrix K is given by
 
a b
K= , a, c R, b C.
b c
Cayley-Hamilton Theorem 167

Problem 5. Let z1 , z2 , z3 C. Assume that at least two of the three complex


numbers are non-zero. Consider the matrix
 
z1 z2
A= .
z3 0

Calculate exp(A) applying the Cayley-Hamilton theorem. The characteristic


equation for A is given by

2 z1 z2 z3 = 0

with the eigenvalues


1 1
q q
+ = z1 + z12 + 4z2 z3 , = z1 z12 + 4z2 z3 .
2 2
Cayley-Hamilton theorem then states that

A2 z1 A z1 z3 I2 = 02 .

Problem 6. Calculate

0 0

2 2
0

2

2
0
sec
.
0
2
2 0


2
0 0 2
Chapter 9

Linear Differential
Equations

Problem 1. Let A be an n n matrix over R. Consider the initial value


problem of the system of linear differential equations
du(t)
+ Au(t) = g(t), u(0) = u0 (1)
dt
where g(t) = (g1 (t), g2 (t), . . . , gn (t))T . The solution of the initial value problem
is
t
Z
u(t) = etA u0 + e(t )A g( )d. (2)
0

(i) Discretize the system with the implicit Euler method with step size h.
(ii) Compare the two solutions of the two systems for the matrix

0 0 1
A = 0 1 0,
1 0 0

initial values u0 = (1 , 1 , 1)T with g(t) = (1, 0, 1)T and the step size h = 0.1.

Problem 2. Let L and K be two n n matrices. Assume that the entries


depend on a parameter t and are differentiable with repect to t. Assume that
K 1 (t) exists for all t. Assume that the time-evolution of L is given by

L(t) = K(t)L(0)K 1 (t).

168
Linear Differential Equations 169

(i) Show that L(t) satisfies the matrix differential equation

dL
= [L, B](t)
dt
where [ , ] denotes the commutator and

dK 1
B= K (t).
dt
(ii) Show that if L(t) is hermitian and K(t) is unitary, then the matrix B(t) is
skew-hermitian.

Problem 3. Consider a system of linear ordinary differential equations with


periodic coefficients  
du u2
= A(t)u, u=
dt u2
where A(t) is a 2 2 matrix of periodic functions with period T . By the classical
Floquet theory, any fundamental matrix (t), which is defined as a nonsingular
matrix satisfying the matrix differential equation
d
= A(t)(t)
dt
can be expressed as
(t) = P (t) exp(T R).
Here P (t) is nonsingular matrix of periodic functions with the same period T ,
and R, a constant matrix, whose eigenvalues 1 and 2 are called the character-
istic exponents of the periodic sytem (1). For a choice of fundamental matrix
(t), we have
exp(T R) = (t0 )(t0 + T )
which does not depend on the initial time t0 . The matrix exp(T R) is called the
monodromy matrix of the periodic system (1). Calculate

tr exp(T R).

Problem 4. Let  
1 1
A= .
0 1
(i) Calculate exp(tA), where t R.
(ii) Find the solution of the initial value problem of the differential equation
   
du1 /dt u1
=A
du2 /dt u2
170 Problems and Solutions

with the initial conditions u1 (t = 0) = u10 , u2 (t = 0) = u20 . Use the result from
(i).

Problem 5. Solve the initial value problem for the matrix differential equation

dA
[B, A()] =
d
where A() and B are 2 2 matrices with
 
0 1
B= .
1 0

Problem 6. Consider the initial problem of the matrix differential equation

dX
= A(t)X, X(0) = In
dt
where A(t) is an n n matrix which depends smoothly on t and In is the n n
identity matrix. It is known that the solution of this matrix differential equation
can locally be written as
X(t) = exp((t))
where (t) is obtained as an infinite series

X
(t) = k (t).
k=1

This is the so-called Magnus expansion.


Implement this recursion in SymbolicC++ and apply it to
 
cos(t) sin(t)
A(t) = .
sin(t) cos(t)

Problem 7. Let a, b R. Consider the linear matrix differential equation

d2 X dX
+a + bX = 0.
dt2 dt
Find the solution of the initial value problem.

Problem 8. Let A be an n n matrix over R. The autonomous system of


first order differential equations du/dt = Au admits the solution of the initial
Linear Differential Equations 171

value problem u(t) = exp(A)u(0). Differentiation of the differential equations


yields the second order system
d2 u du
=A = A2 u.
dt2 dt
Thus we can write
du dv
= v = Au, = A2 u = Av
dt dt
or in matrix form     
du/dt 0n In u(0)
=
dv/dt A2 0 n v(0)
where 0n is the n n zero matrix and In is the n n identity matrix. Find the
solution of the initial value problem. Assume that A is invertible.

Problem 9. (i) Let A be an n n matrix over R. Let u = (u1 , . . . , un )T .


Solve the initial value problem of the system of linear differential equations
du
= Au
dt
using the Laplace transform, where u(t = 0) = u(0). Note that (applying
integration by parts)
Z
du
L(du/dt)(s) = est dt
0 dt
Z
st t=
est u(t)dt

= e u(t) t=0 + s

0
= sU(s) u(0).
(ii) Apply it to the 2 2 Hadamard matrix
 
1 1 1
H= .
2 1 1

Problem 10. Let  


1 1
A= .
0 1
(i) Calculate exp(tA), where t R.
(ii) Find the solution of the initial value problem of the differential equation
   
du1 /dt u1
=A
du2 /dt u2
with the initial conditions u1 (t = 0) = u10 , u2 (t = 0) = u20 . Use the result from
(i).
Chapter 10

Norms and Scalar Products

Problem 1. Let U1 , U2 be unitary n n matrices. Let v be a normalized


vector in Cn . Consider the norm of a k k matrix M
kM k = max kM xk
kxk=1

where kxk denotes the Euclidean norm. Show that if kU1 U2 k  then
kU1 v U2 vk .

Problem 2. Given the 2 2 matrix


 
cos sin
A() = .
sin cos
Calculate
kA()k = sup kA()xk.
kxk=1

Problem 3. Let A be an n n matrix. Let (A) be the spectral radius of A.


Then we have
Xn n
X
(A) min { max |aij |, max |aij | }.
1in 1jn
j=1 i=1

Let
0 1 2
A = 3 4 5.
6 7 8

172
Norms and Scalar Products 173

Calculate (A) and the right-hand side of the inequality.

Problem 4. Consider the Hilbert space Cn . We define a norm of an n n


matrix A over C
kAk := sup kAxk
kxk=1

where the right-hand side denotes the Euclidean norm. Let U be an nn unitary
matrix. Show that kU k = 1.

Problem 5. Let A be an n n positive semidefinite (and thus hermitian)


matrix. Is
kA1/2 k = kAk1/2 ?

Problem 6. Let A be an n n positive semidefinite matrix. Show that



|x Ay| x Ax y Ay
p

for all x, y C.

Problem 7. Let t R. Consider the symmetric matrix over R



t 1 0
A(t) = 1 t 1 .
0 1 t

Find the condition on t such that (A(t)) < 1, where (A(t)) denotes the spectral
radius of A(t).

Problem 8. (i) Let A be an n n positive semidefinite matrix. Show that


(In + A)1 exists.
(ii) Let B be an arbitrary n n matrix. Show that the inverse of In + B B
exists.

Problem 9. Let A be an n n matrix. One approach to calculate exp(A) is to


compute an eigenvalue decomposition A = XBX 1 and then apply the formula
eA = XeB X 1 . We have using the Schur decomposition

U AU = diag(1 , . . . , n ) + N

where U is unitary, the matrix N = (njk ) is a strictly upper triangular (njk =


0, j k) and (A) = { 1 , . . . , n } is the spectrum of A. Using the Pade
approximation to calculate eA we have

Rpq = (Dpq (A))1 Npq (A)


174 Problems and Solutions

where
p
X (p + q j)!p!
Npq (A) := Aj
j=0
(p + q)!j!(p j)!
q
X (p + q j)!q!
Dpq (A) := (A)j .
j=0
(p + q)!j!(q j)!

Let
0 6 0 0

0 0 6 0
A= .
0 0 0 6
0 0 0 0
Calculate kR11 eA k, where k k denotes the 2-norm.

Problem 10. Let A be an n n matrix with kAk < 1. Then ln(In + A) exists.
Show that
kAk
k ln(In + A)k .
1 kAk

Problem 11. Let A, B be n n matrices over C. Show that

k[A, B]k 2kAk kBk

where [ , ] denotes the commutator.

Problem 12. Let A be an n n matrix. Let



A In 0n
B = In A In
0n In A

where 0n is the n n zero matrix. Calculate B 2 and B 3 .

Problem 13. Denote by k kHS the Hilbert-Schmidt norm and by k kO the


operator norm, i.e.
p
kAkHS := tr(AA ), kAkO := sup kAxk = sup
kxk=1 C, kxk=1
(A A)x=x

where A is an m n matrix over C, x Cn and kxk is the Euclidean norm.


(i) Calculate
 1 1
  1 1

21 0 12 21 0 12 .

and

2 0 2 HS
2 0 2 O
Norms and Scalar Products 175

(ii) Let A be an m n matrix over C. Find the conditions on A such that

kAkHS = kAkO .

Problem 14. Let A, B be n n matrices over C. Then

k[A, B]k kABk + kBAk 2kAk kBk

where k k denotes the norm. Let


   
0 1 1 1 1
A= , B= .
1 0 2 1 1

Calculate k[A, B]k, kABk, kBAk, kAk, kBk pand thus verify the inequality for
these matrices. The norm is given by kCk = tr(CC ).

Problem 15. Let A be an n n matrix. The logarithmic norm is defined by

kIn + hAk 1
[A] := lim .
h0+ h
Let
kAk := sup |Axk.
x=1

Let A be the n n identity matrix In . Find [In ].

Problem 16. Find a 2 2 unitary matrix U such that


   
0 1 0 0
U U= .
0 0 1 0

Problem 17. Consider the Hilbert space Rn . The scalar product hx, yi x, y
Rn is given by
Xn
hx, yi := xT y = xj yj .
j=1
p
Thus the norm is given by kxk = hx, xi. Show that

|xT y| kxk kyk.

Problem 18. Let { v1 , . . . , vm } be a linearly independent set of vectors in


the normed space Rn with m n.
176 Problems and Solutions

(i) Show that there is a number c > 0 such that for every choice of real numbers
c1 , . . . , cm we have
kc1 v1 + + cm vm k c (|c1 | + + |cm |) . (1)
(ii) Consider R2 and    
1 1
v1 = , v2 = .
0 1
Find a c for this case.

Problem 19. Let A be an n n hermitian matrix. Let u, v Cn and C.


Consider the equation
Au u = v.
(i) Show that for nonreal (i.e. it has an imaginary part) the vector v cannot
vanish unless u vanishes.
(ii) Show that for nonreal we have
1
k(A In )1 vk kvk.
|=|

Problem 20. (i) Let A and C be invertible n n matrices over R. Let B be


an n n matrix over R. Assume that
kAk kBk kCk.
Is B invertible?
(ii) Let A, B, C be invertible n n matrices over R with
kAk kBk kCk.
Is
kA1 k kB 1 k kC 1 k ?

Problem 21. Let A be an n n matrix over R. Assume that kAk < 1, where
kAk := sup kAxk.
kxk=1

Show that the matrix B = In + A is invertible, i.e. B GL(n.R). To show that


the expansion
In A + A2 A3 +
converges apply
kAm Am+1 + Am+2 Am+k1 k kAm k k1 + kAk + + kAkk1 k
1 kAkk
= kAkm .
1 kAk
Norms and Scalar Products 177

Then calculate (In + A)(In A + A2 A3 + ).

Problem 22. Let A be an n n matrix over C. Let k.k be a subordinate


matrix norm for which kIn k = 1. Assume that kAk < 1.
(i) Show that (In A) is nonsingular.
(ii) Show that k(In A)1 k (1 kAk)1 .

Problem 23. Let A be an n n matrix. Assume that kAk < 1. Show that

kAk
k(In A)1 In k .
1 kAk

Problem 24. Let A be an n n nonsingular matrix and B an n n matrix.


Assume that kA1 Bk < 1.
(i) Show that A B is nonsingular.
(ii) Show that
kA1 (A B)1 k kA1 Bk
.
kA1 k 1 kA1 Bk

Problem 25. Let M be an m n matrix over C. The Frobenius norm of M


is given by p p
kM kF := tr(M M ) = tr(M M ).
Let Um be m m unitary matrix and Un be an n n unitary matrix. Show that

kUm M kF = kM Un kF = kM k.

Show that kM kF is the square root of the sum of the squares of the singular
values of M .

Problem 26. Let M be an m n matrix over C. Find the rank-1 m n


matrix A over C which minimizes

kM AkF .

Hint: Find the singular value decomposition of M = U V and find A0 with


rank 1 which minimizes
k A0 kF .
Apply the method to
0 1
M = 1 0.
0 1
178 Problems and Solutions

Problem 27. Let A be an n n nonsingular matrix and B an n n matrix.


Assume that kA1 Bk < 1.
(i) Show that A B is nonsingular.
(ii) Show that
kA1 (A B)1 k kA1 Bk
.
kA1 k 1 kA1 Bk

Problem 28. Let A be an n n matrix over C. The spectral radius of the


matrix A is the non-negative number (A) defined by

(A) := max{ |j (A)| : 1 j n }

where j (A) (j = 1, 2, . . . , n) are the eigenvalues of A. We define the norm of A


as
kAk := sup kAxk
kxk=1

where kAxk denotes the Euclidean norm. Is (A) kAk? Prove or disprove.

Problem 29. Let A be an n n matrix over R. The spectral norm is

kAxk2
kAk2 := max .
x6=0 kxk2

It can be shown that kAk2 can also be calculated as


p
kAk2 = largest eigenvalue of AT A.

Note that the eigenvalues of AT A are real and nonnegative. Let


 
2 5
A= .
1 3

Calculate kAk2 using this method.


Chapter 11

Graphs and Matrices

Problem 1. A graph G(V, E) is a set of nodes V (points, vertices) connected


by a set of links E (edges, lines). We assume that there are n nodes. The
adjancy (n n) matrix A = A(G) takes the form with 1 in row i, column j if i
is connected to j, and 0 otherwise. Thus A is a symmetric matrix. Associated
with A is the degree distribution, a diagonal matrix with row-sums of A along
the diagonal, and 0s elsewhere. We assume that dii > 0 for all i = 1, 2, . . . , n.
We define the Laplacian as L := D A. Let

0 1 1 0 0 0 0

1 0 1 1 0 0 0
1 1 0 1 0 1 0

A = 0 1 1 0 0 1 0.

0 0 0 0 0 1 0

0 0 1 1 1 0 1

0 0 0 0 0 1 0

(i) Give an interpretation of A, A2 , A3 .


(ii) Find D and L.
(iii) Show that L admits the eigenvalue 0 = 0 (lowest eigenvalue) with eigen-
vector x = (1, 1, 1, 1, 1, 1, 1)T .

Problem 2. A graph G(V, E) is a set of nodes V (points, vertices) connected


by a set of links E (edges, lines). We assume that there are n nodes. The
adjacency (n n) matrix A = A(G) takes the form with 1 in row i, column j if
i is connected to j, and 0 otherwise. Thus A is a symmetric matrix. Associated
with A is the degree distribution D, a diagonal matrix with row-sums of A along

179
180 Problems and Solutions

the diagonal, and 0s elsewhere. D describes how many connections each node
has. We define the Laplacian as L := D A. Let A = (aij ), i.e. aij are the
entries of adjacency matrix. Find the mimimum of the weighted sum
n
1 X
S= (xi xj )2 aij
2 i,j=1

with the constraint xT x = 1, where xT = (x1 , x2 , . . . , xn ). Use the Lagrange


multiplier method. The sum is over all pairs of squared distances between nodes
which are connected, and so the solution should result in nodes with large num-
bers of inter-connections being clustered together.

Problem 3. Find the eigenvalues of the three adjacent matrices



1 0 1 1 0 1 1 1 0
A = 1 1 1, B = 0 1 0, C = 0 1 0
1 0 1 1 0 1 0 1 1

provided by three simple graphs. Find the energy E(G) of each graph defined
by
X3
E(G) = |j |.
j=1

Problem 4. Let G be a graph with vertices VG = {1, 2, . . . , nG } and edges


EG VG VG . The nG nG adjacency matrix AG is given by (AG )ij =
EG (i, j) {0, 1} where i, j V . In the following products of graphs G1 and G2
with nG1 nG2 nG2 nG2 adjacency matrices. The rows and columns are indexed
by VG1 VG2 , where we use the ordering (i, j) (k, l) if i < k or, i = k and
j l.
The cartesian product G1 G2 of two graphs G1 and G2 is the graph with
vertices
VG1 G2 = VG1 VG2
and edges

EG1 G2 = { ((a, b), (a, b0 )) : a VG1 and (b, b0 ) EG2 }


{ ((a, b), (a0 , b)) : (a, a0 ) EG1 and b VG2 } .

It follows that

(AG1 G2 )(i,j),(k,l) = ij (AG2 )(k,l)  kl (AG1 )(i,j)

where  is the usual addition with the convention that 11 = 1. Thus AG1 G2 =
(AG1 InG2 )  (InG2 AG2 ).
Graphs and Matrices 181

The lexicographic product G1 G2 of two graphs G1 and G2 is the graph with


vertices
VG1 G2 = VG1 VG2
and edges

EG1 G2 = { ((a, b), (a, b0 )) : a VG1 and (b, b0 ) EG2 }


{ ((a, b), (a0 , b0 )) : (a, a0 ) EG1 and b, b0 VG2 } .

Thus EG1 G2 EG1 G2 . It follows that

(AG1 G2 )(i,j),(k,l) = ij (AG2 )(k,l)  (AG1 )(i,j) .

Thus AG1 G2 = (AG1 1nG2 )  (InG2 AG2 ). Here 1nG1 is the nG2 nG2 with
every entry equal to 1.
The tensor product G1 G2 of two graphs G1 and G2 is the graph with vertices

VG1 G2 = VG1 VG2

and edges

EG1 G2 = { ((a, b), (a0 , b0 )) : (a, a0 ) EG1 and (b, b0 ) EG2 } .

It follows that

(AG1 G2 )(i,j),(k,l) = (AG1 )(i,j) (AG2 )(k,l) = (AG1 AG2 )(i1)nG2 +k,(j1)nG2 +l .

Thus AG1 G2 = AG1 AG2 where is the Kronecker product of matrices.


The normal product G1 ? G2 of two graphs G1 and G2 is the graph with vertices

VG1 ?G2 = VG1 VG2

and edges

EG1 ?G2 = EG1 G2 EG1 G2 .

Thus

AG1 ?G2 = AG1 G2  AG1 G2 = (AG1 InG2 )  (InG2 AG2 )  AG1 AG2 .

(i) Show that the Euler path is not preserved (in general) under these operations.
(ii) Show that the Hamilton path is not preserved (in general) under these op-
erations.
Chapter 12

Hadamard Product

Suppose A = (aij ) and B = (bij ) are two n m matrices with entries in some
fields. Then the Hadamard product is the entrywise product of A and B, that
is, the m n matrix A B whose (i, j) entry is aij bij . We have the properties.
Suppose A, B, C are matrices of the same size and is a scalar. Then

AB=B A
A (B + C) = A B + A C
A (B) = (A B).

If A, B be n n diagonal matrices, then A B = AB. If A, B are n n positive


definite matrices and (ajj ) are the diagonal entries of A, then

n
Y
det(A B) det B ajj (1)
j=1

with equality if and only if A is a diagonal matrix.

Problem 1. Let
   
5 1 13 4
A= , B= .
1 1 4 4

First show that A and B are positive definite and then calculate the left and
right-hand side of (1).

182
Hadamard Product 183

Problem 2. Consider the matrices


1 0 0 1 1 0 0 0

0 0 0 0 0 0 1 0
A= , B= .
0 0 0 0 0 1 0 0
1 0 0 1 0 0 0 1
Calculate the Hadamard product A B. Show that
kA Bk kA Ak kB Bk
where the norm is given by the Hilbert-Schmidt norm.

Problem 3. Let A, B, C and DT be n n matrices over R. The Hadamard


product is defined by (A B)ij := aij bij . Show that
tr((A B)(C T D) = tr((A B C)D).

Problem 4. If V and W are matrices of the same order, then their Schur
product V W is defined by (entrywise multiplication)
(V W )j,k := Vj,k Wj,k .
If all entries of V are nonzero, then we say that X is Schur invertible and define
its Schur inverse, V () , by V () V = J, where J is the matrix with all 1s.

The vector space Mn (F) of n n matrices acts on itself in three distinct ways:
if C Mn (F) we can define endomorphisms XC , C and YC by
XC M := CM, C M := C M, YC := M C T .
Let A, B be n n matrices. Assume that XA is invertible and B is invertible
in the sense of Schur. Note that XA is invertible if and only if A is, and B is
invertible if and only if the Schur inverse B () is defined. We say that (A, B) is
a one-sided Jones pair if
XA B XA = B XA B .
We call this the braid relation. Give an example for a one-sided Jones pair.

Problem 5. Let A, B be n n matrices. Let e1 , . . . , en be the standard basis


vectors in Cn . We form the n2 column vectors
(Aej ) (Bek ), j, k = 1, . . . , n.
If A is invertible and B is Schur invertible, then for any j
{ (Ae1 ) (Bej ), (Ae2 ) (Bej ), . . . (Aen ) (Bej ) }
184 Problems and Solutions

is a basis of the vector space Cn . Let



1 1 1 1 1 1
A = 0 1 1, B = 1 1 1 .
0 0 1 1 1 1
Find these bases for these matrices.

Problem 6. Let U be an n n unitary matrix. Can we conclude that U U


is a unitary matrix?

Problem 7. Let B = (bjk ) be a diagonalizable n n matrix with eigenvalues


1 , 2 , . . . , n . Thus, there is nonsingular n n matrix A such that

B = A(diag(1 , 2 , . . . , n ))A1 .

Show that
b11 1

b22
. = (A (A1 )T ) .2

.. ..
bnn n
where is the Hadamard product (Schur product, entrywise product). Thus the
vector of eigenvalues of B is transformed to the vector of its diagonal entries by
the ceofficient matrix A (A1 )T .

Problem 8. Let A, B, C, D be n n matrices over R. Let

sT = ( 1 1 ... 1)

be a row vector in Rn . Show that

sT (A B)(C T D)s = tr(CD)


Pn
where = (ij ) is a diagonal matrix with jj = i=1 aij bij with j = 1, 2, . . . , n.

Problem 9. Given two matrices A and B of the same size. We use A B to


denote the Schur product. If all entries of A are nonzero, then we say that A is
Schur invertible and define its Schur inverse, A() by
() 1
Aij := .
Aij

Equivalently, we have A() A = J, where J is the matrix with all ones. An


n n matrix W is a type-II matrix if
T
W W () = nIn
Hadamard Product 185

where In is the n n identity matrix. Find such a matrix for n = 2.

Problem 10. Let A be an invertible n n matrix. Can we conclude that

A A1

is invertible?

Problem 11. The (n + 1) (n + 1) Hadamard matrix H(n) of any dimension


is generated recursively as follows
 
H(n 1) H(n 1)
H(n) =
H(n 1) H(n 1)

where n = 1, 2, . . . and H(0) = (1). Find H(1), H(2) and H(3).

Problem 12. Show that

tr(A(B C)) (vec(AT B))T vec(C).

Problem 13. Let be the Hadamard product. Let A be a positive semidefinite


n n matrix. Let B be an n n matrix with kBk 1, where k . k denotes the
spectral norm. Show that

max{ kA Bk : kBk 1 } = max ajj

where k k denotes the spectral norm.

Problem 14. Let A, B be n n matrices over C. Show that

rank(A B) (rank(A))(rank(B)).

Problem 15. Show that the Hadamard product is linear.

Problem 16. Let A, B be m n matrices and D and E be m m and n n


diagonal matrices, respectively. Show that

D(A B)E = A (DBE).

Problem 17. Let A, B, C, D be 2 2 matrices. Let be the Hadamard


product. Is
(A B) (C D) = (A C) (B D) ?
186 Problems and Solutions

Problem 18. Let A, B be n n positive semidefinite matrices. Show that


A B is also positive semidefinite.
Chapter 13

Unitary Matrices

An nn matrix U over C is called a unitary matrix if U = U 1 . Thus U U = In


for a unitary matrix. If U and V are unitary n n matrices, then U V is an
n n unitary matrix. If U is an n n unitary matrix and V is an m m unitary
matrix, then U V is a unitary matrix, where denotes the Kronecker product.
The columns in a unitary matrix are pairwise orthonormal. The n n unitary
matrices form a group under matrix multiplication. Let K be a skew-hermitian
matrix, then exp(K) is a unitary matrix. For any unitary matrix U we can find
a skew-hermitian matrix such that U = exp(K). The eigenvalues of a unitary
matrix are of the form ei with R. If U is an n n unitary matrix and V
is an m m unitary matrix, then U V is a unitary matrix, where denotes
the direct sum product. If U is a 2 2 unitary matrix and V is an 2 2 unitary
matrix, then U ? V is a unitary matrix, where ? denotes the star operation.

187
188 Problems and Solutions

Problem 1. (i) Let A, B be n n matrices over R. Show that one can find a
2n 2n unitary matrix U such that
   
A B A + iB 0n
U U = .
B A 0n A iB

Here 0n denotes the n n zero matrix.


(ii) Use the result from (i) to show that
 
A B
det = det(A + iB)det(A + iB) 0.
B A

Problem 2. Let u be a column vector in Cn with u u = 1, i.e. the vector is


normalized. Consider the matrix

U = In 2uu .

(i) Show that U is hermitian.


(ii) Show that U is unitary.

Problem 3. Can one find a 2 2 unitary matrix such that


   
1 0 1 0 1
U U = .
0 1 1 0

Problem 4. Let 1 , 2 , 3 be the Pauli spin matrices and


 
1 1 1
UH =
2 1 1

be the Hadamard matrix. Find



UH 1 UH , UH 2 UH , UH 3 UH .

Problem 5. Find all 2 2 hermitian and unitary matrices A, B such that

AB = ei BA.

Problem 6. Find all (n + 1) (n + 1) matrices A such that

A U A = U
Unitary Matrices 189

where U is the unitary matrix



0 0 i
U = 0 In1 0
i 0 0

and det(A) = 1. Consider first the case n = 2.

Problem 7. Consider the 2 2 hermitian matrices A and B with A 6= B with


the eigenvalues 1 , 2 ; 1 , 2 ; and the corresponding normalized eigenvectors
u1 , u2 ; v1 , v2 , respectively. Form from the normalized eigenvectors the 2 2
matrix 
u1 v1 u1 v2

.
u2 v1 u2 v2
Is this matrix unitary? Find the eigenvalues of this matrix and the correspond-
ing normalized eigenvectors of the 2 2 matrix. How are the eigenvalues and
eigenvectors are linked to the eigenvalues and eigenvectors of A and B?

Problem 8. Let 1 , 2 , 3 be the Pauli spin matrices. Let aj R with


j = 0, 1, 2, 3 and
a20 + a21 + a22 + a23 = 1.
Show that
U = ei (a0 I2 + a1 i1 + a2 i2 + a3 i3 )
is a unitary matrix, where R.

Problem 9. Let In be the n n unit matrix. Is the 2n 2n matrix


 
1 In iIn
=
2 In iIn
unitary?

Problem 10. Consider the two 2 2 unitary matrices


   
1 0 0 1
U1 = , U2 = .
0 1 1 0

Can one find a unitary 2 2 matrix V such that

U1 = V U2 V ?

Problem 11. Let U be an n n unitary matrix.


(i) Is U + U invertible?
190 Problems and Solutions

(ii) Is U + U hermitian?
(iii) Calculate exp((U + U )), where  R

Problem 12. Let U be an n n unitary matrix. Then U + U is a hermitian


matrix. Can any hermitian matrix represented in this form?

Problem 13. (i) Find the condition on the n n matrix A over C such that
In + A is a unitary matrix.
(ii) Let B be an 2 2 matrix over C. Find all solutions of the equation
B + B + BB = 02 .

Problem 14. Find all 2 2 invertible matrices A such that


A + A1 = I2 .

Problem 15. Let z1 , z2 , w1 , w2 C. Consider the 2 2 matrices


   
0 z1 0 w1
U= , V =
z2 0 w2 0
where z1 z1 = 1, z2 z2 = 1, w1 w1 = 1, w2 w2 = 1. This means the matrices U ,
V are unitary. Find the condition on z1 , z2 , w1 , w2 such that the commutator
[U, V ] is again a unitary matrix.

Problem 16. Let 1 , 2 , 3 be the Pauli spin matrices. Let 1 , 2 , 3 C.


Find the conditions on 1 , 2 , 3 such that
U = 1 1 + 2 2 + 3 3
is a unitary matrix.

Problem 17. Consider n n unitary matrices. A scalar product of two n n


matrices U , V can be defined as
1
hU, V i := tr(U V ).
n
Find two 2 2 unitary matrices U , V such that
1
hU, V i = .
2

Problem 18. Let


{ |a0 i, |a1 i, . . . , |an1 i }
Unitary Matrices 191

be an orthonormal basis in the Hilbert space Cn . The discrete Fourier transform


n1
1 X jk
|bj i = |ak i, j = 0, 1, . . . , n
n
k=0

where := exp(2i/n) is the primitive n-th root of unity.


(i) Apply the discrete Fourier transform to the standard basis in C4
1 0 0 0

0 1 0 0
, , , .
0 0 1 0
0 0 0 1
(ii) Apply the discrete Fourier transform to the Bell basis in C4
1 1 0 0

1 0 1 0 1 1 1 1
, , , .
2 0 2 0 2 1 2 1
1 1 0 0

Problem 19. (i) Consider the Pauli spin matrices 0 = I2 , 1 , 2 , 3 . The


matrices are unitary and hermitian. Is the 4 4 matrix
 
1 0 1

2 2 3
unitary?
(ii) Is the 4 4 matrix  
1 0 1

2 i2 3
unitary?

Problem 20. Let U be an 2 2 unitary matrix. Is the 4 4 matrix


   i 
0 0 e 0
V = U + I2
0 1 0 0
unitary?

Problem 21. The Pauli spin matrices


     
0 1 0 i 1 0
1 = , 2 = , 3 =
1 0 i 0 0 1
are hermitian and unitary. Together with the 2 2 identity matrix 0 = I2 they
form an orthogonal basis in Hilbert space of the 2 2 matrices over C with the
192 Problems and Solutions

scalar product tr(AB ). Let X be an n n hermitian matrix. Then (X + iIn )1


exists and
U = (X iIn )(X + iIn )1
is unitary. This is the so-called Cayley transform of X. Find the Cayley trans-
form of the Pauli spin matrices and the 2 2 identiy matrix. Show that these
matrices also form an orthogonal basis in the Hilbert space.

Problem 22. Consider the unitary matrix with determinant +1


 
cosh(r) ei sinh(r)
U (r, ) =
ei sinh(r) cosh(r)
where r, R. Find the eigenvalues and normalized eigenvectors. Construct
another unitary matrix using these normalized eigenvectors as columns of this
matrix.

Problem 23. Show that the two matrices


 i   
e 0 cos sin
A= , B=
0 ei sin cos

are conjugate in the Lie group SU (2).

Problem 24. A wave-scattering problem can be described by its scattering


matrix U . In a stationary problem, U relates the outgoing-wave to the ingoing-
wave amplitudes. The condition of flux conservation implies unitary of U , i.e.

UU = I

where I is the identity operator. If, additionally, the scattering problem is


invariant under the operatiom of time reversal, we also have U = U T , i.e. U is
symmetric. Find all 2 2 unitary matrices that also satisfy U = U T . Do these
matrices form a subgroup of the Lie group U (2)?

Problem 25. Let U be an n n unitary matrix. Let V be an n n unitary


matrix such that V 1 U V = D is a diagonal matrix D. Is V 1 U V a diagonal
matrix?

Problem 26. Let U be an n n unitary matrix. Is U + U invertible?

Problem 27. Let U , V be two n n unitary matrices. Then we can define a


scalar product via
1
hU, V i := tr(U V ).
n
Find 2 2 unitary matrices U , V such that hU, V i = 1/2.
Unitary Matrices 193

Problem 28. Let := exp(2i/4). Consider the 3 3 unitary matrices



0 0 0 0 1 i/2 (1 + i)/2 1/2
=0 2 0 , C = 0 1 0 , = (1 + i)/2 0 (1 i)/2 .
0 0 3 1 0 0 1/2 (1 i)/2 i/2
Do the matrices of the set
:= { j C k ` : 0 j 3, 0 k 1, 0 ` 2 }
form a group under matrix multiplication?

Problem 29. Let := exp(2i/4). Consider the 4 4 unitary matrices


1 0 0 0 0 0 0 1

0 0 0 1 0 0 0
= , = .

0 0 2 0 0 1 0 0
0 0 0 3 0 0 1 0
Let c > 0. The four-state Potts quantum chain is defined by the Hamilton
operator
N
1 X
(j + j2 + j3 ) + c(j 3j+1 + 2j 2j+1 + 3j j+1 )

H =
c j=1

where N is the number of sites and one imposes cyclic boundary conditions
N + 1 1. Let N = 2. Find the eigenvalues and eigenvectors of H.

Problem 30. Let U be an n n unitary matrix and A an arbitrary n n


matrix. Then we know that
1
U eA U 1 = eU AU .
Calculate U eA U with U 6= U 1 .

Problem 31. Consider the Bell matrix


1 0 0 1

1 0 1 1 0
B=
2 0 1 1 0

1 0 0 1
which is a unitary matrix. Each column vector of the matrix is a fully entangled
state. Are the normalized eigenvectors of B are also fully entangled states?

Problem 32. Consider the unitary matrix


 
1 ei11 ei12
U (11 , 12 , 21 , 22 ) = .
2 ei21 ei22
194 Problems and Solutions

Calculate the proudct U (11 , 12 , 21 , 22 )U (11 , 12 , 21 , 22 ) and find the con-


ditions on 11 , 12 , 21 , 22 and 11 , 12 , 21 , 22 such that we have again a
matrix of this form.

Problem 33. Consider the Hamilton operator H = H0 + H1 , where

H0 = ~3 , H1 = ~1 .

Let U and U0 be the unitary matrices

U = exp(iHt/~), U0 = exp(iH0 t/~).

Let n be a positive integer. The Moller wave operators

:= lim U n U0n .
n

Owing to their intertwining property the Moller wave operators transform the
eigenvectors of the free dynamics U0 = exp(iH0 t/~) into eigenvectors of the
interacting dynamics U = exp(iHt/~). Find .

Problem 34. Consider the unitary matrices


i
e 1 0 0 0 0 ei4
U1 (1 , 2 , 3 ) = 0 ei2 0 , U2 (4 , 5 , 6 ) = 0 ei5 0 .
0 0 ei3 ei6 0 0

What is the condition on 1 , . . . , 6 such that [U1 , U2 ] = 03 ?

Problem 35. Consider the matrices


i
e cos ei sin 0 0 0 1/2
U = ei sin ei 0 , N = 0 0 1/ 2 .
0 0 ei 1 1 0

Find the commutator [U, N ].

Problem 36. Let n 2 and even. Let U be a unitary antisymmetric n n


matrix. Show that there exists a unitary matrix V such that
   
T 0 1 0 1
V UV =
1 0 1 0

where denotes the direct sum.

Problem 37. Let U be a unitary and symmetric matrix. Show that there
exists a unitary and symmetric matrix V such that U = V 2 .
Unitary Matrices 195

Problem 38. Is the matrix



1 1 1
1
U= 1 exp(i2/3) exp(i4/3)
3 1 exp(i4/3) exp(i2/3)

unitary? Find the eigenvalues and eigenvectors of U .



Problem 39. (i) Let = ( 5 1)/2 be the golden mean number. Consider
the 2 2 matrices
 i7/10 
ei/10
 
e 0 i
B1 = , B2 = .
0 ei3/10 i ei/10

The matrices are invertible. Are the matrices unitary? Is B1 B2 B1 = B2 B1 B2 ?


(ii) Show that using computer algebra
 
0 i
B22 B14 B21 B1 B21 B1 B2 B12 B2 B11 B25 B1 B21 .
i 0

Problem 40. Let U be an n n unitary matrix. Show that | det(U )| = 1.

Problem 41. Let A be an nn matrix over R. Show that if is an eigenvalue


of A, then is also an eigenvalue of A. Give an example for a 2 2 matrix,
where the eigenvalues are complex.

Problem 42. Let V be an n n normal matrix over C. Assume that all its
eigenvalues have absolute value of 1, i.e. they are of the form ei . Show that V
is unitary.

Problem 43. (i) What are the conditions on 11 , 12 , 21 , 22 R such that


 i 
1 e 11 ei12
U (11 , 12 , 21 , 22 ) = i i
2 e 21 e 22
is a unitary matrix?
(ii) What are the condition on 11 , 12 , 21 , 22 R such that U (11 , 12 , 21 , 22 )
is an element of SU (2)?

Problem 44. Let , R. Are the 4 4 matrices


i
e cosh 0 0 sinh

i
0 e cosh sinh 0
U =

0 sinh ei cosh 0

i
sinh 0 0 e cosh
196 Problems and Solutions

0 ei cosh ei sinh 0

i i
e cosh 0 0 e sinh
V = i ,
e sinh 0 0 ei cosh
0 ei sinh ei cosh 0
unitary?

Problem 45. Is the matrix



1/ 2 0 1/ 2
U = i/ 2 0 i/ 2
0 1 0

unitary?

Problem 46. (i) The electronic scattering matrix has the form

S(1 , 2 , 3 , ) = ei1 0 ei2 3 ei2 ei3 3

where 1 , 2 , 3 [0, 2), [0, /2). Find S(1 , 2 , 3 , ).


(ii) Find
T (1 , 2 , 3 , ) = ei1 0 ei2 3 ei2 ei3 3 .

Problem 47. Let In be the n n idenity matrix. Is the 2n 2n matrix


 
1 In In
V =
2 iIn iIn
unitary? Find the eigenvalues of V .

Problem 48. Consider the unitary matrix


1 0 0 1

1 1 0 1 1 0
U = (I2 I2 + i1 2 ) = .
2 2 0 1 1 0
1 0 0 1
Calculate

U (1 I2 )U 1 , U (2 I2 )U 1 , U (3 I2 )U 1 ,

U (1 1 )U 1 , U (2 2 )U 1 , U (3 3 )U 1 .
Discuss.

Problem 49. Let , , R. Show that any U SU (2) can be written as

U = exp(i3 ) exp(i1 ) exp(i3 ).


Unitary Matrices 197

Problem 50. Let  


0 1
Y = i2 .
1 0
Find all 2 2 matrices S such that

Y SY 1 = S T , S = S.

Problem 51. Can one find a 2 2 unitary matrix U such that


   
cos ei sin 1 0
= U U ?
ei sin cos 0 1

Problem 52. (i) Consider an n n unitary matrix U = (ujk ) with j, k =


1, 2, . . . , n. Show that
S = (sjk ) = (ujk ujk )
is a double stochastic matrix.
(ii) Given a double stochastic n n matrix S. Can we construct the unitary
matrix U which generates the double stochastic matrix as described in (i).

Problem 53. Consider the unitary matrices


 i   
e 0 0 ei
U1 () = , U 2 () = .
0 ei ei 0

What is the condition on such that the commutator of U1 () and U2 () van-


ishes, i.e. [U1 , U2 ] = 02 .

Problem 54. We define the 2 2 matrix


 
0 cos() i sin()
:= cos()1 + sin()2 = .
cos() + i sin() 0

Calculate
exp(i /2).

Problem 55. Let U be a unitary matrix with U = U T . Show that U can be


written as U = V T V , where V is unitary.

Problem 56. Let


1 1
:= (1 5), := (1 + 5)
2 2
198 Problems and Solutions

with the golden mean number. Are the 4 4 matrices


1 0 1 0

1 1 0 1 1 0
U1 = , U2 =
2 0 1 2 0 1

0 1 0 1
unitary? Prove or disprove.

Problem 57. Is the matrix


 
cos() sin()ei
U (, ) =
sin()ei cos()

unitary? If so find the inverse.

Problem 58. Find the square roots of the Pauli spin matrices
       
1 0 0 1 0 i 1 0
0 = , 1 = , 2 = , 3 = .
0 1 1 0 i 0 0 1

Problem 59. Consider the unit vectors in C3



z1 w1
z = z2 , w = w2
z3 w3
i.e.
|z1 |2 + |z2 |2 + |z3 |2 = 1, |w1 |2 + |w2 |2 + |w3 |2 = 1.
Assume that (complex unit cone)

z1 w1 + z2 w2 + z3 w3 = 0.

Show that U SU (3) can be written as



z1 z2 z3
U = w1 w2 w3
u1 u2 u3

where
3
X
uj := jk` zk w` .
k,`=1

Problem 60. Let U be a unitary matrix. Then the determinant must be of


the form ei . Find the determinant of U + U .
Unitary Matrices 199

Problem 61. (i) Let 0 rjk 1 and jk R (j, k = 1, 2). Find the conditions
on rjk , jk such that the 2 2 matrix
 
r11 ei11 r12 ei12
U (rjk , jk ) =
r21 ei21 r22 ei22

is unitary. Then simplify to the special case jk = 0 for j, k = 1, 2.


(ii) Let 1 rjk 0, jk R (j, k = 1, 2, 3). Find the conditions on rjk and jk
such that
r11 ei11 r12 ei12 r13 ei13
r21 ei21 r22 ei22 r23 ei23
r31 ei31 r32 ei32 r33 ei33
is a unitary matrix.

Problem 62. (i) Let 1 , 2 R. Show that

e 1 ei2
 i 
1
U (1 , 2 ) = i i
2 e 2 e 1

is unitary. Is U (1 , 2 ) an element of SU (2)? Find the eigenvalues and eigen-


vectors of U (1 , 2 ). Do the eigenvalues and eigenvectors depend on the s?
Show that
e 1 ei2
 i 
1
V (1 , 2 ) = i i
2 e 2 e 1
is unitary. Is V (1 , 2 ) an element of SU (2)? Find the eigenvalues and eigen-
vectors of V (1 , 2 ). Do the eigenvalues and eigenvectors depend on the s?
(ii) Let 1 , 2 , 3 , 4 R. Show that

ei1 0 0 ei2

1 0 i3 i4
e e 0
U (1 , 2 , 3 , 4 ) =
0 ei4 ei3 0

2
i1
ei2 0 0 e

is unitary. Is U (1 , 2 , 3 , 4 ) an element of SU (4)? Find the eigenvalues and


eigenvectors of U (1 , 2 , 3 , 4 ). Do the eigenvalues and eigenvectors depend on
the s? Let 1 , 2 , 3 , 4 R. Show that

ei1 0 0 ei2

1 0 i3 i4
e e 0
V (1 , 2 , 3 , 4 ) =
0 ei4 ei3 0

2
i1
ei2 0 0 e

is unitary. Is V (1 , 2 , 3 , 4 ) an element of SU (4)? Find the eigenvalues and


eigenvectors of V (1 , 2 , 3 , 4 ). Do the eigenvalues and eigenvectors depend on
the s?
200 Problems and Solutions

(iii) Let 1 , 2 R. Show that

ei2
i
e 1 0
1
U (1 , 2 ) = 0 2 0
2 ei2 0 e i1

is unitary. Is U (1 , 2 ) an element of SU (2)? Find the eigenvalues and eigen-


vectors of U (1 , 2 ). Do the eigenvalues and eigenvectors depend on the s?
Show that
ei2
i
e 1 0
1
V (1 , 2 ) = 0 2 0
2 ei2 0 ei1
is unitary. Is V (1 , 2 ) an element of SU (2)? Find the eigenvalues and eigen-
vectors of V (1 , 2 ). Do the eigenvalues and eigenvectors depend on the s?

Problem 63. Show that the matrix



1/2 0 1/ 2
U = 1/ 2 0 1/ 2
0 1 0

is unitary. Find the eigenvalues and normalized eigenvectors of U . Write down


the spectral decomposition of U . Let

1
1
|i = 1 .
3 1

Find U |i, U 2 |i, U 3 |i. Study U n |i with n .

Problem 64. Find a unitary matrix U with det(U ) = 1 such that



0 1
1
U 1 = 0.
0 2 1

Problem 65. (i) Let 1 , 2 , 3 be the Pauli spin matrices. Let j R for
j = 1, 2, 3. Find the 2 2 matrices

Uk := (I2 ik k )(I2 + ik k )1

where k = 1, 2, 3. Are the matrices unitary?


(ii) Find the 4 4 matrices

Vk := (I4 ik k k )(I4 + ik k k )1
Unitary Matrices 201

where k = 1, 2, 3. Are the matrices unitary?

Problem 66. Find all 3 3 unitary matrices of the form



u11 u12 0
U = u21 u22 u23 .
0 u32 u33

Problem 67. Can one find a unitary matrix U with det(U ) = 1, i.e. U is an
element of SU (2), such that
   
1 0
=U .
0 1

Problem 68. What is the condition on the phases 1 , 2 , 3 such that


 
1 1 ei1
U (1 , 2 , 3 ) = i i
2 e 2 e 3
is a unitary matrix?

Problem 69. Can one find a 2 2 unitary matrix such that


   
1 0 1 0 1
U U = ?
1 0 0 1
Motivate. If so find the matrix U . Discuss.

Problem 70. Can one 2 2 find unitary matrices such that the rows and
columns add up to one. Of course the 2 2 identity matrix is one of them.

Problem 71. Let n 2. Consider the n n matrix (counting from (0, 0))
1
(Ajk ) = (ei(jk) )
n
where j, k = 0, 1, . . . , n 1. Is the matrix unitary? Study first the cases n = 2
and n = 3.

Problem 72. (i) Is the matrix


1 0 0 i

1 0 1 i 0
S=
2 0 i 1 0

i 0 0 1
202 Problems and Solutions

unitary?
(ii) Is the matrix
0 1 0 1

1 1 0 1 0
Z=
2 0 1 0 1

1 0 1 0
unitary?

Problem 73. Let n be a prime number and j, k = 1, . . . , n. Show that the


matrices
1
(U1 )jk = exp((2i/n)(j + k 1)2 )
n
..
.
1
(Ur )jk = exp((2i/n)r(j + k 1)2 )
n
..
.
1
(Un1 )jk = exp(((2i/n)(n 1)(j + k 1)2 )
n
1
(Un )jk = exp((2i/n)jk)
n
(Un+1 )jk = jk

are unitary.

Problem 74. Is the 2 2 matrix


 
u1 + iu2 u3 + iu4
U=
u3 + iu4 u1 iu2

with
u1 = cos(), u2 = sin() cos(),

u3 = sin() sin() cos(), u4 = sin() sin() sin()


unitary?

Problem 75. Find the eigenvalues of the unitary operator


 
U = exp i b b 3 .
4

Note that ei/4 = (1 + i)/ 2, ei/4 = (1 i)/ 2.
Unitary Matrices 203

Problem 76. Is the 3 3 matrix



1/3 1/3 1/
3
U = 1/6 1/ 6 2/ 3
1/ 2 1/ 2 0

a unitary matrix?

Problem 77. Are the 4 4 matrices


1 1 1 1 1 1 1 1

1 1 1 1 1 1 1 1 1 1
A= , B =
2 1 1 1 1 2 1 1 1 1

1 1 1 1 1 1 1 1

untary?

Problem 78. Show that any unitary n n matrix is conjugate to a diagonal


matrix of the form
diag( ei1 ei2 . . . ein ).

Problem 79. (i) What can be said about the eigenvalues of a matrix which
is unitary and hermitian?
(ii) What can be said about th eigenvalues of a matrix which is unitary and
skew-hermitian?
(iii) What can be said about the eigenvalues of a matrix which is unitary and
UT = U?
(iv) What can be said about the eigenvalues of a matrix which is unitary and
U3 = U?

Problem 80. Is the matrix


 
cos() sin()ei
U (, ) =
sin()ei cos()

unitary?

Problem 81. Consider the rotation matrix


 
cos() sin()
R() = .
sin() cos()

Find all 2 2 matrices


 
f11 () f12 ()
M () =
f21 () f22 ()
204 Problems and Solutions

such that R()M ()R1 () = M ( + ).

Problem 82. Are the matrices


   
1/2 3/2 1/2
3/2
V1 = , V2 =
3/2 1/2 3/2 1/2
unitary?

Problem 83. Find all n n matrices T over R such that

U = In + iT

is a unitary matrix.

Problem 84. Let U be an N N unitary matrix. Then


N
Y
det(U IN ) = (eij )
j=1

where j [0, 2). Let N = 2. Find for the matrix


 
0 i
U= .
i 0

Then calculate |ei1 ei2 |.

Problem 85. What are the conditions on 1 , 2 , 3 , 4 R such that the


matrix  i 
1 e 1 ei2
U (1 , 2 , 3 , 4 ) = i i
2 e 3 e 4
is unitary?

Problem 86. Let U be a unitary d d matrix. Then


Y
det(U Id ) = )j = 1d (eij ).

Use this expression to calculate the eigenvalues of the 2 2 matrix


 
0 i
U= .
i 0

Problem 87. Find a unitary 2 2 matrix such that

2 U U 3 = 02 .
Unitary Matrices 205

Problem 88. Find a unitary matrix U which can be written as a direct sum
of two 2 2 matrices and
1 1

   
1 1 1 1 1 1 1 0
U U = .
2 1 2 1 2 1 2 0
1 1

Problem 89. In the Hilbert space C4 the Bell states


1 0 0 1

1 0 1 1 1 1 1 0
, , ,
0 1 1 0

2 2 2 2
1 0 0 1
(i) Let = e2i/4 . Apply the Fourier transformation
1 1 1 1

1 1 2 3
UF =
2 1 2 1 2

3 2
1
to the Bell states and study the entanglement of these states.
(ii) Apply the Haar wavelet transformation
1 1 1 1

1 1 1 1 1
UH =
2 2 2 0 0


0 0 2 2
to the Bell states and study the entanglement of these states.
(iii) Apply the Walsh-Hadamard transformation
1 1 1 1

1 1 1 1 1
UW =
2 1 1 1 1

1 1 1 1
to the Bell states and study the entanglement of these states.
n
Extend to the Hilbert space C2 with the first Bell state given by
1 T
(1 0 0 1)
2

Problem 90. Is the matrix


1
U = (I2 I2 I2 + i1 1 1 + i3 3 3 )
3
206 Problems and Solutions

unitary?

Problem 91. Let A be an n n hermitian matrix with A2 = In .


(i) Is U = 12 (In + iA) unitary?
(ii) Is U = 12 (In iA) unitary?
(iii) Let [0, 2). What is the condition on such that
1
V () = (ei In + ei A)
2
is unitary?

Problem 92. Let U be a unitary 22 matrix. Can the matrix be reconstructed


from
tr(U ), tr(U 2 ), tr(U 3 )?

Problem 93. Is the 3 3 matrix



1 1 1
1
U= i i i
3 i i i
unitary?

Problem 94. Consider the Hadamard matrix


 
1 1 1
UH =
2 1 1
which is a unitary and hermitian matrix with eigenvalues +1 and 1. We
multiply each column with a phase factor eij and obtain the matrix
 i 
1 e 1 ei2
V (1 , 2 ) = i i .
2 e 1 e 2
Is the matrix still unitary? If not find the conditions on 1 and 2 such that
V (1 , 2 ) is unitary.

Problem 95. Let In be the nn identity matrix and Jn be the nn backward


identity matrix i.e. the entries are 1 at the counter diagonal and 0 otherwise.
Are the matrices
1 1 1
U1 = (In + iJn ), U2 = (In iJn ), U3 = ((1 + i)In + (1 i)Jn )
2 2 2
unitary?
Chapter 14

Numerical Methods

Problem 1. Let A be an invertible n n matrix over R. Consider the system


of linear equation Ax = b or
n
X
aij xj = bi , i = 1, 2, . . . , n.
j=1

Let A = C R. This is called a splitting of the matrix A and R is the defect


matrix of the splitting. Consider the iteration
Cx(k+1) = Rx(k) + b, k = 0, 1, 2, . . . .
Let
4 1 0 4 0 0
A = 1 4 1 , C = 0 4 0
0 2 4 0 0 4
and
3 0
b = 2, x(0) = 0.
2 0
The iteration converges if (C 1 R) < 1, where (C 1 R) denotes the spectral
radius of C 1 R. Show that (C 1 R) < 1. Perform the iteration.

Problem 2. Let A be an n n matrix over R and let b Rn . Consider the


linear equation Ax = b. Assume that ajj 6= 0 for j = 1, 2, . . . , n. We define
the diagonal matrix D = diag(ajj ). Then the linear equation Ax = b can be
written as
x = Bx + c

207
208 Problems and Solutions

with B := D1 (A D), c := D1 b. The Jacobi method for the solution of the


linear equation Ax = b is given by

x(k+1) = Bx(k) + c, k = 0, 1, . . .

where x(0) is any initial vector in Rn . The sequence converges if

(B) := max |j (B)| < 1


j=1,...,n

where (B) is the spectral radius of B. Let



2 1 0
A = 1 2 1.
0 1 2

(i) Show that the Jacobi method can applied for this matrix.
(ii) Find the solution of the linear equation with b = (1 1 1)T .

Problem 3. Let A be an n n matrix over R. The (p, q) Pade approximation


to exp(A) is defined by

Rpq (A) := (Dpq (A))1 Npq (A)

where
p
X (p + q j)!p!
Npq (A) = Aj
j=0
(p + q)!j!(p j)!

q
X (p + q j)!q!
Dpq (A) = (A)j .
j=0
(p + q)!j!(q j)!

Nonsingularity of Dpq (A) is assured if p and q are large enough or if the eigen-
values of A are negative. Find the Pade approximation for the matrix
 
0 1
A=
1 0

and p = q = 2. Compare with the exact solution.

Problem 4. Let A be an n n matrix over R. Then we have the Taylor


expansion

X (1)k X (1)k
sin(A) := A2k+1 , cos(A) := A2k .
(2k + 1)! (2k)!
k=0 k=0
Numerical Methods 209

To calculate sin(A) and cos(A) from a truncated Taylor series approximation


is only worthwhile near the origin. We can use the repeated application of the
double angle formula

cos(2A) 2 cos2 (A) In , sin(2A) 2 sin(A) cos(A).

We can find sin(A) and cos(A) of a matrix A from a suitably truncated Taylor
series approximates as follows

S0 = Taylor approximate to sin(A/2k )

C0 = Taylor approximate to cos(A/2k )


and the recursion
2
Sj = 2Sj1 Cj1 , Cj = 2Cj1 In

where j = 1, 2, . . .. Here k is a positive integer chosen so that, say kAk 2k .


Apply this recursion to calculate sine and cosine of the 2 2 matrix
 
2 1
A= .
1 2

Use k = 2.

Problem 5. Let A be an n n matrix. We define the j k approximant of


exp(A) by
k  ` !j
X 1 A
fj,k (A) := . (1)
`! j
`=0

We have the inequality


1
keA fj,k (A)k kAkk+1 ekAk (2)
j k (k + 1)!

and fj,k (A) converges to eA , i.e.

lim fj,k (A) = lim fj,k (A) = eA .


j k

Let  
0 1
A= .
0 0
Find f2,2 (A) and eA . Calculate the right-hand side of the inequality (2).

Problem 6. The power method is the simplest algorithm for computing eigen-
vectors and eigenvalues Consider the vector space Rn with the Euclidean norm
210 Problems and Solutions

kxk of a vector x R. The iteration is as follows: Given a nonsingular n n


matrix M and a vector x0 with kx0 k = 1. One defines
M xt
xt+1 = , t = 0, 1, . . .
kM xt k
This defines a dynamical system on the sphere S n1 . Since M is invertible we
have
M 1 xt+1
xt = , t = 0, 1, . . .
kM 1 xt+1 k
(i) Apply the power method to the nonnormal matrix
   
1 1 1
A= and x0 = .
0 1 0
(ii) Apply the power method to the Bell matrix
1 0 0 1 1

1 0 1 1 0 0
B= and x0 = .
2 0 1 1 0 0
1 0 0 1 0

Problem 7. Let A be an n n matrix over R and let u be an n-vector in


Rn (column vector) with u 6= u. In numerical linear algebra we often have to
compute
2uuT
 
In T (1)
u u
where In is the n n identity matrix. Naively we would form the matrix (In
2uuT /uT u) from the vector u and then form the matrix product explicitly with
A. This would require O(m3 ) flops. Provide a faster computation for expression
(1).

Problem 8. Consider an n n symmetric tridiagonal matrix over R. Let


fn () := det(A In ) and
1 1 0 0

1 2 2 0
..
0

2 . 0

fk () = det . .. .. .. ..
.
. . . . .

0 k1 k1

0 0 k1 k
for k = 1, 2, . . . , n and f0 () = 1, f1 () = 0. Then
2
fk () = ( )fk1 () k1 fk2 ()
Numerical Methods 211

for k = 2, 3, . . . , n. Find f4 () for the 4 4 matrix



0 1 0 0

1 0 2 0
.
0 2 0 3

0 0 3 0
Chapter 15

Binary Matrices

An m n matrix A is a binary matrix if ajk {0, 1} for j = 1, . . . , m and


k = 1, . . . , n.

Problem 1. For a 2 2 binary matrix


 
a11 a12
A= , ajk { 0, 1 }
a21 a22

we define the determinant as

det A = (a11 a22 ) (a12 a21 )

where is the AND-operation and is the XOR-operation.


(i) Find the determinant for the following 2 2 matrices
           
1 0 0 1 1 1 0 1 1 0 1 1
, , , , , .
0 1 1 0 0 1 1 1 1 1 1 0

(ii) Find the determinant for the following 2 2 matrices


         
0 0 1 0 0 1 1 0 0 0
, , , ,
0 0 0 0 0 0 0 0 0 1
         
1 1 1 0 0 0 0 1 1 1
, , , , .
0 0 1 0 1 1 0 1 1 1

212
Binary Matrices 213

Problem 2. The determinant of a 3 3 matrix



a11 a12 a13
A = a21 a22 a23
a31 a32 a33
is given by

det A = a11 a22 a33 + a12 a23 a31 + a13 a21 a32
a13 a22 a31 a11 a23 a32 a12 a21 a33 .

For a binary matrix B we replace this expression by

det B = (b11 b22 b33 ) (b12 b23 b31 ) (b13 b21 b32 )
(b13 b22 b31 ) (b11 b23 b32 ) (b12 b21 b33 ).

(i) Calculate the determinant for the binary matrices



1 0 0 1 1 1
0 1 0, 0 1 1.
0 0 1 0 0 1

(ii) Calculate the determinant for the binary matrices



1 1 0 1 0 0 1 0 1
1 1 0, 1 0 0, 0 1 0.
0 0 0 1 0 0 1 0 1

Problem 3. The finite field GF (2) consists of the elements 0 and 1 (bits) which
satisfies the following addition (XOR-operation) and multiplication (AND-operation)
tables
0 1 0 1
0 0 1 0 0 0
1 1 0 1 0 1

Find the determinant of the binary matrices



1 0 1 1 1 1
A = 0 1 0, B = 0 1 1.
1 0 1 0 0 1

Problem 4. A boolean function f : { 0, 1}n { 0, 1} can be transformed from


the domain {0, 1} into the spectral domain by a linear transformation

Ty = s
214 Problems and Solutions

where T is a 2n 2n orthogonal matrix, y = (y0 , y1 , . . . , y2n 1 )T , is the two


valued ({+1, 1} with 0 1, 1 1) truth table vector of the boolean function
and sj (j = 0, 1, . . . , 7) are the spectral coefficients (s = (s0 , s1 , . . . , s2n 1 )T ).
Since T is invertible we have
T 1 s = y.
For T we select the Hadamard matrix. The 2n 2n Hadamard matrix H(n) is
recursively defined as
 
H(n 1) H(n 1)
H(n) = , n = 1, 2, . . .
H(n 1) H(n 1)
with H(0) = (1) (1 1 matrix). The inverse of H(n) is given by
1
H 1 (n) =
H(n).
2n
Now any boolean function can be expanded as the arithmetical polynomial
1
2n s0 s1 (1)xn s2 (1)xn1 s2n 1 (1)x1 x2 xn

f (x1 , . . . , xn ) =
2n+1
where denotes the modulo-2 addition.

Consider the boolean function f : { 0, 1 }3 { 0, 1 } given by


f (x1 , x2 , x3 ) = x1 x2 x3 + x1 x2 x3 + x1 x2 x3 .
Find the truth table, the vector y and then calculate, using H(3), the spectral
coefficients sj , (j = 0, 1, . . . , 7).

Problem 5. Consider the binary matrices


   
0 1 1 1
A= , B= .
1 0 1 1
Calculate the Hadamard product A B.

Problem 6. Consider the two permutation matrices (NOT-gate and XOR-


gate)
0 0 0 1 1 0 0 0


0 0 1 0 0 1 0 0
N = , X = .
0 1 0 0 0 0 0 1
1 0 0 0 0 0 1 0
Can we generate all other permutation matrices from these two permutation
matrices?

Problem 7. How many 3 3 binary matrices can one form which contain
three 1s? Write down these matrices. Which of them are invertible?
Chapter 16

Groups

Problem 1. (i) Find the group generated by the matrix

1 0 0 0

0 0 i 0
A=
0 i 0 0

0 0 0 1

under matrix multiplication.


(ii) Find the group generated by the 6 6 matrix

02 02 I2
B = I2 02 02
02 I2 02

under matrix multiplication.

Problem 2. The Pauli matrix 1 is not only hermitian, unitary and his own
inverse, but also a permutation matrix. Find all 2 2 matrices A such that

11 A1 = A.

Problem 3. Let z C and z 6= 0.


(i) Do the 2 2 matrices
   
z 0 0 z
,
0 z 1 z 1 0

215
216 Problems and Solutions

form a group under matrix multiplication?


(ii) Do the 3 3 matrices

z 0 0 0 0 z
0 1 0 , 0 1 0
0 0 z 1 z 1 0 0

form a group under matrix multiplication?

Problem 4. Find all 3 3 permutation matrices P such that



1/ 2 0 1/ 2 1/ 2 0 1/ 2
P 1 0 1 0 P = 0 1 0 .
1/ 2 0 1/ 2 1/ 2 0 1/ 2

Show that these matrices form a group, i.e. a subgroup of the 3 3 permutation
matrices.

Problem 5. (i) Let c R and c 6= 0. Do the matrices

c c c c

c c c c
A(c) =
c c c c

c c c c

form a group under matrix multiplication?


(ii) Find the eigenvalues of A(c).

Problem 6. Do the matrices


   
1 0 0 ei
I2 = , U () = .
0 1 ei 0

form a group under matrix multiplication?

Problem 7. Consider the Pauli spin matrices


       
1 0 0 1 0 i 1 0
0 = , 1 = , 2 = , 3 = .
0 1 1 0 i 0 0 1

(i) Do the sixteen 4 4 matrices (j = 0, 1, 2, 3)


       
j 0 2 j 02 0 2 j 02 j
, , ,
02 j 02 j j 02 j 02

form a group under matrix multiplication?


Groups 217

(ii) Do the sixteen 4 4 matrices (j = 0, 1, 2, 3)


       
j 0 2 j 02 02 j 02 j
, , ,
02 j 02 j j 02 j 02
form a Lie algebra under the commutator?

Problem 8. For the vector space of the nn matrices over R we can introduce
a scalar product via
hA, Bi := tr(AB T ).
Consider the Lie group SL(2, R) of the 2 2 matrices with determinant 1. Find
X, Y SL(2, R) such that
hX, Y i = 0
where neither X nor Y can be 2 2 identity matrix.

Problem 9. Consider the diagonal matrix D and the permutation matrix P



1 0 0 0 1 0
D = 0 exp(2i/3) 0 , P = 0 0 1.
0 0 exp(2i/3) 1 0 0
(i) What group is generated by D and P ?
(ii) Calculate the commutator [D, P ]. Discuss.

Problem 10. Do the numbers 1, i, 1, i form a group under multiplication?


Do the vectors
1 i 1 i

1 i 1 1 1 i 1 1
, , ,
2 1 2 i 2 1 2 i

i 1 i 1
form an orthonormal basis in C4 ?

Problem 11. Let I2 , 02 be the 2 2 identity and zero matrix, respectively.


Find the group generated by the 4 4 matrix
 
02 I2
A= .
I2 0 2

Problem 12. Do the matrices


a11 0 0 a14

0 a22 a23 0
A=
0 a32 a33 0

a41 0 0 a44
218 Problems and Solutions

with det(A) 6= 0 form a group under matrix multiplication?

Problem 13. Consider the permutation group S3 . The matrix representation


of the permutations (12) and (13) are

0 1 0 0 0 1
(12) 7 1 0 0 , (13) 7 0 1 0.
0 0 1 1 0 0

Show that the other permutations of S3 can be constructed from products of


these two matrices.

Problem 14. Do the set of 2 2 matrices


 i(+) 
e cosh( ) ei() sinh( )
ei() sinh( ) ei(+) cosh( )

form a group under matrix multiplication, where , , R?

Problem 15. Let 0 < /4. Consider the transformation


1
X(x, y, ) = p (x cos() + iy sin())
cos(2)
1
Y (x, y, ) = p (ix sin() + y cos()).
cos(2)

(i) Show that X 2 + Y 2 = x2 + y 2 .


(ii) Do the matrices
 
1 cos() i sin()
cos(2) i sin() cos()
p

form a group under matrix multiplication?

Problem 16. Let , R and 6= 0. Do the matrices

1 cos()
 
cos()
M (, ) =
sin() cos()

form a group under matrix multiplication?

Problem 17. In the Lie group U (N ) of the N N unitary matrices one can
find two N N matrices U and V such that

U V = e2i/N V U.
Groups 219

Any N N hermitian matrix H can be written in the form


N
X 1 N
X 1
H= hjk U j V k .
j=0 k=0

Using the expansion coefficients hjk one can associate to the hermitian matrix
H the function
N
X 1 N
X 1
h(q, q) = hjk e2i(jp+kq)
j=0 k=1

where p = 0, 1, . . . , N 1 and q = 0, 1, . . . , N 1. Consider the case N = 2 and


   
1 0 0 1
U= , V = .
0 ei 1 0

(i) Consider the hermitian and unitary matrix


 
0 i
.
i 0

Find h(p, q).


(ii) Consider the hermitian and projection matrix
 
1 1 1
.
2 1 1
Find h(p, q).

Problem 18. Consider the 3 3 permutation matrix



0 1 0
P = 0 0 1.
1 0 0

(i) Find P 2 , P 3 . Do the matrices P , P 2 , P 3 form a group under matrix multi-


plication?
(ii) Find the eigenvalues and eigenvectors of P . Do the eigenvalues of P form a
group under multiplication?

Problem 19. Consider the two 3 3 permutation matrices



0 1 0 0 0 1
C1 = 0 0 1, A = 0 1 0.
1 0 0 1 0 0
Can the remaining four 3 3 matrices be generated from C1 and A using matrix
multiplication?
220 Problems and Solutions

Problem 20. Consider the two 4 4 permutation matrices

0 1 0 0 0 0 0 1

0 0 1 0 0 0 1 0
C1 = , A= .
0 0 0 1 0 1 0 0
1 0 0 0 1 0 0 0

Can the remaining twenty-two 4 4 matrices be generated from C1 and A using


matrix multiplication?

Problem 21. Can one find a 4 4 permutation matrix P such that



0 0 0
P0 0 PT = 0 .
0 0

Problem 22. (i) Consider the matrices


   
a11 a12 0 1
A= , 1 = .
a21 a22 1 0

Find the condition on A such that 1 A1 = A. Assume that A is also invertible.


Do these matrices from a group?
(ii) Find the condition on A such that

(1 1 )(A A)(1 1 ).

Problem 23. Can one find an such that


   
cos sin 0 1 cos sin
?
sin cos 1 0 sin cos

Problem 24. Consider the permutation matrix



0 1 0
C = 0 0 1.
1 0 0

Find the condition on a 3 3 matrix A such that

CAC T = A.

Note that C T = C 1 .
Groups 221

Problem 25. Consider the permutation matrix

0 1 0 0

0 0 1 0
C= .
0 0 0 1
1 0 0 0

Find the condition on a 4 4 matrix A such that

CAC T = A.

Note that C T = C 1 .

Problem 26. The Lie group 0(2) is generated by a rotation R1 and a reflection
R2    
cos sin cos sin
R1 = , R2 = .
sin cos sin cos
Find the trace, determinant and eigenvalues of R1 and R2 .

Problem 27. (i) Consider the group G of all 4 4 permutation matrices.


Show that
1 X
g
|G|
gG

is a projection matrix. Here |G| denotes the number of elements in the group.
(ii) Consider the subgroup given by the matrices

1 0 0 0 0 0 0 1

0 1 0 0 0 0 1 0
, .
0 0 1 0 0 1 0 0

0 0 0 1 1 0 0 0

Show that
1 X
g
|G|
gG

is a projection matrix.

Problem 28. Let A be the 2 2 matrix


 
a11 ei11 a12 ei12
A=
a21 ei21 a22 ei22

where ajk R, ajk > 0 for j, k = 1, 2 and a12 = a21 . We also have jk R for
j, k = 1, 2 and impose 12 = 21 . What are the conditions on ajk and jk such
that I2 + iA is a unitary matrix?
222 Problems and Solutions

Problem 29. Let , , R and , 6= 0. Consider the matrices


 
cos sin
A(, , ) = .
1 sin 1 cos

Do the matrices form a group under matrix multiplication?

Problem 30. Show that the four 2 2 matrices


       
1 0 1 0 0 1 0 1
A= , B= , C= , D=
0 1 0 1 1 0 1 0

from a group under matrix multiplication. Is the group abelian?

Problem 31. Let x R. Is the matrix


cos x 0 sin x 0

0 cos x 0 sin x
A(x) =
sin x 0 cos x 0

0 sin x 0 cos x

an orthogonal matrix?

Problem 32. (i) Let  


a11 a12
A= .
a12 a11
where a11 , a12 R. Find all invertible 2 2 matrices S over R such that

SAS 1 = A.

Obviously the identity matrix I2 would be such as matrix.


(ii) Do the matrices S form a group under matrix multiplication? Prove or
disprove.
(iii) Use the result form (i) to calculate

(S S)(A A)(S S)1 .

Discuss.

Problem 33. Let , , R. Do the 3 3 matrices



cos() sin()
M (, , ) = sin() cos()
0 0 1

form a group under matrix multiplication?


Groups 223

Problem 34. Is the invertible matrix


0 0 0 1

0 1 0 0
U =
0 0 1 0

1 0 0 0

an element of the Lie group SO(4)?

Problem 35. Do the eight 2 2 matrices


       
1 0 1 0 0 1 0 1
, , ,
0 1 0 1 1 0 1 0
       
1 1 1 1 1 1 1 1 1 1 1 1
, , ,
2 1 1 2 1 1 2 1 1 2 1 1
form a group under matrix multiplication? If not add the matrices so that one
has a group.

Problem 36. The Heisenberg group is the set of upper 3 3 matrices of the
form
1 a c
H = 0 1 b
0 0 1
where a, b, c can be taken from some (arbitrary) commutative ring.
(i) Find the inverse of H.
(ii) Given two elements x, y of a group G, we define the commutator of x and
y, denoted by [x, y] to be the element x1 y 1 xy. If a, b, c are integers (in the
ring Z of the integers) we obtain the discrete Heisenberg group H3 . It has two
generators
1 1 0 1 0 0
x = 0 1 0, y = 0 1 1.
0 0 1 0 0 1
Find
z = xyx1 y 1 .
Show that xz = zx and yz = zy.
(iii) The derived subgroup (or commutator subgroup) of a group G is the sub-
group [G, G] generated by the set of commutators of every pair of elements of
G. Find [G, G] for the Heisenberg group.
(iv) Let
0 a c
A = 0 0 b
0 0 0
224 Problems and Solutions

and a, b, c R. Find exp(A).


(v) The Heisenberg group is a simple connected Lie group whose Lie algebra
consists of matrices
0 a c
L = 0 0 b.
0 0 0
Find the commutators [L, L0 ] and [L, L0 ], L0 ]], where [L, L0 ] := LL0 L0 L.

Problem 37. Find all 2 2 matrices S over C with determinant 1 (i.e. they
are elements of SL(2, C)) such that
   
1 0 1 0 1
S S= .
1 0 1 0

Obviously, the 2 2 identity matrix is such an element.

Problem 38. There are six 3 3 permutation matrices which form a group
under matrix multiplication.
(i) Can the six elements be generated from the two permutation matrices

0 1 0 0 0 1
A = 0 0 1, B = 0 1 0.
1 0 0 1 0 0

using matrix multiplication?


(ii) Does A, A2 , A3 provide a subgroup?

Problem 39. There are twenty-four 4 4 permutation matrices which form a


group under matrix multiplication.
(i) Can the 24 elements be generated from the two permutation matrices

0 1 0 0 0 0 0 1

0 0 1 0 0 0 1 0
A= , B=
0 0 0 1 0 1 0 0

1 0 0 0 1 0 0 0
using matrix multiplication?
(ii) Does A, A2 , A3 , A4 provide a subgroup?

Problem 40. Let , , R. The Heisenberg group H3 (R) consists of all


3 3 upper trian matrices of the form

1
M (, , ) = 0 1
0 0 1
Groups 225

with matrix multiplication as composition. Let t R. Consider the matrices



1 t 0 1 0 0 1 0 t
A(t) = 0 1 0 , B(t) = 0 1 t , C(t) = 0 1 0.
0 0 1 0 0 1 0 0 1

(i) Show that {A(t) : t R}, {B(t) : t R}, {C(t) : t R} are one-parameter
subgroups in H3 (R).
(ii) Show that {C(t) : t R} is the center of H3 (R).

Problem 41. Consider the Pauli spin matrices 1 , 2 , 3 . Let x1 , x2 , x3 R.


Show that
sin(r)
ei(x1 1 +x2 2 +x3 3 ) = I2 cos(r) + i(x1 1 + x2 2 + x3 3 )
r
where r2 = x21 + x22 + x23 .

Problem 42. (i) Consider the 2 2 matrix


 
0 1
J := i2 = .
1 0

Show that any 2 2 A SL(2, C) satisfies

AT JA = J

where J denotes the transposed matrix of A.


(ii) Let A satisfying AT JA = J. Is

(A A)T (J J)(A A) = J J ?

Is
(A A)T (J J)(A A) = J J ?
Is
(A ? A)T (J ? J)(A ? A) = J ? J ?

Problem 43. The group SL(2, F3 ) consists of unimodular 2 2 matrices with


integer entries taken modulo three. Let
 
1 2
A=
0 1

be an element of SL(2, F3 ). Find the the inverse of A.


226 Problems and Solutions

Problem 44. (i) Do the 4 4 matrices


i
e cosh() 0 0 sinh()

0 ei cosh() sinh() 0
g(, ) =

0 sinh() ei cosh() 0

sinh() 0 0 ei cosh()
form a group?
(ii) Do the 4 4 matrices
i
e cos() 0 0 sin()

i
0 e cos() sin() 0
g(, ) =

0 sin() ei cos() 0

i
sin() 0 0 e cos()
form a group?

Problem 45. Consider the cyclic 3 3 matrix



0 1 0
C = 0 0 1.
1 0 0
(i) Show that the matrices C, C 2 , C 3 form a group under matrix multiplication.
Is the group commutative?
(ii) Find the eigenvalues of C and show that the form a group under multiplica-
tion.
(iii) Find the normalized eigenvalues of C. Show that the form a orthonormal
basis in C3 .
(iv) Use the eigenvalues and normalized eigenvectors to write down the spectral
decomposition of C.
(v) Use the result from (iv) to find K such C = exp(K).
(vi) Use these results to find L such that C 2 = exp(L).

Problem 46. (i) Let x R. Show that the matrix


cos(x) 0 sin(x) 0

0 cos(x) 0 sin(x)
A(x) =
sin(x) 0 cos(x) 0

0 sin(x) 0 cos(x)
is invertible. Find the inverse. Do these matrices form a group under matrix
multiplication?
(ii) Let x R. Show that the matrix
cosh(x) 0 sinh(x) 0

0 cosh(x) 0 sinh(x)
B(x) =

sinh(x) 0 cosh(x) 0

0 sinh(x) 0 cosh(x)
Groups 227

is invertible. Find the inverse. Do these matrices form a group under matrix
multiplication.

Problem 47. For the vector space of the n n matrices over R we can
introduce a scalar product via
hA, Bi := tr(AB T ).
Consider the Lie group SL(2, R) of the 2 2 matrices with determinant 1. Find
X, Y SL(2, R) such that
hX, Y i = 0
where neither X nor Y can be 2 2 identity matrix.

Problem 48. Which group is generated by



1 0 1
A = 0 1 0?
0 0 1
First find the inverse of A.

Problem 49. The free group 2 with two generators g1 and g2 has the matrix
representation    
1 0 1 2
g1 = , g2 = .
2 1 0 1
Obviously g1 and g2 are elements of SL(2, R). Find the inverse of g1 and g2 .
Calculate
g1 g21 g1 g2 g11 g2 .

Problem 50. Consider the 3 3 matrix



cos() 0 sin()
A() = 0 1 0 .
sin() 0 cos()
(i) Show that A() is an element of SO(3, R).
(ii) Find the eigenvalues and normalized eigenvectors of A().
(iii) Find the eigenvalues and normalized eigenvectors of A() A().

Problem 51. (i) Find the group (matrix multiplication) generated by the 44
matrix
0 0 0 1

0 1 0 0
A= .
0 0 1 0
1 0 0 0
228 Problems and Solutions

Is the group commutative?


(ii) Find the determinant of all these matrices from (i). Do these numbers form
a group under multiplication?
(iii) Find all the eigenvalues of these matrices. Do these numbers form a group
under multiplication?
(iv) Let R. Find exp(A). Is exp(A) an element of SL(4, R)?

Problem 52. Consider the two spin-1 matrices



0 0 i 0 i 0
L2 = 0 0 0 , L3 = i 0 0.
i 0 0 0 0 0

Let , R. Calculate

T (, ) = exp(iL3 ) exp(iL2 ).

Is T (, ) an elements of SO(3, R?

Problem 53. Let R. Consider the spin matrix


 
1 0 i
S2 = .
2 i 0

(i) Find
A() = exp(iS2 ).
(ii) Calculate
dA()
B= .
d =0
Then find
C() = exp(B).
Discuss.
(iii) Find
D() = exp(iS2 S2 ).
(iv) Calculate
dD()
E= .
d =0
Then find
G() = exp(E).
Discuss.
Groups 229

Problem 54. Let , R. Do the matrices


 
cosh() ei sinh()
ei sinh() cosh()

form a group under matrix multiplication? Are the matrices unitary?

Problem 55. Let x, y, z Z. Do the matrices



1 x z
0 1 y
0 0 1

form a group under matrix multiplication?

Problem 56. Find the group generated by the 3 3 matrices



1 0 0 0 0 1
G1 = 0 1 0 , G2 = 1 0 0.
0 0 1 0 1 0

Set G0 = G21 = I3 .

Problem 57. (i) Show that the matrix


 
0 1
U=
i 0
is unitary.
(ii) Find the eigenvalues and eigenvectors of U .
(iii) Find the group generated by U . Find the group generated by U U .

Problem 58. Consider the six 3 3 permutation matrices



1 0 0 1 0 0 0 1 0
0 1 0, 0 0 1, 0 0 1,
0 0 1 0 1 0 1 0 0

0 1 0 0 0 1 0 0 1
1 0 0, 0 1 0, 1 0 0.
0 0 1 1 0 0 0 1 0
Find two of these permutation matrices which generate all six permutation ma-
trices.

Problem 59. Consider the six 3 3 permutation matrices. Which two of the
matrices generate all the other ones.
230 Problems and Solutions

Problem 60. (i) Find all invertible 2 2 matrices over R such that
   
0 1 0 0
T T 1 = .
0 0 1 0

(ii) Do these matrices form a group?

Problem 61. (i) Show that the matrices


   
0 1 1 0
A= , B=
0 1 1 0

are similar. This means find an invertible 2 2 matrix S, i.e. S GL(2, R),
such that SAS 1 = B.
(ii) Is there an invertible 2 2 matrix S such that

(S S)(A A)(S 1 S 1 ) = B B.

Problem 62. Find the group generated by the two 2 2 matrices


   
1 0 1 1 3
3 = , R=
0 1 2 3 1

under matrix multiplication. Is the group commutative?

Problem 63. Find all M GL(2, C) such that


   
0 i 0 i
M 1 M= .
i 0 i 0

Thus we consider the invariance of the Pauli matrix 2 . Show that these matrices
form a group under matrix multiplication.

Problem 64. Find the group generated by the matrices


     
1 0 1/2 3/2 1/2 3/2
I2 = , R1 = , R2 =
0 1 3/2 1/2 3/2 1/2

under matrix multiplication.

Problem 65. Let 3 = 1. What group is generated by the matrices



1 0 0 0 0 0 0
A = 0 0 , B = 0 1 0 , C = 0 1 0
0 2 0 2 0 0 2 0 0
Groups 231

under matrix multiplication? Is the matrix


1
P = (A + B + C)
3
a projection matrix?

Problem 66. Find all 2 2 matrices A over R with det(A) = 1 such that
   
0 1 1 0 1
A A = .
0 0 0 0

Do these matrices form a group under matrix multiplication?

Problem 67. Find all 2 2 matrices A over C with det(A) = 1 such that
   
0 1 0 0
A A1 = .
0 0 1 0

Find the group generated by these matrices under matrix multiplication.

Problem 68. Find all 2 2 invertible matrices A with det(A) = 1 such that
   
0 i 1 0 i
A A = .
i 0 i 0

Do these matrices form a group under matrix multiplication?

Problem 69. (i) Find the group generated by the 3 3 permutation matrices

0 1 0 0 0 1
A = 0 0 1, B = 0 1 0
1 0 0 1 0 0

under matrix multiplication.


(ii) Find the group generated by the 4 4 permutation matrices

0 1 0 0 0 0 0 1

0 0 1 0 0 0 1 0
A= , B=
0 0 0 1 0 1 0 0

1 0 0 0 1 0 0 0

under matrix multiplication.


Chapter 17

Lie Groups

Problem 1. (i) Find all 2 2 matrices A over R such that


det(A) = a11 a22 a12 a21 = 1
(i.e. A is an element of the Lie group SL(2, R)) and
   
1 1 1 1
A = .
2 1 2 1
(ii) Do these matrices form a group under matrix multiplication?

Problem 2. The generators of the braid group B3 are given by


   
1 0 1 1
1 = , 2 = .
1 1 0 1
Thus 1 and 2 are elements of the Lie group SL(2, R).
(i) Find 11 and 21 . Find 1 2 and 11 2 .
(ii) Is 1 2 1 = 2 1 2 ?

Problem 3. Let R. Consider the hermitian matrix which is an element


of the noncompact Lie group SO(1, 1)
 
cosh() sinh()
A() = .
sinh() cosh()
Find the Cayley transform
B = (A iI2 )(A + iI2 )1 .

232
Lie Groups 233

Note that B is a unitary matrix and therefore an element of the compact Lie
group U (n). Find B( ).

Problem 4. If A SL(2, R), then it can be uniquely be written in the form


   
cos sin a b
exp .
sin cos b a
Find this decomposition for the matrix
 
1 1
A= .
0 1

Problem 5. The unit sphere


4
X
S 3 := { (x1 , x2 , x3 , x4 ) R4 : x2j = 1 }
j=1

we identitify with the Lie group SU (2)


 
x1 + ix2 x3 + ix4
(x1 , x2 , x3 , x4 ) 7 .
x3 + ix4 x1 ix2
(i) Map the standard basis of R4 into SU (2) and express these matrices using
the Pauli spin matrices and the 2 2 identity matrix.
(ii) Map the Bell basis
1 1 0 0

1 0 1 0 1 1 1 1
, , ,
2 0 0 2 1 2 1

2
1 1 0 0
into SU (2) and express these matrices using the Pauli spin matrices and the
2 2 identity matrix.

Problem 6. Is the 3 3 matrix



sin cos sin cos cos
O(, ) = sin sin cos cos sin
cos 0 sin
an element of the compact Lie group SO(3)?

Problem 7. Are the matrices


1 1 1 1 1


1 1 1 1 1 0 1 1
A = 1 0 1, B = 1 0 0 1 1

1 1 0 1 1 1 0 0

1 1 1 0 1
234 Problems and Solutions

elements of SL(3, R) and SL(5, R), respectively? We have to test that det(A) =
1 and det(B) = 1.

Problem 8. (i) Let R. Do the matrices


 
cos i sin
A() =
i sin cos
form a group under matrix multiplication?
(i) Let R. Do the matrices
 
cosh i sinh
A() =
i sinh cosh
form a group under matrix multiplication?

Problem 9. Consider the Lie group SL(n, C), i.e. the n n matrices over C
with determinant 1. Can we find A, B SL(n, C) such that [A, B] is an element
of SL(n, C)?

Problem 10. Consider the 2 2 matrices


   
1 0 0 1
A() = , B= .
1 1 0
Both are elements of the non-compact Lie group SL(2, C). Can one finds C
such that the commutator [A(), B] is again an element of SL(2, C)?

Problem 11. (i) Let A, B be elements of SL(n, R). Is A B an element of


SL(n2 , R).
(ii) Let A, B be elements of SL(n, R). Is A B an element of SL(2n, R).
(iii) Let A, B be elements of SL(2, R). Is
a11 0 0 a12

0 b11 b12 0
A ? B :=
0 b21 b22 0

a21 0 0 a22
an element of SL(4, R)?

Problem 12. (i) The matrix


 
cos sin
A() =
sin cos
is an element of the Lie group SO(2, R). Find the spectral decomposition of
A().
Lie Groups 235

(ii) The matrix


 
cosh sinh
B() =
sinh cosh

is an element of the Lie group SO(1, 1, R). Find the spectral decomposition of
B().
(iii) Find the spectral decomposition of A() B().

Problem 13. Let A be an n n matrix over C. Show that if AT = A, then


eA O(n, C).

Problem 14. The Lie group SU (2) is defined by

SU (2) := { U 2 2 matrix : U U = I2 , det U = 1 }.

Let (3-sphere)

S 3 := { (x1 , x2 , x3 , x4 ) R4 : x21 + x22 + x23 + x24 = 1 }.

Show that SU (2) can be identified as a real manifold with the 3-sphere S 3 .

Problem 15. Both


   
cos sin cosh sinh
A() = , B() =
sin cos sinh cosh

are elements of the Lie group SL(2, R). Are

A() B(), A() B(), A() ? B()

elements of SL(4, R)?

Problem 16. The Lie group SU (1, 1) consists of all 2 2 matrices T over the
complex numbers with
 
1 0
T M T = M, M = , det(T ) = 1.
0 1

Find the conditions on 0 , 1 , 2 , 3 R such that


 
0 + i3 1 + i2
T =
1 i2 0 i3

is an element of SU (1, 1).


236 Problems and Solutions

Problem 17. Let n be an integer with n 2. Let p, q be integers with p, q 1


and n = p + q. Let Ip be the p p identity matrix and Iq be the q q identity
matrix. Let  
Ip 0
Ip,q = Ip (Iq ).
0 Iq
The Lie group O(p, q) is the set of all n n matrices defined by

O(p, q) := { A GL(n, R) : AT Ip,q A = Ip,q }.

Show that this is the group of all invertible linear maps of Rn that preserves the
quadratic form
X p Xn
xj yj xj yj .
j=1 j=p+1

Problem 18. Consider the Pauli spin matrices 1 , 2 , 3 . Then i1 , i2 , i3


are elements of the Lie group SU (2). Consider the unitary matrix (constructed
from the four Bell states)

1 0 0 i

1 0 i 1 0
U= .
2 0 i 1 0
1 0 0 i

Show that
U (i1 i1 )U, U (i2 i2 )U, U (i3 i3 )U
are elements of the Lie group SO(4).

Problem 19. Consider the differentiable manifold

S 3 = { (x1 , x2 , x3 , x4 ) : x21 + x22 + x23 + x24 = 1 }.

(i) Show that the matrix


 
x3 + ix4 x1 ix2
U (x1 , x2 , x3 , x4 ) = i
x1 + ix2 x3 + ix3

is unitary. Show that the matrix is an element of SU (2).


(ii) Consider the parameters (, , ) with 0 < , 0 < 4, 0 < 2.
Show that

x1 (, , ) + ix2 (, , ) = cos(/2)ei(+)/2
x3 (, , ) + ix4 (, , ) = sin(/2)ei()/2
Lie Groups 237

is a parametrization. Thus the matrix given in (i) takes the form

cos(/2)ei(+)/2
 
sin(/2)ei()/2
i .
cos(/2)ei(+)/2 sin(/2)ei()/2

(iii) Let (1 , 2 , 3 ) = (, , ) with 0 < , 0 < 4, 0 < 2. Show


that
Z Z 4 Z 2 3
1 X U 1 U 1 U
d d d jk` tr(U 1 U U )=1
24 2 0 0 0 j k `
j,k,`=1

where 123 = 321 = 132 = +1, 213 = 321 = 132 = 1 and 0 otherwise.
(iv) Consider the metric tensor field

g = dx1 dx1 + dx2 dx2 + dx3 dx3 + dx4 dx4 .

Using the parametrization show that


1
gS 3 = (d d + d d + d d + cos()d d + cos()d d).
4
(v) Consider the differential one forms e1 , e2 , e3 defined by
dx1
e1 x4 x3 x2 x1
dx
e2 = x3 x4 x1 x2 2 .
dx3
e3 x2 x1 x4 x3
dx4

Show that
gS 3 = de1 de1 + de2 de2 + de3 de3 .
(vi) Show that
3
X
dej = jk` ek e`
k,`=1

i.e. de1 = 2e2 e3 , de2 = 2e3 e1 , de3 = 2e1 e2 .

Problem 20. The Lie group SU (2, 2) is defined as the group of transfor-
mation on the four dimensional complex space C4 leaving invariant the indefinite
quadratic form
|z1 |2 + |z2 |2 |z3 |2 |z4 |2 .
The Lie algebra su(2, 2) is defined as the 4 4 matrices X with trace 0 and
X L + LX = 04 , where L is the 4 4 matrix
 
I2 02
L= .
02 I2
238 Problems and Solutions

Is
0 0 0 1

0 0 1 0
X=
0 1 0 0

1 0 0 0
an element of the Lie algebra su(2, 2). Find exp(zX). Discuss.

Problem 21. (i) Can any element of the Lie group SU (1, 1) be written as
 i/2    i/2 
e 0 cosh(/2) sinh(/2) e 0
?
0 ei/2 sinh(/2) cosh(/2) 0 ei/2

Each element in the this product is an element of the Lie group SU (1, 1).
(ii) Can any element of the compact Lie group SU (2) be written as
   i/2  
cos(/2) sin(/2) e 0 cos(/2) sin(/2)
?
sin(/2) cos(/2) 0 ei/2 sin(/2) cos(/2)

Problem 22. (i) Consider the non-compact Lie group SO(1, 1, R) with the
element  
cosh() sinh()
A() = .
sinh() cosh()
Find the inverse of A(). Find the eigenvalues and eigenvectors of A().
(ii) Let be the direct sum. Find the determinant, eigenvalues and normalized
eigenvectors of the 3 3 matrix

(1) A().

(iii) Find the determinant, eigenvalues and normalized eigenvectors of the matrix

cosh() 0 sinh()
0 1 0 .
sinh() 0 cosh()

Find the inverse of this matrix.

Problem 23. Let R.


(i) Is the 2 2 matrix
    
cos() sin() 1 1 1 1 0
M1 () =
sin() cos() 2 1 1 0 i

unitary? Prove or disprove. If so is the matrix an element of the Lie group


SU (2)?
Lie Groups 239

(ii) Is the 8 8 matrix


     
cos() sin() 1 1 1 1 0
M2 () =
sin() cos() 2 1 1 0 i

unitary? Prove or disprove. If so is the matrix an element of the Lie group


SU (4)?
(iii) Let be the direct sum. Is the 6 6 matrix
     
cos() sin() 1 1 1 1 0
M3 () =
sin() cos() 2 1 1 0 i

unitary? Prove or disprove. If so is the matrix an element of the Lie group


SU (4)?

Problem 24. Write down two 2 2 matrices A and B which are elements of
the Lie group O(2, R) but not elements of SO(2, R).
(i) Is AB an element of the Lie group SO(2, R)?
(ii) Is A B an element of the Lie group SO(4, R)?
(iii) Is A B an element of SO(4, R)?

Problem 25. Let R. Consider the 2 2 matrix


 
1 e e
A() = .
2 e e

(i) Find the trace, determinant, eigenvalues and normalized eigenvectors of the
matrix A().
(ii) Calculate
dA()
X := .
d =0

Find exp(X) and compare with A(), i.e. is exp(X) = A()? Discuss.

Problem 26. Consider the matrix ( R)


 
cos() sin()
A() = .
sin() cos()

(i) Is A() an element of SO(2)?


(ii) Consider the transformation
 0  
x1 x1
= A() .
x02 x2

Is (x01 )2 + (x02 )2 = x21 + x22 ? Prove or disprove.


240 Problems and Solutions

(iii) We define

cos() 0 0 sin()

0 cos() sin() 0
A() ? A() = .

0 sin() cos() 0
sin() 0 0 cos()

Is A() ? A() an element of SO(4).


(iv) Is A() A() an element of SO(4)?
(v) Find
dA()
X=
d =0
and then B() = exp(X). Compare B() and A() and discuss.

Problem 27. Let z = x + iy with x, y R. Consider the 2 2 matrix


 
cos(z) sin(z)
sin(z) cos(z)

One has

cos(x + iy) = cos(x) cos(iy) sin(x) sin(iy) = cos(x) cosh(y) i sin(x) sinh(y)
sin(x + iy) = sin(x) cos(iy) + cos(x) sin(iy) = sin(x) cosh(y) + i cos(x) sinh(y).

Let x = 0. Then we arrive at the matrix


 
cosh(y) i sinh(y)
M (y) = .
i sinh(y) cosh(y)

(i) Do these matrices (y R form a group under matrix multiplication?


(ii) Calculate
d
X= M (y)
dy y=0

and then exp(yX) with y R. Discuss.

Problem 28. Let , R. Do the 3 3 matrices



cos() sin() 0 1 0 0
A(, ) = sin() cos() 0 0 cosh() sinh()
0 0 1 0 sinh() cosh()

form a group under matrix multiplication? For = = 0 we have the indenity


matrix.

Problem 29. Let A, B SL(2, R).


Lie Groups 241

(i) Is trA = trA1 ? Prove or disprove.


(ii) Is
tr(AB) = trA trB tr(AB 1 ) ?
Prove or disprove.

Problem 30. The group of complex rotations O(n, C) is defined as the group
of all n n complex matrices O, such that OOT = OT O = In , where T means
transpose. These transformations preserve the real scalar product
n
X
xy = xj yj
j=1

so that (Ox) Oy = x y, where x and y are complex vectors in general, i.e.


xj , yj C.
(i) Show that the matrix ( R)
 
cosh i sinh
O=
i sinh cosh
is an element of O(n, C).
(ii) Find the partial derivatives under complex orthogonal transformations O
O(n, C)
Xn
wj (x) := (Ox)j = Ojk xk , j = 1, 2, . . . , n
k=1
i.e. /wj with j = 1, 2, . . . , n.

Problem 31. The Lie group SL(2, C) consists of all 22 matrices over C with
determinant equal to 1. The group is not compact. Explain why. The maximal
compact Lie subgroup of SL(2, C) is SU (2). Give a 2 2 matrix A which is an
element of SL(2, C), but not an element of SU (2). .

Problem 32. Consider the Lie group SL(2, R), i.e. the set of all real 2 2
matrices with determinant equal to 1. A dynamical system in SL(2, R) can be
defined by
Mk+2 = Mk Mk+1 k = 0, 1, 2, . . .
with the initial matrices M0 , M1 SL(2, R). Let Fk := trMk . Is
Fk+3 = Fk+2 Fk+1 Fk k = 0, 1, 2, . . . ?
Prove or disprove.
Hint. Use that property that for any 2 2 matrix A we have
A2 Atr(A) + I2 det(A) = 0.
Chapter 18

Lie Algebras

Problem 1. Let L be a finite dimensional Lie algebra and Z(L) the center of
L. Show that ad : L g`(L) is a homomorphisim of the Lie algebra L with
kernel Z(L).

Problem 2. Consider the Lie algebra s`(2, R) with the basis


     
0 1 0 0 1 0
e= , f= , h= .
0 0 1 0 0 1
Show that the Lie algebra s`(2, R) has no proper nontrivial ideals.

Problem 3. Consider the Lie algebra s`(2, R) with the basis


     
0 1 0 0 1 0
e= , f= , h= .
0 0 1 0 0 1
Find a basis of s`(2, R) with all basis elements are normal matrices.

Problem 4. Consider the 2 2 matrices over R


   
1 0 0 1
A= , B= .
1 0 0 1
Calculate the commutator C = [A, B] and check whether C can be written as a
linear combination of A and B. If so we have a basis of a Lie algebra.

Problem 5. Show that s`(2, F) has no non-trivial ideals if char(F) 6= 2.

242
Lie Algebras 243

Problem 6. Consider the simple Lie algebra s`(2, R) and the basis E, F , H
with the commutators
[E, F ] = H, [H, E] = 2E, [H, F ] = 2F.
Let U (s`(2, R)) be the universal enveloping algebra. Then any element of U (s`(2, R))
can be expressed as a sum of product of the form F j H k E ` where j, k, ` 0.
Show that
EF 2 = 2F + 2F H + F 2 E.
Hint: Utilize that EF 2 [E, F 2 ] + F 2 E.

Problem 7. A basis of the Lie algebra s`(2, R) is given by


     
0 1 0 0 1 0
E= , F = , H= .
0 0 1 0 0 1
(i) Which of these matrices are nonnormal?
(ii) Use linear combinations to find a basis where all elements are normal ma-
trices.

Problem 8. Study the Lie algebra s`(2, F), where charF = 2.

Problem 9. A Lie algebra is simple if it contains no nontrivial ideals and


semi-simple if its contains no nontrivial abelian (commutative) ideals. Is the Lie
algebra s`(2, R) with the basis
     
0 1 0 0 1 0
, ,
0 0 1 0 0 1
simple?

Problem 10. Do the matrices



0 1 0 0 0 0 1 0 0
E+ = 0 0 1 , E = 1 0 0, H = 0 0 0
0 0 0 0 1 0 0 0 1
form a basis for the Lie algebra s`(2, R).

Problem 11. Consider the Pauli spin matrices


       
1 0 0 1 0 i 1 0
0 = , 1 = , 2 = , 3 = .
0 1 1 0 i 0 0 1
(i) Do the sixteen 4 4 matrices (j = 0, 1, 2, 3)
       
j 0 2 j 02 0 2 j 02 j
, , ,
02 j 02 j j 02 j 02
244 Problems and Solutions

form a group under matrix multiplication?


(ii) Do the sixteen 4 4 matrices (j = 0, 1, 2, 3)
       
j 0 2 j 02 02 j 02 j
, , ,
02 j 02 j j 02 j 02
form a Lie algebra under the commutator?

Problem 12. The isomorphism of the Lie algebras s`(2, C) and so(3, C) has
the form
  0 b c i(b + c)
a b
cb 0 2ia .
c a
i(b + c) 2ia 0
Let z C. Find

   0 bc i(b + c)
a b
exp z , exp z c b 0 2ia .
c a
i(b + c) 2ia 0

Problem 13. In the decomposition of the simple Lie algebra s`(3, R) one finds
the 3 3 matrices

a11 a12 0 0 0 b13
A = a21 a22 0 , B = 0 0 b23
0 0 a11 a22 b31 b32 0

where ajk , bjk R. Find the commutators [A, A0 ], [B, B 0 ] and [A, B]. Discuss.

Problem 14. We know that


     
0 1 1 0 0 0
X= , H= , Y =
0 0 0 1 1 0

is an ordered basis of the simple Lie algebra s`(2, R) with

[X, H] = 2X, [X, Y ] = H, [Y, H] = 2Y.

Consider
0 0 0 1 1 0 0 0 0 0 0 0

0 0 1 0 0 1 0 0 0 0 0 0
X=
e , H=
e , Y =
e .
0 0 0 0 0 0 1 0 0 1 0 0
0 0 0 0 0 0 0 1 1 0 0 0
Find the commutators

[X,
e H],
e [X,
e Ye ], [Ye , H].
e
Lie Algebras 245

Problem 15. The simple Lie algebra s`(2, R) has a basis given by the matrices
     
1 0 0 1 0 0
H= , E= , F =
0 1 0 0 1 0

with the commutation relations

[H, E] = 2E, [H, F ] = 2F, [E, F ] = H.

The universal enveloping algebra U (s`(2, R)) of the Lie algebra s`(2, R) is the
associative algebra with generators H, E, F and the relations

HE EH = 2E, HF F H = 2F, EF F E = H.

Find a basis of the Lie algebra s`(2, R) so that all matrices are invertible. Find
the inverse matrices of these matrices. Give the commutation relations.

Problem 16. A Chevalley basis for the semisimple Lie algebra s`(3, R) is given
by
0 0 0 0 1 0 0 0 1
X1 = 0 0 1 , X2 = 0 0 0 , X3 = 0 0 0
0 0 0 0 0 0 0 0 0

0 0 0 0 0 0 0 0 0
Y1 = 0 0 0 , Y2 = 1 0 0 , Y3 = 0 0 0
0 1 0 0 0 0 1 0 0

0 0 0 1 0 0
H1 = 0 1 0 , H2 = 0 1 0
0 0 1 0 0 0
where Yj = XjT for j = 1, 2, 3. The Lie algebra has rank 2 owing to H1 , H2
and [H1 , H2 ] = 0. Another basis could be formed by looking at the linear
combinations
Uj = Xj + Yj , Vj = Xj Vj .
(i) Find the table of the commutator.
(ii) Calculate the vectors of commutators
     
[H1 , X1 ] [H1 , X2 ] [H1 , X3 ]
, ,
[H2 , X1 ] [H2 , X2 ] [H2 , X3 ]

and thus find the roots.

Problem 17. Given the spin matrices


   
0 1 0 0
s+ = , s = .
0 0 1 0
246 Problems and Solutions

Consider the 4 4 matrices


       
02 s 02 02 s+ 02 s 02
, , , .
02 02 s+ 02 02 s+ 02 s
Calculate the commutators of these matrices and extend the set so that one finds
a basis of a Lie algebra.

Problem 18. Let L1 and L2 be two Lie algebras. Let : L1 L2 be a Lie


algebra homomorphism. Show that im() is a Lie subalgebra of L2 and ker()
is an ideal in L1 .

Problem 19. Let m, n {1, 0, 1}. Classify the Lie algebra with the genera-
tors L1 , L0 , L1 given
i[Ln , Lm ] = (n m)Ln+m .

Problem 20. (i) The standard basis for the vector space of the 2 2 matrices
is given by
       
1 0 0 1 0 0 0 0
E11 = , E12 = , E21 = , E22 = .
0 0 0 0 1 0 0 1
We define the star composition of two 2 2 matrices as the 4 4 matrix
a11 0 0 a12

0 b11 b12 0
A ? B := .
0 b21 b22 0
a21 0 0 a22
Show that the sixteen 4 4 matrices Ejk ? E`m (j, k, `, m = 1, 2) form a basis in
the vector space of the 4 4 matrices.
(ii) The matrices
     
0 1 0 0 1 0
X= , Y = , H=
0 0 1 0 0 1
form a basis of the Lie algebra s`(2, R). Do the nine 4 4 matrices
X ? X, X ? Y, X ? H, Y ? X, Y ? Y, Y ? H, H ? X, H ? Y, H ? H
form a basis of a Lie algebra?

Problem 21. Consider the Lie algebra of real-skew symmetric 3 3 matrices



0 a3 a2
A = a3 0 a1 .
a2 a1 0
Lie Algebras 247

Let R be a real orthogonal 3 3 matrix, i.e. RRT = I3 . Show that RART is a


real-skew symmetric matrix.

Problem 22. The matrices


     
1 0 0 1 0 0
H= , E= , F =
0 1 0 0 1 0

form a basis of the simple Lie algebra s`(2, R). Define the matrices

(H) = H I2 +I2 H, (E) = E H 1 +H E, (F ) = F H 1 +H F.

Find the commutators

[(H), (E)], [(H), (F )], [(E), (F )].

Discuss.

Problem 23. The Heisenberg group is the set of upper 3 3 matrices of the
form
1 a c
H = 0 1 b
0 0 1
where a, b, c can be taken from some (arbitrary) commutative ring.
(i) Find the inverse of H.
(ii) Given two elements x, y of a group G, we define the commutator of x and
y, denoted by [x, y] to be the element x1 y 1 xy. If a, b, c are integers (in the
ring Z of the integers) we obtain the discrete Heisenberg group H3 . It has two
generators
1 1 0 1 0 0
x = 0 1 0, y = 0 1 1.
0 0 1 0 0 1
Find
z = xyx1 y 1 .
Show that xz = zx and yz = zy.
(iii) The derived subgroup (or commutator subgroup) of a group G is the sub-
group [G, G] generated by the set of commutators of every pair of elements of
G. Find [G, G] for the Heisenberg group.
(iv) Let
0 a c
A = 0 0 b
0 0 0
and a, b, c R. Find exp(A).
248 Problems and Solutions

(v) The Heisenberg group is a simple connected Lie group whose Lie algebra
consists of matrices
0 a c
L = 0 0 b.
0 0 0
Find the commutators [L, L0 ] and [L, L0 ], L0 ]], where [L, L0 ] := LL0 L0 L.

Problem 24. Consider the 3 3 matrices



1 0 0 0 0 0 0 0 1
h1 = 0 0 0 , h2 = 0 1 0 , h3 = 0 0 0
0 0 1 0 0 0 1 0 0

0 1 0 0 0 0
e = 0 0 0, f = 1 0 1.
0 1 0 0 0 0
Show that the matrices form a basis of a Lie algebra.

Problem 25. Consider the two 2 2 matrices


   
1 0 0 1
A= , B=
0 1 1 0

where A is the Pauli spin matrix 3 and B the Pauli spin matrix 1 . The two
matrices A, B are linearly independent. Let A, B be the generators of a Lie
algebra. Classify the Lie algebra generated.

Problem 26. The Lie group SU (1, 1) consists of all 2 2 matrices T over the
complex numbers with
 
1 0
T M T = M, M = , det(T ) = 1.
0 1

Find the conditions on 0 , 1 , 2 , 3 R such that


 
0 + i3 1 + i2
T =
1 i2 0 i3

is an element of SU (1, 1).

Problem 27. (i) Find the Lie algebra generated by the 2 2 matrices
   
0 0 0 0
A= , B= .
0 1 1 0
Lie Algebras 249

(ii) Find the Lie algebra generated by the 3 3 matrices



0 0 0 0 0 0 0 0 0
A = 0 1 0 , B = 1 0 0, C = 0 0 0.
0 0 1 0 0 0 1 0 0

Problem 28. Can one find nonzero 2 2 matrices L1 , L2 , L3 over C such that

[L1 , L2 ] = L3 , [L2 , L3 ] = 02 , [L3 , L1 ] = L2 .

Problem 29. The Lie group SU (2, 2) is defined as the group of transfor-
mation on the four dimensional complex space C4 leaving invariant the indefinite
quadratic form
|z1 |2 + |z2 |2 |z3 |2 |z4 |2 .
The Lie algebra su(2, 2) is defined as the 4 4 matrices X with trace 0 and
X L + LX = 04 , where L is the 4 4 matrix
 
I2 02
L= .
02 I2

Is
0 0 0 1

0 0 1 0
X=
0 1 0 0

1 0 0 0
an element of the Lie algebra su(2, 2). Find exp(zX). Discuss.

Problem 30. Show that a basis of the Lie algebra s`(2, C) is given by
     
1 0 i 1 0 1 1 i 0
, , ,
2 i 0 2 1 0 2 0 i
     
1 0 1 1 0 i 1 1 0
, , .
2 1 0 2 i 0 2 0 1

Problem 31. Consider the non-commutative two-dimensional Lie algebra with


[A, B] = A where    
0 1 0 0
A= , B= .
0 0 0 1
Show that the 4 4 matrices

{A I2 + I2 A, B I2 + I2 B}
250 Problems and Solutions

also form a noncommutative two-dimensional Lie algebra under the commutator.

Problem 32. The g`(1|1) superalgebra involves two even (denoted by h and
z) and two odd (denoted by e, f ) generators. The following commutation and
anti-commutation relations hold

[z, e] = [z, f ] = [z, h] = 0, [h, e] = e, [h, f ] = f, [e, f ]+ = z

and e2 = f 2 = 0. Find a 2 2 matrix representation.

Problem 33. Consider the two 2 2 matrices


   
1 0 0 1
A= , B=
0 1 1 0

where A is the Pauli spin matrix 3 and B the Pauli spin matrix 1 . The two
matrices A, B are linearly independent. Let A, B be the generators of a Lie
algebra. Classify the Lie algebra generated.

Problem 34. A basis of the simple Lie algebra s`(2, R) is given by the traceless
2 2 matrices
     
1 0 1 1 1 0 1 0 1
X1 = , X2 = , X3 = .
2 1 0 2 0 1 2 1 0

(i) Find the commutators [X1 , X2 ], [X2 , X3 ], [X3 , X1 ].


(ii) Let z C. Find
ezX1 , ezX2 , ezX3 .
(iii) Let u, v, r R. Show that

g(u, v, r) = euX3 erX2 evX3


 
sin(u/2) sin(v/2) cos(v/2) sin(u/2)
= er/2
cos(u/2) sin(v/2) cos(u/2) cos(v/2)
 
cos(u/2) cos(v/2) cos(u/2) sin(v/2)
+er/2 .
cos(v/2) sin(u/2) sin(u/2) sin(v/2)

(iv) Find g(u, v, r)1 .

Problem 35. Let Ejk (j, k = 1, 2, 3, 4) be the standard basis in the vector
space of 4 4 matrices. This means that Ejk is the matrix with +1 at entry
(jk) (jth column and kth row) and 0 otherwise. Show that the 15 matrices

X1 = E12 , X2 = E23 , X3 = E13 , X4 = E34 , X5 = E24 , X6 = E14

Y1 = E21 , Y2 = E32 , Y3 = E31 , Y4 = E43 , Y5 = E42 , Y6 = E41


Lie Algebras 251

1
H1 = diag(3, 1, 1, 1, 1),
4
1
H2 = diag(1, 1, 1, 1),
2
1
H3 = diag(1, 1, 1, 3).
4
Show that these matrices form a basis (Cartan-Weyl basis) of the Lie algebra
s`(4, C).

Problem 36. Do the eight 3 3 skew-hermitian matrices



0 i 0 0 1 0
1 1
1 = i 0 0 , 2 = 1 0 0 ,
2 0 0 0 2 0 0 0

i 0 0 0 0 i
1 1
3 = 0 i 0 , 4 = 0 0 0,
2 0 0 0 2 i 0 0

0 0 1 0 0 0
1 1
5 = 0 0 0 , 6 = 0 0 i ,
2 1 0 0 2 0 i 0

0 0 0 i 0 0
1 1
7 = 0 0 1 , 8 = 0 i 0
2 0 1 0 6 0 0 2i
together with the 33 unit matrix form an orthonormal basis in the vector space
of 3 3 matrices over the complex number. Find all pairwise scalar products
hA, Bi := tr(AB ). Discuss.

Problem 37. The semisimple Lie algebra s`(3, R has dimension 8. The stan-
dard basis is given by

1 0 0 0 0 0
h1 = 0 1 0, h2 = 0 1 0 ,
0 0 0 0 0 1

0 1 0 0 0 0
e1 = 0 0 0, e2 = 0 0 1 ,
0 0 0 0 0 0

0 0 0 0 0 0
f1 = 1 0 0, f2 = 0 0 0 ,
0 0 0 0 1 0
252 Problems and Solutions

0 0 1 0 0 0
e13 = 0 0 0, f13 = 0 0 0 .
0 0 0 1 0 0
Find the commutator table.

Problem 38. Consider a four dimensional vector space with basis e1 , e2 , e3 ,


e4 . Assume that the non-zero commutators are given

[e2 , e3 ] = e1 , [e1 , e4 ] = 2e1 , [e2 , e4 ] = e2 , [e3 , e4 ] = e2 + e3 .

Do these relations define a Lie algebra? If so find the adjoint representation.

Problem 39. Let A, B, C be nonzero nn matrices. Assume that [A, B] = 0n


and [C, A] = 0n . Can we conclude that [B, C] = 0n ? Is the Jacobi identity

[A, [B, C]] + [C, [A, B]] + [B, [C, A]] = 0n

of any use?

Problem 40. Let 1 , 2 , 3 be the Pauli spin matrices. Applying the star
operation ? we obtain
0 0 0 1 0 0 0 i 1 0 0 0

0 0 1 0 0 0 i 0 0 1 0 0
1 ?1 = , 2 ?2 = , 3 ?3 = .
0 1 0 0 0 i 0 0 0 0 1 0
1 0 0 0 i 0 0 0 0 0 0 1
(i) Find

[1 , 2 ], [2 , 3 ], [3 , 1 ], [1 ?1 , 2 ?2 ], [2 ?2 , 3 ?3 ], [3 ?3 , 1 ?1 ].

Discuss.
(ii) Find

[1 , 2 ]+ , [2 , 3 ]+ , [3 , 1 ]+ , [1 ?1 , 2 ?2 ]+ , [2 ?2 , 3 ?3 ]+ , [3 ?3 , 1 ?1 ]+ .

Discuss.

Problem 41. Consider the semisimple Lie algebra s`(n + 1, F). Let Ei,j
(i, j {1, 2, . . . , n + 1}) denote the standard basis, i.e. (n + 1) (n + 1) matrices
with all entries zero except for the entry in the i-th row and j-th column which
is one. We can form a Cartan-Weyl basis with

Hj := Ej,j Ej+1,j+1 , j {1, 2, . . . , n}.

Show that Ei,j are root vectors for i 6= j, i.e. there exists H,i,j F such that

[H, Ei,j ] = H,i,j Ei,j


Lie Algebras 253

for all H span{ H1 , . . . , Hn }.

Problem 42. Consider the vector space R3 and the vector product . The
vector product is not associative. The associator of three vectors u, v, w is
defined by
ass(u (v w)) := (u v) w u (v w).
The associator measures the failure of associativity.
(i) Consider the unit vectors

1 0 0
u = 0, v = 1, w = 0.
0 0 1

Find the associator.


(ii) Consider the normalized vectors

1 0 1
1 1
u= 0 , v = 1, w= 0 .
2 1 0 2 1

Find the associator.

Problem 43. Consider vectors in the vector space R3 and the vector product.
Consider the mapping of the vectors in R3 into 3 3 skew-symmetric matrices

a 0 c b
b c 0 a .
c b a 0

Calculate the vector product



a2 a1
b2 b1
c2 c1

and the commutator [M1 , M2 ], where



0 c1 b1 0 c2 b2
M1 = c1 0 a1 , M2 = c2 0 a2 .
b1 a1 0 b2 a2 0

Discuss.
Chapter 19

Inequalities

Problem 1. Let A be an n n positive semidefinite matrix. Let B be an


n n positive definite matrix. Then we have Kleins inequality

tr(A(ln(A) ln(B))) tr(A B).

(i) Let    
1/2 1/2 1/2 0
A= , B= .
1/2 1/2 0 1/2
Calculate the left-hand side and the right-hand side of the inequality.
(ii) When is the inequality an equality?

Problem 2. Let A, B be n n hermitian matrices. Then (Golden-Thompson-


Symanzik inequality)
treA+B tr(eA eB ).
Let A = 3 and B = 1 . Calculate the left and right-hand side of the inequality.

Problem 3. Let A, B, C be positive definite n n matrices. Then (Lieb


inequality)
Z
tr(e ln(A)ln(B)+ln(C)
) tr A(B + uIn )1 C(B + uIn )1 du.
0

(i) Let      
2 1 1 1 4 2
A= , B= , C= .
1 1 1 2 2 5

254
Inequalities 255

Calculate the left-hand side and right-hand side of the inequality.


(ii) Give a sufficient condition such that one has an equality.

Problem 4. Let A be an n n skew-symmetric matrix over R. Show that

det(In + A) 1

with equality holding if and only if A = 0.

Problem 5. Let v be a normalized (column) vector in Cn and let A be an


n n hermitian matrix. Is
v eA v ev Av
for all normalized v? Prove or disprove.

Problem 6. Let A be an n n matrix over C and v, u Cn . Is

kAv Auk kAk kv uk ?

Problem 7. Let H be an n n hermitian matrix and v be a normalized


(column) vector in Cn . Is
v eH v ev Hv ?

Problem 8. Let A be an n n matrix over R. Show that there exists nonnull


vectors x1 , x2 in Rn such that

xT1 Ax1 xT Ax xT2 Ax2



xT1 x1 xT x xT2 x2

for every nonnull vector x in Rn .

Problem 9. Let A be an n n matrix over C. Is

kIn + Ak 1 + kAk ?
Chapter 20

Braid Group

Let n 2. The braid group Bn of n strings has n1 generators {1 , 2 , . . . , n1 }


(pairwise distinct) satisfying the relations

j j+1 j = j+1 j j+1 for j = 1, 2, . . . , N 2(Y ang Baxterrelation)


j k = k j for |j k| 2
j j1 = j1 j = e

where e is the identity element. Thus it is generated by elements j (j inter-


changes elements j and j + 1). Thus actually one should write 12 , 23 , . . . ,
n1n instead of 1 , 2 , . . . , n1 . The braid group Bn is a generalization of the
permutation group.

The word written in terms of letters, generators from the set

{ 1 , . . . , n1 , 11 , . . . , n1
1
}

gives a particular braid. The length of the braid is the total number of used let-
ters, while the minimal irreducible length (referred sometimes as the primitive
word) is the shortest non-contractible length of a particular braid which remains
after applying all the group relations given above.

Let n 2. The braid group on n strings, denoted by Bn , is an abstract group


defined via the presentation

Bn = h1 , . . . , n1 i

256
Braid Group 257

with the relations

i j = j i if |i j| 2
i i+1 i = i+1 i i+1 if 1 i n 2.

The generators 1 , 2 , . . . , n1 are called the standard generators of Bn .

The pure braid group denoted by Pn is defined as the kernel of the homomor-
phism Bn Sn defined by i (i, i + 1), (1 i n 1. It is finitely generated
by the elements
1 1 1
Aij = j1 j2 i+1 i2 i+1 j2 j1 , 1 i < j n.
258 Problems and Solutions

Problem 1. Consider the braid group B5 with the generators 1 , 2 , 3 , 4 .


Simplify
11 41 31 2 3 2 31 4 1 .

Problem 2. Consider the braid group B3 . A faithful representation for the


generators 1 and 2 is
   
1 0 1 1
1 = , 2 = .
1 1 0 1

Both are elements of SL(2, Z). Find the inverse of 1 and 2 . Do the elements
1 , 2 , 11 , 21 and the 2 2 identity matrix form a group under matrix
multiplication?

Problem 3. Consider the unitary 2 2 matrices


 i   
e 0 cos() i sin()
A() = , B() =
0 ei i sin() cos()

where R.
(i) Find the conditions on 1 , 2 , 3 R such that (braid like relation)

A(1 )B(2 )A(3 ) = B(3 )A(2 )B(1 ).

(ii) Do the matrices A() form a group under matrix multiplication?


(iii) Do the matrices B() form a group under matrix multiplication?

Problem 4. Find all invertible 2 2 matrices A, B such that (braid-like


relation)
ABA = BAB.

Problem 5. Can one find 22 matrices A and B with [A, B] 6= 0 and satisfying
the braid-like relation
ABBA = BAAB.

Problem 6. (i) Do the 2 2 unitary matrices


 i/4   
e 0 1 1 i
A= , B=
0 iei/4 2 i 1

satisfy the braid-like relation

ABA = BAB.
Braid Group 259

(ii) Find the smallest n N such that An = I2 .


(iii) Find the smallest m N such that B m = I2 .

Problem 7. If V and W are matrices of the same order, then their Schur
product V W is defined by (entrywise multiplication)
(V W )j,k := Vj,k Wj,k .
If all entries of V are nonzero, then we say that X is Schur invertible and define
its Schur inverse, V () , by V () V = J, where J is the matrix with all 1s.

The vector space Mn (F) of n n matrices acts on itself in three distinct ways:
if C Mn (F) we can define endomorphisms XC , C and YC by
XC M := CM, C M := C M, YC := M C T .
Let A, B be n n matrices. Assume that XA is invertible and B is invertible
in the sense of Schur. Note that XA is invertible if and only if A is, and B is
invertible if and only if the Schur inverse B () is defined. We say that (A, B) is
a one-sided Jones pair if
XA B XA = B XA B .
We call this the braid relation. Give an example for a one-sided Jones pair.

Problem 8. The braid linking matrix B is a square symmetric k k matrix


defined by B = (bij ) with bii the sum of half-twists in the i-th branch, bij the
sum of the crossings between the i-th and the j-th branches of the ribbon graph
with standard insertion. Thus the i-th diagonal element of B is the local torsion
of the i-th branch. The off-diagonal elements of B are twice the linking numbers
of the ribbon graph for the i-th and j-th branches. Consider the braid linking
matrix
1 0 1
B= 0 2 1 .
1 1 0
Discuss. Draw a graph.

Problem 9. Consider the five 4 4 matrices


i 0 0 0 1 0 i 0

0 i 0 0 1 0 1 0 i
B1 = , B2 = ,
0 0 1 0 2 i 0 1 0
0 0 0 1 0 i 0 1
i 0 0 0 1 i 0 0

0 1 0 0 1 i 1 0 0
B3 = , B4 = ,
0 0 1 0 2 0 0 1 i
0 0 0 i 0 0 i 1
260 Problems and Solutions

i 0 0 0

0 1 0 0
B5 = .
0 0 i 0
0 0 0 1
Is

B1 B2 B1 = B2 B1 B2 , B2 B3 B2 = B3 B2 B3 , B3 B4 B3 = B4 B3 B4 , B4 B5 B4 = B5 B4 B5 ?

Problem 10. Let n 3 and let 1 , . . . , n1 be the generators. The braid


group Bn on n-strings where n 3 has a finite presentation of Bn given by

h1 , . . . , n1 : i j = j i , i+1 i i+1 = i i+1 i i

where 1 i, j < n 1, |i j| > 1 or j = n 1. Here i j = j i and


i i+1 i = i+1 i i+1 are called the braid relations. The second one is also
called the Yang-Baxter equation.
(i) Consider B3 , a = 1 2 1 and b = 1 2 . Show that a2 = b3 .
(ii) Consider B3 . The cosets [1 ] of 1 and [2 ] of 2 map to the 2 2 matrices
   
1 1 1 1 0
[1 ] 7 R = , [2 ] 7 L =
0 1 1 1

where L, R SL(2, Z). Thus L1 , R1 SL(2, Z). Show that

RL1 R = L1 RL1 .

Problem 11. (i) Do the matrices


   
t 1 1 0
S1 = , S2 =
0 1 t t

satisfy the braid-like relation S1 S2 S1 = S2 S1 S2 .


(ii) Do the matrices S1 S1 and S2 S2 satisfy the braid-like relation

(S1 S1 )(S2 S2 )(S1 S1 ) = (S2 S2 )(S1 S1 )(S2 S2 ) ?

Problem 12. Consider the five 4 4 matrices

i 0 0 0 1 0 i 0 i 0 0 0

0 i 0 0 1 0 1 0 i 0 1 0 0
B1 = , B2 = , B3 = ,
0 0 1 0 2 i 0 1 0 0 0 1 0
0 0 0 1 0 i 0 1 0 0 0 i
Braid Group 261

1 i 0 0 i 0 0 0

1 i 1 0 0 0 1 0 0
B4 = , B5 = .
2 0 0 1 i 0 0 i 0
0 0 i 1 0 0 0 1
Are the matrices unitary? Is (braid-like relation)

Bj Bj+1 Bj = Bj+1 Bj Bj+1 , j = 1, 2, 3, 4

Problem 13. Let 1 , 2 , 3 be the Pauli spin matrices. Consider the 4 4


matrix
R = a(, )1 1 + b(, )(2 2 + 3 3 )
where
1 2 + 2 1
a(, ) = , b(, ) = .
4 2 2 2 2 2
Does R satisfy the braid like relation

(R I2 )(I2 R)(R I2 ) = (I2 R)(R I2 )(I2 R) ?

Problem 14. Consider the 4 4 matrix


a(u) 0 0 d(u)

0 b(u) c(u) 0
R(u) = .
0 c(u) b(u) 0
d(u) 0 0 a(u)

What is the condition on a(u), b(u), c(u), d(u) such that R(u) satisfies the braid
like relation

(R(u) I2 )(I2 R(u))(R(u) I2 ) = (I2 R(u))(R(u) I2 )(I2 R(u)) ?

Problem 15. Consider the five 4 4 matrices


i 0 0 0 1 0 i 0 i 0 0 0

0 i 0 0 1 0 1 0 i 0 1 0 0
B1 = , B2 = , B3 =

0 0 1 0 2 i 0 1 0 0 0 1 0

0 0 0 1 0 i 0 1 0 0 0 i

1 i 0 0 i 0 0 0

1 i 1 0 0 0 1 0 0
B4 = , B5 = .
2 0 0 1 i 0 0 i 0
0 0 i 1 0 0 0 1
262 Problems and Solutions

Show that

B1 B2 B1 = B2 B1 B2 , B2 B3 B2 = B3 B2 B3 , B3 B4 B3 = B4 B3 B4 , B4 B5 B4 = B5 B4 B5

and
B1 B3 = B3 B1 , B2 B4 = B4 B2 , B3 B5 = B5 B3 .

Problem 16. (i) Let R be an m m matrix and In be the n n identity


matrix. Consider the braid-like relation

(R In )(In R)(R In ) = (In R)(R In )(In R).

Let m = 4 and n = 2. Does


1 0 0 1

1 0 1 1 0
R=
0 1 1 0

2
1 0 0 1

satisfy the braid like relation?


(ii) Does
a 0 0 0

0 0 b 0
R=
0 c 0 0

0 0 0 d
satisfy the braid like relation? Or what is the condition on a, b, c, d so that the
condition is satisfied?
(iii) Does
0 0 0 a

0 b 0 0
R=
0 0 c 0

d 0 0 0
satisfy the braid like relation? Or what is the condition on a, b, c, d so that the
condition is satisfied?

Problem 17. Let T be an n n matrix and R be an n2 n2 matrix. Consider


the equation
R(T T ) = (T T )R.
(i) Let n = 2 and  
0 1
T = .
1 0
Find all 4 4 matrices R which satisfy the equation.
Braid Group 263

(ii) Let n = 2 and


1 0 0 0

0 0 1 0
R= .
0 1 0 0
0 0 0 1
Find all T which satisfy the equation.

Problem 18. (i) Let A, B be invertible n n matrices with AB 6= BA.


Assume that
ABA = BAB and ABBA = In .

Show that A4 = B 4 = In .
(ii) Find all 2 2 matrices A and B which satisfy the conditions given in (i).
(iii) Find all 3 3 matrices A and B which satisfy the conditions given in (i).
(iv) Find all 4 4 matrices A and B which satisfy the conditions given in (i).

Problem 19. Find all 4 4 matrices A, B with [A, B] 6= 04 satisfying the


conditions
ABA = BAB, ABBA = I4 .

The first condition is the braid relation and the second condition ABBA = I4
runs under Dirac game.

Problem 20. (i) Do the matrices


   
t 1 1 0
S1 = , S2 =
0 1 t t

satisfy the braid-like relation S1 S2 S1 = S2 S1 S2 .


(ii) Do the matrices S1 S1 and S2 S2 satisfy the braid-like relation

(S1 S1 )(S2 S2 )(S1 S1 ) = (S2 S2 )(S1 S1 )(S2 S2 ) ?

Problem 21. Consider the matrices

1 0 0 0 1 3 3 1

1 1 0 0 0 1 2 1
A= , B= .
1 2 1 0 0 0 1 1
1 3 3 1 0 0 0 1

Is
AB 1 A = B 1 AB 1 ?
264 Problems and Solutions

Problem 22. Consider the braid group Bn . Let e1 , e2 , . . . , en be the standard


basis in Rn . Then u Rn can be written as
n
X
u= ck e k .
k=1

Consider the linear operators Bj (j = 1, 2, . . . , n1) in Rn (, , , R, , 6=


0) defined by

Bj u := c1 e1 + + (cj+1 + )ej + (cj + )ej+1 + + cn en

and the corresponding inverse operator Bj1

1 1
Bj1 u = c1 e1 + + (cj+1 )ej + (cj )ej+1 + + cn en .

Show that the linear operators Bj satisfy the braid condition

Bj Bj+1 Bj = Bj+1 Bj Bj+1

if + = + .

Problem 23. Let Bn denote the braid group on n strands. Bn is generated


by the elementary braids { b1 , b2 , . . . , bn1 } with the braid relations

bj bj+1 bj = bj+1 bj bj+1 , 1 j < n 1,

bj bk = bk bj , |j k| 2.
Let { e1 , e2 , . . . , en } denote the standard basis in Rn . Then u Rn can be
written as
Xn
u= ck ek , c1 , c2 , . . . , cn R.
k=1

Consider the operators Bj (, , , R and , 6= 0) defined by

Bj u := c1 e1 + . . . + (cj+1 + )ej + (cj+1 + )ej+1 + . . . + cn en

and the corresponding inverse operation


1 1
Bj1 u := c1 e1 + . . . + (cj+1 )ej + (cj )ej+1 + . . . + cn en .

Use computer algebra to show that B1 , B2 , . . . , Bn1 saitisfy the braid condition

Bj Bj+1 Bj u = Bj+1 Bj Bj+1 u

if
+ = + .
Braid Group 265

Problem 24. Consider the braid group B3 with the generators {1 , 2 } and
the relation
1 2 1 = 2 1 2 .
Let t 6= 0. Show that a matrix representation is given by
   
t 1 1 0
1 = , 2 =
0 1 t t

with    
1/t 1/t 1 0
11 = , 21 = .
0 1 1 1/t
Let
= 1 2 11 21 11 .
Find
f (t) = det( I2 ).
Find minima and maxima of f .

Problem 25. The free group, 2 , with two generators g1 and g2 admits the
matrix representation
   
1 0 1 2
g1 = , g2 = .
2 1 0 1

(i) Find the inverse of the matrices.


(ii) Calculate g1 g2 g1 and g2 g1 g2 . Discuss.

Problem 26. Find the conditions on a, b, c, d, e, f R such that


       
a b d e a b d e a b d e
= .
0 c 0 f 0 c 0 f 0 c 0 f

Find nontrivial solutions to these conditions.


Chapter 21

vec Operator

Problem 1. Consider the 2 3 matrix


 
a11 a12 a13
A= .
a21 a22 a23

Let B = AT . Thus B is a 3 2 matrix. Find the 6 6 permutation matrix P


such that
vec(B) = P vec(A).

Problem 2. Let A be an m n matrix over C and B be a s t matrix over


C. Find the permutation matrix P such that

vec(A B) = P (vec(A) vec(B)).

Problem 3. Let A be an m n matrix over C. Using


n
X n
X
vecmn A := ej,n (Aej,n ) = (In A) ej,n ej,n
j=1 j=1

and
m X
n

X
vec1
mn x = (ej,n ei,m ) x ei,m ej,n .
i=1 j=1

Show that
vec1
mn (vecmn A) = A.

266
vec Operator 267

Problem 4. Let A be an m n matrix over C and B be a s t matrix over


C. Show that
A B = vec1
msnt (LA,st (vecst B))

where
n
X
LA,st := (In It A Is ) ej,n It ej,n Is .
j=1

Problem 5. (i) Let AX + XB = C, where C is an m n matrix over R. What


are the dimensions of A, B, and X?
(ii) Solve the equation

  1 1 1  
0 1 1 0 1
X +X 1 1 1 =

1 0 0 1 0
1 1 1
for the real valued matrix X.

Problem 6. Let A be an m n matrix over C and B be an s t matrix over


C. Define
R(A B) := vecA (vecB)T .
Find an algebraic expression for R. Find

  1
0 1
R 2 .
1 0
3
R is the reshaping operator.

Problem 7. Show that

tr(ABCD) (vec(DT ))(A C T )vec(B T ).

Problem 8. (i) Let K be a given n n nonnormal matrix over C. We want


to find all unitary n n matrices such that

U KU = K .

Now we can write KU = U K and therefore

KU U K = 0n

where 0n is the n n zero matrix. This is a matrix equation (a special case of


Sylvester) and using the vec-operator and Kronecker product we can cast the
matrix equation into a vector equation. Write down this linear equation.
268 Problems and Solutions

(ii) Apply it to the case n = 2 with


 
i i
K= .
0 i

Problem 9. Let A, B be n n matrices. Show that

vec(A B) = vec(A) vec(B)

where denotes the Hadamard product.

Problem 10. Let A, B be n n matrices over C. Show that

tr(vec(B)(vec(A))T ) = tr(AT B).

Problem 11. Let A, B be n n matrices over C. Consider the maps

f1 (A, B) = A B
f2 (A, B) = vec(A)(vec(B))T
f3 (A, B) = vec(A)(vec(B T )T
f4 (A, B) = vec(B)(vec(AT ))T .

Is

tr(f1 (A, B)) = tr(f2 (A, B)), tr(f2 (A, B)) = tr(f3 (A, B)), tr(f3 (A, B)) = tr(f4 (A, B)).

Problem 12. Let A, B be n n matrices over C. Show that

tr(vec(B)(vec(A))T ) tr(AT B).

Problem 13. Let A be an 2 2 matrix.


(i) Find vec(A I2 + I2 A).
(ii) Find vec(A A).
(iii) Find the condition on A such that

vec(A A) = vec(A I2 + I2 A).


Chapter 22

Star Product

Consider the 2 2 matrices


   
a11 a12 b11 b12
A= , B= .
a21 a22 b21 b22
We define the composition (star product)
b11 0 0 b12

0 a11 a12 0
A ? B := .
0 a21 a22 0
b21 0 0 b22
The extension to 4 4 matrix is: Let
a11 a12 a13 a14 b11 b12 b13 b14

a21 a22 a23 a24 b21 b22 a23 b24
A= , B= .
a31 a32 a33 a34 b31 a32 a33 b34
a41 a42 a43 a44 b41 b42 b43 b44
Then A ? B is the 8 8 matrix
b11 b12 0 0 0 0 b13 b14

b21 b22 0 0 0 0 b23 b24
0 0 a11 a12 a13 a14 0 0

0 0 a21 a22 a23 a24 0 0

.
0 0 a31 a32 a33 a34 0 0

0 0 a41 a42 a43 a44 0 0

b31 b32 0 0 0 0 b33 b34

b41 b42 0 0 0 0 b43 b44

269
270 Problems and Solutions

Problem 1. (i) What can be said about the trace of A ? B? What can be said
about the determinant of A ? B?
(ii) Let A1 , A2 , A3 , A4 be a basis in the vector space of 2 2 matrices over C.
Let B1 , B2 , B3 , B4 be a basis in the vector space of 2 2 matrices over C. Do
the 16 matrices Aj ? Bk (j, k = 1, 2, 3, 4) form a basis in the vector space of 4 4
matrices?
(iii) Given the eigenvalues of A and B. What can be said about the eigenvalues
of A ? B?
(iv) Can one find 4 4 permutation matrices P and Q such that
P (A ? B)Q = A B ?
Here denotes the direct sum

Problem 2. Consider the 2 2 matrices A, B over C


   
a11 a12 b11 b12
A= , B= .
a21 a22 b21 b22
Let A ? B be the star product.
(i) Answer the following questions: Let A and B be normal matrices. Is A ? B
normal. Let A and B be invertible matrices. Is A ? B an invertible matrix?
Let A and B be unitary matrices. Is A ? B a unitary matrix? Let A and B be
nilpotent matrices. Is A ? B a nilpotent matrix? Answer these questions also for
A ? A.
(ii) What is the conditions on A and B such that
A?B =AB?

Problem 3. Let A, B be 2 2 matrices. We define


a11 0 0 a12

0 b11 b12 0
A ? B := .
0 b21 b22 0
a21 0 0 a22
Can one find a permutation matrix such that
a11 a12 0 0

a21 a22 0 0 T
= P (A ? B)P .
0 0 b11 b12

0 0 b21 b22

Problem 4. (i) Let A, B be 2 2 matrices. Let A ? B be the star product.


The 2 2 matrices
   
1 0 0 1
E= , N=
0 1 1 0
Star Product 271

form a group under matrix multiplication. Do the four 4 4 matrices A ? A,


A ? B, B ? A, B ? B form a group under matrix multiplication?
(ii) Let G be a finite group represented by 2 2 matrices. Let the order be
n with the group elements g1 = e, g2 , . . . , gn . Do the 4 4 matrices gj ? gk
(j, k = 1, . . . , n) form a group under matrix multiplication.

Problem 5. Let A, B be 2 2 matrices. Let A ? B be the star product. Show


that one can find a 4 4 permutation matrix P such that
a11 a12 0 0

a a22 0 0
P (A ? B)P T = 21 .
0 0 b11 b12
0 0 b21 b22

Problem 6. Let A, B be invertible 2 2 matrices. Let A ? B be the star


product. Is A ? B invertible?

Problem 7. (i) The 2 2 matrices


   
1 0 0 1
E= , N=
0 1 1 0
form a group under matrix multiplication. Do the four 4 4 matrices A ? A,
A ? B, B ? A, B ? B form a group under matrix multiplication?
(ii) Let G be a finite group represented by 2 2 matrices. Let the order be
n with the group elements g1 = e, g2 , . . . , gn . Do the 4 4 matrices gj ? gk
(j, k = 1, . . . , n) form a group under matrix multiplication.

Problem 8. Among others one can form a 44 matrix from two 22 matrices
A and B using the direct sum A B, the Kronecker product A B and the star
product
a11 0 0 a12

0 b11 b12 0
A ? B := .
0 b21 b22 0
a21 0 0 a22
Given the eigenvalues and eigenvectors of A and B. What can be said about the
eigenvalues and eigenvectors of A B, A B, A ? B?

Problem 9. (i) Let A, B be invertible 2 2 matrices. Is A ? B invertible?


(ii) Let U and V be elements of SU (2). Is U ? V an element of SU (4)?
(iii) Let X and Y be elements of SL(2, R). Is X ? Y an element of SL(4, R)?

Problem 10. Let A, B be normal 2 2 matrices with eigenvalues 1 , 2 and


1 , 2 , respectively. What can be said about the eigenvalues of A ? B B ? A?
272 Problems and Solutions

Problem 11. (i) Given the eigenvalues of A and B. What can be said about
the eigenvalues of A ? B?
(ii) Can one find 4 4 permutation matrices P and Q such that

P (A ? B)Q = A B ?

Here denotes the direct sum

Problem 12. Let


   
A1 A2 B1 B2
A= , B=
A3 A4 B3 B4

where Aj , Bj (j = 1, 2, 3, 4) are 2 2 matrices. We define the product

A1 02 02 A2

0 B1 B2 02
A ? B := 2
02 B3 B4 02

A3 02 02 A4
where 02 is the 2 2 zero matrix. Thus A ? B is an 8 8 matrix.
(i) Assume that A and B are invertible. Is A ? B invertible?
(ii) Assume that A, B are unitary. Is A ? B unitary?

Problem 13. Let A, B be the 4 4 matrices


   
A1 A2 B1 B2
A= , B=
A3 A4 B3 B4

where Aj , Bj (j = 1, 2, 3, 4) are 2 2 matrices. We define the product

A1 02 02 A2

02 B1 B2 02
A ? B :=
02 B3 B4 02

A3 02 02 A4
where 02 is the 2 2 zero matrix. Thus A ? B is an 8 8 matrix.
(i) Assume that A and B are invertible. Is A ? B invertible?
(ii) Assume that A, B are unitary. Is A ? B unitary?

Problem 14. Let A, B be 3 3 matrices. We define the composition


a11 0 a12 0 a13

0 b11 b12 b13 0
AB := a21 b21 a22 b22 b23 a23 .

0 b31 b32 b33 0

a31 0 a32 0 a33
Star Product 273

Let
1/ 2 0 1/ 2
M = 0 1 0 .
1/ 2 0 1/ 2
Find the eigenvalues of M and M M .

Problem 15. Let P and Q be 2 2 projection matrices. Is the 4 4 matrix


P ? Q a projection matrix? Apply it to P ? P where
 
1 1 1
P = .
2 1 1

Problem 16. Let A and B be 2 2 matrices and A ? B the star product. Let
1 , 2 be the eigenvalues of A and 1 , 2 be the eigenvalues of B, respectively.
What can be said about the eigenvalues of A ? B? Apply the result to the matrix

cosh() 0 0 sinh()

0 cos() sin() 0
.

0 sin() cos() 0

sinh() 0 0 cosh()

Problem 17. Among others one can form a 4 4 matrix from two 2 2
matrices A and B using the direct sum A B, the Kronecker product A B
and the star product A ? B. Given the eigenvalues and eigenvectors of A and
B. What can be said about the eigenvalues and eigenvectors of A B, A B,
A ? B?

Problem 18. The matrix


 
cos() sin()
A() =
sin() cos()

admits the eigenvalues + () = ei and () = ei with the corresponding


normalized eigenvectors
   
1 1 1 1
, .
2 i 2 i
Let
cos() 0 0 sin()

0 cos() sin() 0
A() ? A() = .

0 sin() cos() 0
sin() 0 0 cos()
274 Problems and Solutions

Find the eigenvalues and normalized eigenvectors of A() ? A().

Problem 19. Consider the star product A ? B of two 2 2 matrices A and B


and the product
b11 0 0 b12

b 0 0 b22
A  B = 21 .
0 a11 a12 0
0 a21 a22 0
Is there a 4 4 permutation matrix P such that P (A ? B)P T = A  B?

Problem 20. Let A be a 3 3 matrix and B be a 2 2 matrix. We define


b11 0 0 0 b12

0 a11 a12 a13 0
A ? B := 0 a21 a22 a23 0 .

0 a31 a32 a33 0

b21 0 0 0 b22

Let  
1 1 1
A= , B = I3 .
2 1 1
Find A ? B and calculate the eigenvalues and normalized eigenvectors of A ? I3 .

Problem 21. The star product of two 4 4 matrices A, B is defined by

a11 a12 0 0 0 0 a13 a14



a21 a22 0 0 0 0 a23 a24
0 0 b11 b12 b13 b14 0 0

0 0 b21 b22 b23 b24 0 0

A ? B := .
0 0 b31 b32 b33 b34 0 0
0 0 b41 b42 b43 b44 0 0

a31 a32 0 0 0 0 a33 a34

a41 a42 0 0 0 0 a43 a44

(i) Show that tr(A ? B) = tr(A) + tr(B).


(ii) Show that det(A ? B) = det(A) det(B).
Chapter 23

Nonnormal Matrices

A square matrix M over C is called normal if

M M = M M.

A square matrix M over C is called nonnormal if

M M 6= M M.

Examples for nonnormal matrices are

0 1 1 1

  0 1 1
0 1 0 0 1 1
, 0 0 1, .
0 0 0 0 0 1

0 0 0
0 0 0 0

If M is a nonnormal matrix, then M and M T are nonnormal matrices. If M


is nonnormal and invertible, then M 1 is nonnormal. If M is nonnormal, then
M + M , M M , the commutator [M, M ] and the anti-commutator [M, M ]
are normal matrices.

275
276 Problems and Solutions

Problem 1. Let A be a nonnormal n n matrix over C.


(i) Show that A + A is a normal matrix.
(ii) Show that AA is a normal matrix.

Problem 2. (i) Let R. Consider the matrix


 
0 sin()
A() = .
cos() 0

(i) Is the matrix A() nonnormal for all ?


(ii) What is the condition on that the matrix A() is invertible?
(iii) Is the matrix A() A() nonnormal for all ?

Problem 3. Let A be a non-zero 2 2 matrix with A2 = 02 . Are these


matrices nonnormal?

Problem 4. (i) Let R. Consider the matrix


 
0 sinh()
A() = .
cosh() 0

(i) Is the matrix A() non-normal for all ?


(ii) What is the condition on that the matrix A() is invertible?

Problem 5. Let z1 , z2 , z3 C. Consider the 2 2 matrix


 
z1 z2
A(z1 , z2 , z3 ) = .
z3 0

What is the condition on z1 , z2 , z3 such that A(z1 , z2 , z3 ) is a normal matrix?

Problem 6. (i) Is A A a normal matrix, where denotes the Hadamard


product?
(ii) Let U be a unitary n n matrix. Show that U AU is also nonnormal.
(iii) Is the commutator [A, A ] a normal matrix? Is the anti-commutator [A, A ]+
a normal matrix?
(iv) Is A A a normal matrix, where denotes the Kronecker product?
(v) Is A A a normal matrix, where denotes the direct sum?
(vi) Let A also be invertible. Is A1 also nonnormal?
(vii) Is exp(A) nonnormal?
(viii) Is sinh(A) nonnormal?
(ix) Is A2 nonnormal?
Nonnormal Matrices 277

Problem 7. Let sj := 2 sin(2j/5) with j = 1, 2, . . . , 5. Consider the 5 5


matrix
s1 1 0 0 1

1 s2 1 0 0
M = 0 1 s3 1 0 .

0 0 1 s4 1

1 0 0 1 s5
Show that the matrix is nonnormal. Find the eigenvalues and eigenvectors. Is
the matrix diagonalizable?

Problem 8. Let  6= 0. Show that the matrix


 
1 
A=
0 1

is nonnormal. Give the eigenvalues and eigenvectors.

Problem 9. Let a > 0, b 0 and [0, ]. What are the conditions a, b,


such that  
0 a
A(a, b, ) =
ei b 0

is a normal matrix?

Problem 10. Let A, B be nonnormal matrices. Is AB nonnormal? Is AB


nonnormal?

Problem 11. Can we conclude that an invertible matrix is normal?

Problem 12. Show that all non-diagonalizable matrices are nonnormal.

Problem 13. Show that not all nonnormal matrices are non-diagonalizable.

Problem 14. Find all 2 2 matrices over C which are nonnormal but diago-
nalizable.

Problem 15. Let A be a nonnormal invertible matrix. How do we construct


a matrix B such that A = exp(B). Study first the two examples
   
1 1 2 1
A1 = , A2 = .
0 1 0 1/2

Both are elements of the Lie group SL(2, R).


278 Problems and Solutions

Problem 16. Let a11 , a22 ,  R. What is the condition on a11 , a22 ,  such
that the matrix  
a11 e
A=
e a22
is normal?

Problem 17. Let A be a nonnormal matrix. Is [A, A ] a normal matrix? Is


[A, A ]+ a nonnormal matrix?

Problem 18. Let A be a normal n n matrix and let S be an invertible


nonnormal matrix. Is SAS 1 a normal matrix? Study the case that
   
1 1 1 1 1
A= , S= .
2 1 1 0 1

Of course the eigenvalues of A and SAS 1 are the same.

Problem 19. Consider the invertible nonnormal matrix


   
1 1 1 1
A= A1 = .
0 1 0 1

Is the matrix A A1 normal?

Problem 20. Consider the nonnormal matrix


 
0 1
A= .
0 0

Find sinh(A) and sinh1 (A) = arcsinh(A).

Problem 21. Let A, B be nonzero n n hermitian matrices. We form the


matrix K as
K := A + iB.
This matrix is nonnormal if [A, B] 6= 0 and normal if AB = BA. In the following
we assume that K is nonnormal. Assume that we can find a unitary nn matrix
U such that
U KU = K .
What can be said about the eigenvalues of K? In physics such a unitary matrix
is called a quasi-symmetry operator.

Problem 22. Consider the nonnormal matrix


 
0 i
A= .
0 0
Nonnormal Matrices 279

Find a unitary matrix U such that U AU = A .

Problem 23. Consider the nonnormal matrix


 
i i
K= .
0 i

Find a unitary matrix U such U KU = K . Find the eigenvalues and normalized


eigenvectors of K and K . Discuss.

Problem 24. Can one find a 2 2 matrix A such that


 
2 0 1
A = ?
0 0

Hint. The condition provides four equations. Since the determinant of the
matrix of the right-hand side is equal to 0, the determinant of A, i.e. det(A) =
a11 a22 a12 a21 must also be 0.

Problem 25. Can one find a 2 2 matrix A such that


 
1 1
A2 = ?
0 1

Hint. The condition provides four equations. Since the determinant of the
matrix of the right-hand side is equal to 1, the determinant of A, i.e. det(A) =
a11 a22 a12 a21 must be +1 or 1.

Problem 26. Let A, B be normal matrices. Can we conclude that AB is a


normal matrix?

Problem 27. (i) Consider the nonnormal 2 2 matrix


 
0 1
A= .
0 0

Can we find a 2 2 matrix X such that X 2 = A, i.e. X would be the square


root of A.
(ii) Consider the nonnormal 3 3 matrix

0 1 1
B = 0 1 1.
0 0 0

Can we find a 3 3 matrix Y such that Y 2 = B, i.e. Y would be the square


root of B.
280 Problems and Solutions

Problem 28. Let A be an normal n n matrix. Is the matrix A iIn normal?

Problem 29. Find all nonnormal 2 2 matrices A such that


AA + A A = I2 .
An example is  
0 1
A= .
0 0

Problem 30. (i) Can one find a nonormal matrix A such that

eA eA = eA eA ?
(ii) Can one find a nonormal matrix A such that

eA eA = eA+A ?

Problem 31. Let s {1/2, 1, 3/2, 2, . . .} be the spin quantum number. The
spin matrices s1 and s3 are defined as the (2s + 1) (2s + 1) matrices s1 =
(s+ + s )/2 and s3 = diag(s, s 1, . . . , s) where the entries of s+ and s are
all zero except for the entries given by (here rows and columns are numbered
s, s 1, s 2, . . . , s)
p
(s+ )m+1,m = (s m)(s + m + 1) m = s 1, s 2, . . . , s
and s = sT+ .
2
Calculate
p [s1 , s3 ] in terms of s. Find the k[s1 , s3 ]k in terms of s, where kAk :=
tr(AA ) is the Frobenius norm.

Problem 32. A measure of nonnormality is given by


m(A) := kA A AA k
where kk denotes some matrix norm. Let s3 and s1 be real valued and symmetric
n n matrices (for example, spin matrices). Calculate m(s3 + exp(i)s1 ).

Problem 33. Let A be a nonnormal matrix, i.e. AA 6= A A


(i) Is tr(AA ) = tr(A A)?

(ii) Is treAA = treA A ?

Problem 34. Consider the non-normal matrix


 
i 1
A= .
0 i
Nonnormal Matrices 281

Is
eA+A = eA eA ?

Problem 35. Let Q be a non-normal invertible n n matrix. Is

Q Q1

non-normal?

Problem 36. Let , C. What are the conditions on , such that the
matrix  

M (, ) =
0 1
is normal?

Problem 37. Let M be an n n nonnormal matrix.


(i) Is
M
 
0n
M 0n
nonnormal?
(ii) Is
M
 
0n
M 0n
nonnormal?

Problem 38. Let z C. Is the matrix


 
1 0
1 ez ez

nonnormal?

Problem 39. Prove or disprove the following statements.


(i) If the n n matrix A is nonnormal, then there exists no matrix B such that
B 2 = A.
(ii) Let A, B be nonzero n n matrices with B 2 = A. Then the matrix A is
normal.
(iii) An n n nonnormal matrix is not diagonalizable.

Problem 40. We know that all real symmetric matrices are diagonalizable.
Are all complex symmetric matrices are diagonalizable?

Problem 41. An n n matrix A is called nonnormal if AA 6= A A.


282 Problems and Solutions

e = U AU 1 nonnormal for all U .


(i) Let U be a unitary matrix. Is A
e = QAQ1 is normal?
(ii) Let Q SL(n, C). Can one find Q such that A

Problem 42. Is every invertible matrix normal? Prove or disprove.

Problem 43. Let R. Find the condition on such that


 
0 cos()
sin() 0

is a normal matrix.

Problem 44. Let s R. For which values of s is the matrix



1 + is is is
A(s) = is is is
is is 1 + is

nonnormal.
Chapter 24

Spectral Theorem

Problem 1. Let A be a normal matrix with eigenvalues 1 , . . . , n and


pairwise orthonormal eigenvectors aj (column vectors), i.e. aj ak = jk . Then
we can write A as (spectral decomposition)
n
X
A= j aj aj .
j=1

Analogously for a normal matrix B we have


n
X
B= k bk bk .
k=1

(i) Find the condition on j , aj and k , bk such that tr(AB ) = 0, i.e. the two
matrices are orthogonal to each other.
(ii) Find the condition on j , aj and k , bk such that [A, B] = 0n , i.e. the the
commutator of the matrices vanishes.

Problem 2. Let A be a normal matrix with eigenvalues 1 , . . . , n and


pairwise orthonormal eigenvectors aj (column vectors), i.e. aj ak = jk . Then
we can write A as (spectral decomposition)
n
X
A= j aj aj .
j=1

Analogously for a normal matrix B we have


n
X
B= k bk bk .
k=1

283
284 Problems and Solutions

Let z C. Use the spectral decomposition to calculate


ezA BezA .

Problem 3. Let A be an n n normal matrix over C with eigenvalues 1 ,


. . . , n and corresponding pairwise orthonormal eigenvectors uj (j = 1, . . . , n).
Then the matrix A can be written as (spectral decomposition)
n
X n
X
A= j uj uj j |uj ihuj |.
j=1 j=1

(i) Let z C. Use this spectral decomposition to calculate exp(zA).


(ii) Apply it to A = 1 .

Problem 4. (i) Consider the permutation matrix


 
0 1
P = .
1 0
Find the eigenvalues and normalized eigenvectors and thus the spectral decom-
position of P .
(ii) Find the matrix X such that exp(X) = P .

Problem 5. (i) Consider the permutation matrix



0 0 1
P = 0 1 0.
1 0 0
Find the eigenvalues and normalized eigenvectors of P and thus the spectral
decomposition of P .
(ii) Find the matrix X such that exp(X) = P .

Problem 6. Consider the two 3 3 permutation matrices (which are of course


then also unitary matrices)

0 1 0 0 0 1
U1 = 0 0 1 , U2 = 1 0 0 .
1 0 0 0 1 0

We want to find efficiently K1 and K2 such that U1 = eK1 and U2 = eK2 . We


would apply the spectral decomposition theorem to find K1 , i.e.
3
X
K1 = ln(j )vj vj
j=1
Spectral Theorem 285

where j are the eigenvalues of U1 and vj are the corresponding normalized


eigenvectors. But then to find K2 we would apply the property that U12 = U2 . Or
could we actually apply that U2 = U1T ? Note that U1 , U2 , I3 form a commutative
subgroup of the group of 3 3 permutation under matrix multiplication.

Problem 7. Let A be a hermitian nn matrix. Assume that all the eigenvalues


1 , . . . , n are pairwise different. Then the normalized eigenvectors uj (j =
1, . . . , n) satisfy uj uk = 0 for j 6= k and uj uj = 1. We have (spectral theorem)
n
X
A= j uj uj .
j=1

Let ek (k = 1, . . . , n) be the standard basis in Cn . Calculate U AU , where


n
X
U= uk ek .
k=1

Problem 8. Let A be a positive definite nn matrix. Thus all the eigenvalues


are real and positive. Assume that all the eigenvalues 1 , . . . , n are pairwise
different. Then the normalized eigenvectors uj (j = 1, . . . , n) satisfy uj uk = 0
for j 6= k and uj uj = 1. We have (spectral theorem)
n
X
A= j uj uj .
j=1

Let ek (k = 1, . . . , n) be the standard basis in Cn . Calculate

ln(A).

Note that the unitary matrix


n
X
U= uk ek
k=1

e = U AU is a diagonal matrix.
transforms A into a diagonal matrix, i.e. A

Problem 9. The spectral theorem for n n normal matrices over C is as


follows: A matrix A is normal if and only if there exists an n n unitary matrix
U and a diagonal matrix D such that D = U AU . Use this theorem to prove
that the matrix
0 1 1
A = 0 0 1
0 0 0
286 Problems and Solutions

is not normal.

Problem 10. Consider the normal matrix



0 1 0
A = 1 0 1.
0 1 0
Find the eigenvalues and normalized eigenvectors of A and thus the spectral
decomposition. Thus this result to calculate exp(zA), where z C.

Problem 11. (i) Find the spectral decomposition of the normal matrix

0 1 0
A = 1 0 1.
0 1 0
(ii) Find the spectral decomposition of exp(A).

Problem 12. (i) Find the eigenvalues, normalized eigenvectors and spectral
decomposition of the permutation matrices

0 1 0 0 0 1
P1 = 0 0 1 , P2 = 1 0 0 .
1 0 0 0 1 0
(ii) Use the spectral decomposition to find the matrices A1 and A2 such that
P1 = exp(A1 ), P2 = exp(A2 ).

Problem 13. Consider the permutation matrix


0 0 0 1

0 0 1 0
P = .
1 0 0 0
0 1 0 0
(i) Find the eigenvalues and normalized eigenvectors of P .
(ii) Show that the eigenvalues form a group under multiplication.
(iii) Use the result from (i) and the spectral representation to find K such that
P = exp(K).

Problem 14. Let A be an n n matrix over C. An n n matrix B is called


a square root of A if B 2 = A. Find the square roots of the 2 2 identity matrix
applying the spectral theorem. The eigenvalues of I2 are 1 = 1 and 2 = 1. As
normalized eigenvectors choose
 i   i 
e 1 cos() e 1 sin()
,
ei2 sin() ei2 cos()
Spectral Theorem 287


which form an orthonormal
in C2
basis . Four cases ( 1 , 2 ) = (1, 1), ( 1 , 2 ) =
(1, 1), ( 1 , 2 ) = (1, 1), ( 1 , 2 ) = (1, 1) have to be
studied. The
first and last case are trivial. So study the second case ( 1 , 2 ) = (1, 1).
The second case and the third case are equivalent.

Problem 15. Consider the vectors in C2 (sometimes called spinors)


cos(/2)ei/2 sin(/2)ei/2
   
v1 = , v2 = .
sin(/2)ei/2 cos(/2)ei/2
(i) First show that they are normalized and orthonormal.
(ii) Assume that for the vector v1 is an eigenvector with the corresponding
eigenvalue +1 and that the vector v2 is an eigenvector with the corresponding
eigenvalue 1. Apply the spectral theorem to find the corresponding 2 2
matrix.
(iii) Since the vectors v1 and v2 form an orthonormal basis in C2 we can form
an orthonormal basis in C4 via the Kronecker product
w11 = v1 v1 , w12 = v1 v2 , w21 = v2 v1 , w22 = v2 v2 .
Assume that the eigenvalue for w11 is +1, for w12 1, for w21 1 and for w22
+1. Apply the spectral theorem to find the corresponding 4 4 matrix.

Problem 16. Given a normal 5 5 matrix which provides the characteristic


equation
5 + 43 3 = 0
with the eigenvalues

1 = 3, 2 = 1, 3 = 0, 4 = 1, 5 = 3
and the corresponding normalized eigenvectors

3/6 1/2 1/ 3 1/2 3/6

1/2 1/2 0 1/2 1/2


v1 = 1/ 3 , v2 = 0 , v3 = 1/ 3 , v4 = 0 , v5 = 1/ 3 ,

1/2 1/2 0 1/2 1/2


3/6 1/2 1/ 3 1/2 3/6
Reconstruct the matrix applying the spectral theorem.

Problem 17. Let A be an n n matrix over C. An n n matrix B is called


a square root of A if B 2 = A. Find the square root of the 2 2 identity matrix
I2 applying the spectral theorem. Hint. An orthonormal basis in C2 which are
also eigenvectors of the 2 2 identity matrix is given by
 i   i 
e 1 cos() e 1 sin()
v1 = , v 2 = .
ei2 sin() ei2 cos()
288 Problems and Solutions

Thus I2 = 1 v1 v1 + 2 v2 v2 with 1 = 1 and 2 = 1. Note that since 1 = 1
we have to consider the four cases
p p p p p p p p
( 1 , 2 ) = (1, 1), ( 1 , 2 ) = (1, 1), ( 1 , 2 ) = (1, 1), ( 1 , 2 ) = (1, 1).

Problem 18. Consider the unitary matrix (Hadamard gate)


 
1 1 1 1
UH = (3 + 1 )
2 1 1 2
with eigenvalues +1 and 1. Find the square
roots of this matrix applying
the spectral theorem. Since 1 = 1 and 1 = i four cases have to be
considered
(1, i), (1, i), (1, i), (1, i).

Problem 19. The star product of the Hadamard gate with itself provides the
Bell matrix
1 0 0 1

1 0 1 1 0
UH ? UH = .
2 0 1 1 0
1 0 0 1
Use the result from the previous problem to find the square roots of the Bell
matrix. Note that the eigenvalues of the Bell matrix are +1 (twice) and 1
(twice).
Chapter 25

Mutually Unbiased Bases

Two orthonormal basis in the Hilbert space Cd

A = { e1 , . . . , ed }, B = { f1 , . . . , fd }

are called mutually unbiased if for every 1 j, k d


1
|hej , fk i| = .
d
Problem 1. For the Pauli spin matrices 3 , 1 , 2 with eigenvalues +1 and
1 the normalized eigenvectors are given by
   
1 0
B1 = , ,
0 1
    
1 1 1 1
B2 = , ,
2 1 2 1
    
1 1 1 1
B3 = , .
2 i 2 i
This is a set of three mutually unbiased basis.
(i) Consider now 3 3 , 1 1 , 2 2 . Show that
               
1 1 1 0 0 1 0 0
B1 = , , ,
0 0 0 1 1 0 1 1
                
1 1 1 1 1 1 1 1 1 1
B2 = , , ,
2 1 1 2 1 1 1 1 1 1

289
290 Problems and Solutions
                
1 1 1 1 1 1 1 1 1 1 1 1
B3 = , , ,
2 i i 2 i i 2 i i 2 i i
provides mutually unbiased basis in the Hilbert space C4 .
(ii) Consider the case in the Hilbert space C8 with

3 3 3 , 1 1 1 , 2 2 2 .

Extend the n Kronecker products.

Problem 2. (i) Let H1 and H2 be two finite dimensional Hilbert spaces


with dim(H1 ) = d1 and dim(H2 ) = d2 . Let B1,1 = {|j1 i} and B1,2 = {|j2 i}
(j1 , j2 = 1, . . . , d1 ) be two orthonormal bases in the Hilbert space H1 . They are
called mutually unbiased iff
1
|hj1 |j2 i|2 = for all j1 , j2 = 1, . . . , d1 .
d1

Let B2,1 = {|k1 i} and B2,2 = {|k2 i} (k1 , k2 = 1, . . . , d2 ) be orthonormal bases in


the Hilbert space H2 . Assume in the following that B1,1 and B1,2 are mutually
unbiased bases in the Hilbert space H1 and B2,1 and B2,2 are mutually unbiased
bases in the Hilbert space H2 , respectively. Show that

{ |j1 i |k1 i }, { |j2 i |k2 i }, j1 , j2 = 1, . . . , d1 , k1 , k2 = 1, . . . , d2

are mutually unbiased bases in the finite dimensional product Hilbert space
H1 H2 with dim(H1 H2 ) = d1 d2 and the scalar product in the product
Hilbert space
(hj1 | hk1 |)(|j2 i |k2 i = hj1 |j2 ihk1 |k2 i.
(ii) Apply the result from (i) to H1 = C2 with
   
1 0
B1,1 = { |1i1,1 = , |2i1,1 = }
0 1
   
1 1 1 1
B1,2 = { |1i1,2 = , |2i1,2 = }
2 1 2 1
and H2 = C3 with

1 0 0
B2,1 = { |1i2,1 = 0 , |2i2,1 = 1 , |3i2,1 = 0 }
0 0 1

1 1/ 3 1/ 3
1
B2,2 = { |1i2,2 = 1 , |2i2,2 = i/2 1/(2 3) , |3i2,2 = i/2 1/(23) }.
3 1 i/2 1/(2 3) i/2 1/(2 3)
Mutually Unbiased Bases 291

(iii) Consider the Hilbert space M (2, C) of the 2 2 matrices over the complex
numbers and the mutually unbiased bases
       
1 0 0 1 0 0 0 0
, , ,
0 0 0 0 1 0 0 1

and
       
1 1 1 1 1 1 1 1 1 1 1 1
, , , .
2 1 1 2 1 1 2 1 1 2 1 1

Construct mutually unbiased bases applying the Kronecker product for the
Hilbert space M (4, C).

Problem 3. Let d 2. Consider the Hilbert space Cd and e0 , e1 , . . . , ed1


be the standard basis
1 0 0

0 1 0
.
0 0 .

e0 = . , e1 = . , . . . ed1 = . .
.. .. 0
0 0 1

Let d := ei2/d . Then we can form new orthonormal bases via


d1
1 X bn(n1)/2nm
vm;b |m; bi = en , b, m = 0, 1, . . . , d 1
d n=0

where the d labelled by the b are the bases and m labels the state within a basis.
Thus we find mutually unbiased bases.
(i) Consider d = 2 and b = 0. Find the basis |0; 0i, |1; 0i.
(ii) Consider d = 3 and b = 0. Find the basis |1; 0i, |2; 0i, |3; 0i.
(iii) Consider d = 4 and b = 0. Find the basis |0; 0i, |1; 0i, |2; 0i, |3; 0i. Find out
whether the states can be written as Kronecker product of two vectors in C2 ,
i.e. whether the states are entangled or not?
Chapter 26

Integration

Let A be an n n matrix over C. If f : C C is analytic in a region


containing the spectrum of A, then f (A) can be defined through the Cauchy
integral formula Z
1
f (A) = (zIn A)1 f (z)dz.
2i
Problem 1. Let Pj (j = 0, 1, 2, . . .) be the Legendre polynomials
1
P0 (x) = 1, P1 (x) = x, P2 (x) = (3x2 1), . . . .
2
Calculate the infinite dimensional matrix A = (Ajk )
Z +1
dPk (x)
Ajk = Pj (x) dx
1 dx
where j, k = 0, 1, . . .. Consider the matrix A as a linear operator in the Hilbert
space `2 (N0 ). Is kAk < ?

Problem 2. Let CnN be the vector space of all n N complex matrices. Let
Z CnN . Then Z Z T , where T denotes transpose. One defines a Gaussian
measure on CnN by
1
d(Z) := exp(tr(ZZ ))dZ
nN
where dZ denotes the Lebesgue measure on CnN . The Fock space F(CnN )
consists of all entire functions on CnN which are square integrable with respect

292
Integration 293

to the Gaussian measure d(Z). With the scalar product


Z
hf |gi := f (Z)g(Z)d(Z), f, g F(CnN )
CnN

one has a Hilbert space. Show that this Hilbert space has a reproducing kernel
K. This means a continuous function K(Z, Z 0 ) : CnN CnN C such that
Z
f (Z) = K(Z, Z 0 )f (Z 0 )d(Z 0 )
CnN

for all Z CnN and f F(CnN ).

Problem 3. Let A be an n n positive definite matrix over R, i.e. xT Ax > 0


for all x Rn . Calculate Z
exp(xT Ax)dx.
Rn

Problem 4. Consider the matrices


  Z t
cos(t) sin(t)
A(t) = , B(t) = A(s)ds.
sin(t) cos(t) 0

Find the commutator [A(t), B(t)]. Discuss. What is the condition such that
[A(t), B(t)] = 02 .

Problem 5. Let R. Consider an n n matrix A(), where the entries of


A depends smoothly on . Then one has the identity
Z 1 Z 1
d A() (1s)A() dA() sA() dA() (1s)A()
e e e ds esA() e ds.
d 0 d 0 d

Let n = 2 and  
cos() sin()
A() = .
sin() cos()
Calculate the right-hand sides of the identities.
Chapter 27

Differentiation

Consider the m n matrix


f (x)
11 f12 (x) f13 (x) f1n (x)
f21 (x) f22 (x) f23 (x) f2n (x)
F (x) = .. .. .. .. ..

. . . . .

fm1 (x) fm2 (x) fm3 (x) fmn (x)
with fjk are differentiable function of x. Then dF (x)/dx is defined entry-wise.

Problem 1. (i) Consider the 2 2 matrix


 
cos() sin()
F () =
sin() cos()

with det(F ()) = 1. Is the determinant preserved under repeated differentiation


of F () with respect to ?
(ii) Consider the 2 2 matrix
 
cos() sin()
G() =
sin() cos()

with det(G()) = 1. Is the determinant preserved under repeated differentia-


tion of G() with respect to ?

Problem 2. Let j be a positive integer. Let A, B be n n matrices over R.


Calculate
1
lim ((A + B)j Aj ).
0

294
Differentiation 295

Calculate
d
tr(A + B)j =0 .

d

Problem 3. Find the partial differential equation given by the condition



0 u/x1 u/x2
det u/x1 2 u/x21 2 u/x1 x2 .
u/x2 2 u/x2 x1 2 u/x22
Find a nontrivial solution of the partial differential equation.

Problem 4. Consider the 2 2 matrix


 
f1 () f2 ()
A() =
f3 () f4 ()
where fj (j = 1, 2, 3, 4) are smooth functions and det(A()) > 0 for all . Show
that
tr((dA())A()1 ) = d(ln(det(A())))
where d is the exterior derivative.

Problem 5. Let u1 (t), u2 (t), u3 (t) R3 . Solve the initial value problem of
the nonlinear autonomous system of first order differential equations
du1 du2 du3
= u2 u3 , = u3 u1 , = u1 u2
dt dt dt
where denotes the vector product.

Problem 6. Let u(t) R3 . Solve the initial value problem for the differential
equation
d2 u du
=u
dt2 dt
where denotes the vector product.

Problem 7. Let fjk : R R be analytic functions, where j, k = 1, 2. Find


the differential equations for fjk such that
        
f11 () f12 () d f11 () f12 () d f11 () f12 () f11 () f12 ()
= .
f21 () f22 () d f21 () f22 () d f21 () f22 () f21 () f22 ()

Problem 8. Let : R R be an analytic functions. Consider the matrices


 i(t)   
e 1 1 ei(t)
A(t) = , B(t) = .
1 eid(t)/dt eid(t)/dt 1
296 Problems and Solutions

(i) Find the differential equation for from the condition

tr(AB) = 0.

(ii) Find the differential equation for from the condition

det(AB) = 0.

Problem 9. Consider the invertible 2 2 matrix


 
cos(x) sin(x)
A() = .
sin(x) cos(x)

Show that
d(ln(det(A))) = tr(A1 dA)
where d denotes the exterior derivative.

Problem 10. (i) Consider the analytic function f : R2 R2

f1 (x1 , x2 ) = sinh(x2 ), f2 (x1 , x2 ) = sinh(x1 ).

Show that this function admits the (only) fixed point (0, 0). Find the functional
matrix at the fixed point
 
f1 /x1 f1 /x2
.
f2 /x1 f2 /x2 (0,0)

(ii) Consider the analytic function g : R2 R2

g1 (x1 , x2 ) = sinh(x1 ), g2 (x1 , x2 ) = sinh(x2 ).

Show that this function admits the (only) fixed point (0, 0). Find the functional
matrix at the fixed point
 
g1 /x1 g1 /x2
.
g2 /x1 g2 /x2 (0,0)

(iii) Multiply the two matrices found in (i) and (ii).


(iv) Find the composite function h : R2 R2

h(x) = (f g)(x) = f (g(x)).

Show that this function also admits the fixed point (0, 0). Find the functional
matrix at this fixed point
 
h1 /x1 h1 /x2
.
h2 /x1 h2 /x2 (0,0)
Differentiation 297

Compare this matrix with the matrix found in (iii).

Problem 11. (i) Consider the 2 2 matrix


 
cos(t) sin(t)
V (t) = .
sin(t) cos(t)

Calculate dV (t)/dt and then find the commutator [dV (t)/dt, V (t)].
(ii) Let V (t) be a 2 2 matrix where all the entries are smooth functions of
t. Calculate dV (t)/dt and then find the conditions on the entries such that
[dV (t)/dt, V (t)] = 02 .

Problem 12. Let fj (x1 , x2 ) (j = 1, 2, 3) be real valued smooth functions.


Consider the matrix
 
f3 if2 + f1
N (x1 , x2 ) = f1 1 + f2 2 + 3 .
if2 + f1 f3

Find dN , N . Then calculate d(N dN ). Find the conditions of f1 , f2 , f3 such


that
d(N dN ) = 02
where 02 is the 2 2 zero matrix.

Problem 13. Let A be an n n matrix. Assume that the inverse of A exists,


i.e. det(A) 6= 0. Then the inverse B = A1 can be calculated as


ln(det(A)) = bkj .
ajk

Apply this formula to the 2 2 matrix A


 
a11 a12
A=
a21 a22

with det(A) = a11 a22 a12 a21 6= 0.

Problem 14. Consider the four 6 6 matrices

1 0 0 0 0 0 0 0 0 0 0 0

0 1 0 0 0 0 0 0 0 0 0 1
0 0 1 0 0 0 0 0 0 0 1 0

0 = , 1 =
0 0 0 1 0 0 0 0 0 0 0 0

0 0 0 0 1 0 0 0 1 0 0 0

0 0 0 0 0 1 0 1 0 0 0 0
298 Problems and Solutions

0 0 0 0 0 1 0 0 0 0 1 0

0 0 0 0 0 0 0 0 0 1 0 0
0 0 0 1 0 0 0 0 0 0 0 0

2 = , 3 = .
0 0 1 0 0 0 0 1 0 0 0 0
0 0 0 0 0 0 1 0 0 0 0 0

1 0 0 0 0 0 0 0 0 0 0 0
and
T
= ( E1 E2 E3 cB1 cB2 cB3 ) .
Show that
T
0 + 1 + 2 + 3 = (0 0 0 0 0 0)
ct x1 x2 x3
are Maxwells equations
1 E B
= B, = E.
c2 t t
Chapter 28

Hilbert Spaces

Problem 1. The vector space of all nn matrices over C form a Hilbert space
with the scalar product defined by
hA, Bi := tr(AB ).
This implies a norm kAk2 = tr(AA ).
(i) Consider the Lie group U (n). Find two unitary 2 2 matrices U1 , U2 such
that kU1 U2 k takes a maximum.
(ii) Are the matrices
   
0 1 1 1 1
U1 = , U2 =
1 0 2 1 1
such a pair?

Problem 2. Consider the four 2 2 matrices


       
1 1 1 1 0 0 1 1 1 1 0 0
P = , Q= , R= , S= .
2 0 0 2 1 1 2 0 0 2 1 1
(i) Show that they form an orthonormal basis in the Hilbert space of the 2 2
matrices with the scalar product hX, Y i = tr(XY ).
(ii) Find the multiplication table.

Problem 3. Consider the Hilbert space Md (C) of d d matrices with scalar


product hA, Bi := tr(AB ), A, B Md (C). Consider an orthogonal basis of d2
d d hermitian matrices B1 , B2 , . . . , Bd2 , i.e.
hBj , Bk i = tr(Bj Bk ) = djk

299
300 Problems and Solutions

since Bk = Bk for a hermitian matrix. Let M be a d d hermitian matrix. Let

mj = tr(Bj M ) j = 1, . . . , d2 .

Given mj and Bj (j = 1, . . . , d2 ). Find M .

Problem 4. Consider the Hilbert space H of the 2 2 matrices over the


complex numbers with the scalar product

hA, Bi := tr(AB ), A, B H.

Show that the rescaled Pauli matrices j = 1 j , j = 1, 2, 3


2
     
1 0 1 1 0 i 1 1 0
1 = , 2 = , 3 =
2 1 0 2 i 0 2 0 1

plus the rescaled 2 2 identity matrix


 
1 1 0
0 =
2 0 1

form an orthonormal basis in the Hilbert space H.

Problem 5. Can we find an invertible 2 2 matrix S over the real numbers


such that    
0 1 1 1 0
S S = ?
1 0 0 1
Chapter 29

Miscellaneous

Problem 1. (i) For n = 4 the transform matrix for the Daubechies wavelet is
given by

c0 c1 c2 c3 c0 1 + 3

c3 c2 c1 c0 c1 1 3 + 3
D4 = , = .
c2 c3 c0 c1 c2 4 2 3 3
c1 c0 c3 c2 c3 1 3
Is D4 orthogonal? Prove or disprove.
(ii) For n = 8 the transform matrix for the Daubechies wavelet is given by
c0 c1 c2 c3 0 0 0 0

c3 c2 c1 c0 0 0 0 0
0 0 c0 c1 c2 c3 0 0

0 0 c3 c2 c1 c0 0 0

D8 = .
0 0 0 0 c0 c1 c2 c3
0 0 0 0 c3 c2 c1 c0

c2 c3 0 0 0 0 c0 c1

c1 c0 0 0 0 0 c3 c2
Is D8 orthogonal? Prove or disprove.

Problem 2. Consider the 2n 2n matrix


 
0n In
J := .
In 0n
We define that the 2n 2n matrix H over R is Hamiltonian if (JH)T = JH. We
define that the 2n 2n matrix S over R is symplectic if S T JS = J. Show that if
H is Hamiltonian and S is symplectic, then the matrix S 1 HS is Hamiltonian.

301
302 Problems and Solutions

Problem 3. Let A be an n n matrix over R. Consider the 2n 2n matrices


   
In In A + 2In In
S= , Se = .
A In + A In 0n

Can we find an invertible 2n 2n matrix T such that Se = T 1 ST ?

Problem 4. Let
0 0 1 0 1 i 0 0

0 0 0 1 1 0 0 i 1
J = , P = .
1 0 0 0 2 i 1 0 0
0 1 0 0 0 0 1 i
Find P P . Show that P JP is a diagonal matrix.

Problem 5. Let J be the 2n 2n matrix


 
0n In
J := .
In 0n
We define symplectic G-reflectors to be those 2n 2n symplectic matrices that
have a (2n 1)-dimensional fixed-point subspace. It can be shown that any
symplectic G-reflector can be expressed in the form
G = I2n + uuT J (1)
for some 0 6= F, 0 6= u F2n and u is considered as a column vector. The
underlying field is F. Conversely, any G given by (1) is always a symplectic
G-reflector. Show that det(G) = +1.

Problem 6. Consider the two polynomials


p1 (x) = a0 + a1 x + + an xn , p2 (x) = b0 + b1 x + + bm xm
where n = deg(p1 ) and m = deg(p2 ). Assume that n > m. Let
p2 (x)
r(x) = .
p1 (x)
We expand r(x) in powers of 1/x, i.e.
c1 c2
r(x) = + 2 +
x x
From the coefficients c1 , c2 , . . . ,c2n1 we can form an n n Hankel matrix
c
1 c2 c n
c2 c3 cn+1
Hn =
.. .. .. .
..
. . . .
cn cn+1 c2n1
Miscellaneous 303

The determinant of this matrix is proportional to the resultant of the two poly-
nomials. If the resultant vanishes, then the two polynomials have a non-trivial
greatest common divisor. Apply this theorem to the polynomials

p1 (x) = x3 + 6x2 + 11x + 6, p2 (x) = x2 + 4x + 3.

Problem 7. Consider the 3 3 matrix



1/2
3/6 6/3
A = 3/6 5/6 2/3 .
6/3 2/3 1/3

Show that AT = A1 by showing that the column of the matrix are normalized
and pairwise orthonormal.

Problem 8. Find the Moore-Penrose pseudo inverses of



1 0  
0 1, 1
, (1 1).
0
1 0

Problem 9. Let A be an n n matrix over R. Show that there exists nonnull


vectors x1 , x2 in Rn such that

xT1 Ax1 xT Ax xT Ax2


T
T 2T
x1 x1 x x x2 x2

for every nonnull vector x in Rn .

Problem 10. A generalized Kronecker delta can be be defined as follows



1 if J = (j1 , . . . , jr ) is an even permutation of I = (i1 , . . . , ir )
I,J := 1 if J is an odd permutation of I
0 if J is not a permutation of I

Find 126,621 , 126,651 , 125,512 .

Problem 11. Let cj > 0 for j = 1, . . . , n. Show that the n n matrices



cj ck
  
1/cj + 1/ck
,
cj + ck cj ck

(k = 1, . . . , n) are positive definite.


304 Problems and Solutions

Problem 12. Let R Cmm and S Cnn be nontrivial involutions. This


means that R = R1 6= Im and S = S 1 6= In . A matrix A Cmn is called
(R, S)-symmetric if RAS = A. Consider the case m = n = 2 and the Pauli spin
matrices    
0 1 0 i
R= , S= .
1 0 i 0
Find all 2 2 matrices A over C such that RAS = A.

Problem 13. Let X Rnn . Show that X can be written as

X = A + S + cIn

where A is antisymmetric (AT = A), S is symmetric (S T = S) with tr(S) = 0


and c R.

Problem 14. Consider vectors in the vector space R3 and the vector product.
Consider the mapping of the vectors in R3 into 3 3 skew-symmetric matrices

a 0 c b
b c 0 a .
c b a 0

Calculate the vector product



a1 a2
b1 b2
c1 c2

and the commutator [M1 , M2 ], where



0 c1 b1 0 c2 b2
M1 = c1 0 a1 , M2 = c2 0 a2 .
b1 a1 0 b2 a2 0

Discuss.

Problem 15. Let    


z1 w1
z= , w=
z2 w2
be elements of C2 . Solve the equation z w = w z.

Problem 16. Let A, B be n n matrices over C. We define the quasi-


multiplication
1
A B := (AB + BA).
2
Miscellaneous 305

Obviously A B = B A. Show that

(A2 B) A = A2 (B A).

This is called the Jordan identity.

Problem 17. Let j (j = 0, 1, 2, 3) be the Pauli spin matrices, where 0 = I2 .


Does the set of 4 4 matrices
   
j 02 02 j
, , j, k = 0, 1, 2, 3
02 k k 02

form a group under matrix multiplication. If not add the elements to find a
group. Here 02 is the 2 2 zero matrix.

Problem 18. Let A be a symmetric 2 2 matrix over R


 
a00 a01
A= .
a10 a11

Thus a01 = a10 . Assume that

a00 a01 = a201 , a00 a11 = a01 a11 .

Find all matrices A that satisfy these conditions.

Problem 19. Let A, B be 3 3 matrices. We define the composition

a11 0 a12 0 a13



0 b11 b12 b13 0
AB := a21 b21 a22 b22 b23 a23 .

0 b31 b32 b33 0

a31 0 a32 0 a33

Let
1/ 2 0 1/ 2
M = 0 1 0 .
1/ 2 0 1/ 2
Find the eigenvalues of M and M M .

Problem 20. Find a 3 3 matrix A over R which satisfies

A2 AT + AT A2 = 2A, AAT A = 2A, A3 = 0.

Thus the matrix is nilpotent.


306 Problems and Solutions

Problem 21. Find the Cayley transform of the Hermitian matrix


 
h11 h12
H= , h11 , h22 R, h12 C.
h12 h22

Problem 22. Let S be an invertible n n matrix. Find the inverse of the


2n 2n matrix
0n S 1
 

S 0n
where 0n is the n n zero matrix.

Problem 23. Consider the n n matrices


0 1 0 ... 0

0 0 1 ... 0
.
. , B = diag(1 2 . . . n1 )

A=
.
0 0 0 ... 1
1 0 0 ... 0
where is the n-th primitive root of unity. We have An = B n = In and n = 1.
We have
AB = BA.
Let R = A In and S = B In . Find RS. Let X = A A and Y = B B.
Find XY . Find the commutator [X, Y ].

Problem 24. Find all 2 2 matrices C over R such that


C T C + CC T = I2 , C 2 = 02 .

Problem 25. Let j , j R with j = 1, 2, 3. Any 3 3 unitary symmetric


matrix U can be written in the product form
i i
e 1 0 0 1 12 13 e 1 0 0
U = 0 ei2 0 12 2 23 0 ei2 0
i3 i3
0 0 e 13 23 3 0 0 e
where jk = Njk exp(ijk ) with Njk , jk R. It follows that
Ujj = j exp(2ij ), Ujk = Njk exp(i(j + k + jk )).
The unitary condition U U = I3 provides
3
X
2
Njk + j = 1, j = 1, 2, 3
k6=j
Miscellaneous 307

and

N12 (1 exp(i12 ) + 2 exp(i12 )) = N13 N23 exp(i( + 23 13 ))

and cyclic (1 2 3 1). Write the unitary symmetric matrix



0 0 i
W = 0 i 0
i 0 0

in this form.

Problem 26. Consider the map f : C2 7 R3



  sin(2) cos()
cos()
7 sin(2) sin() .
ei sin()
cos(2)

(i) Consider the map for the special case = 0, = 0.


(ii) Consider the map for the special case = /4, = /4.

Problem 27. (i) Consider the hermitian 3 3 matrices to describe a particle


with spin-1

0 1 0 0 i 0 1 0 0
~ ~
S1 := 1 0 1 , S2 := i 0 i , S3 := ~ 0 0 0 .
2 0 1 0 2 0 i 0 0 0 1

With S+ := S1 + iS2 , S := S1 iS2 we find



0 1 0 0 0 0
S+ = 2~ 0 0 1 , S = 2~ 1 0 0. (3)
0 0 0 0 1 0

An example of a spin-1 particle is the photon. Let m, n be normalized vectors


in R3 which are orthogonal, i.e. mT n = 0. Find the eigenvalues of the 3 3
matrix
K = (m S)2 (n S)2
where m S = m1 S1 + m2 S2 + m3 S3 .
(ii) Show that
Pm = I3 (m S)2
is a projection operator.

Problem 28. Let A, B be 2 2 matrices over R. Find A, B such that

mink[A, B] I2 k
308 Problems and Solutions

where [ , ] denotes the commutator and for the norm k k consider the Frobenius
norm and max-norm.

Problem 29. Let A be an n n matrix over R. Assume that A1 exists. Let


u, v Rn , where u, v are considered as column vectors. (i) Show that if

vT A1 u = 1

then A + uvT is not invertible.


(ii) Assume that vT A1 u 6= 1. Show that

A1 uvT A1
(A + uvT )1 = A1 .
1 + vT A1 u

Problem 30. Can we find an invertible 2 2 matrix S over the real numbers
such that    
0 1 1 1 0
S S = ?
1 0 0 1

Problem 31. Let A and B be n n matrices over R. Assume that A 6= B,


A3 = B 3 and A2 B = B 2 A. Is A2 + B 2 invertible?

Problem 32. Let  


3 2
A=
4 3
and I2 be the 2 2 identity matrix. For j 1, let dj be the greatest common
divisor of the entries of Aj I2 . Show that

lim dj = .
j

Problem 33. Let a, d R and b C. Consider the hermitian matrix


 
a b
K= .
b d

Show that the matrix can be written as linear combination of the 2 2 identity
matrix and the Pauli spin matrices.

Problem 34. (i) Consider the polynomial

p(x) = x2 sx + d, s, d C.
Miscellaneous 309

Find a 2 2 matrix A such that its characteristic polynomial is p.


(ii) Consider the polynomial

q(x) = x3 + sx2 qx + d, s, q, d C.

Find a 3 3 matrix B such that its characteristic polynomial is q.

Problem 35. Let A, B be n n matrices over C. The matrix A is called


similar to the matrix B if there is a n n invertible matrix S such that

A = S 1 BS.

If A is similar to B, then B is also similar to A, since B = SAS 1 .


(i) Consider the two matrices
   
1 0 1 0
A= , B= .
2 1 0 1

Are the matrices similar?


(ii) Consider the two matrices
   
1 0 0 1
C= , D= .
0 1 1 0

Are the matrices similar?

Problem 36. (i) Consider the matrix



1/2 1/2 1/ 2
R = 1/2 1/2
1/ 2 .
1/ 2 1/ 2 0

Show that R1 = R = R. Use these properties and tr(R) to find all the
eigenvalues of R. Find the eigenvectors.
(ii) Let
0 1 0 0 0 1
A1 = 1 0 0 , A2 = 0 0 1 .
0 0 1 1 1 0
Calculate RA1 R1 and RA2 R1 . Discuss.

Problem 37. (i) Find the conditions on the 2 2 matrices over C such that

ABA = BAB.

Find solutions where AB 6= BA, i.e. [A, B] 6= 02 .


310 Problems and Solutions

(ii) Find the conditions on the 2 2 matrices A and B such that

A B A = B A B.

Find solutions where AB 6= BA, i.e. [A, B] 6= 02 .

Problem 38. Let A be an n n matrix over R. Assume that A1 exists. Let


u, v Rn , where u, v are considered as column vectors. (i) Show that if

vT A1 u = 1

then A + uvT is not invertible.


(ii) Assume that vT A1 u 6= 1. Show that

A1 uvT A1
(A + uvT )1 = A1 .
1 + vT A1 u

Problem 39. Let 1 , 2 , 3 be the Pauli spin matrices. Let

U (, , ) = ei3 /2 ei2 /2 ei3 /2

where , , are the three Euler angles with the range 0 < 2, 0
and 0 < 2. Show that
 i/2
cos(/2)ei/2 ei/2 sin(/2)ei/2

e
U (, , ) = . (1)
ei/2 sin(/2)ei/2 ei/2 cos(/2)ei/2

Problem 40. Let A and B be n n matrices over R. Assume that A 6= B,


A3 = B 3 and A2 B = B 2 A. Is A2 + B 2 invertible?

Problem 41. Let A, B be n n matrices over C. Assume that A and A + B


are invertible. Show that

(A + B)1 A1 A1 B(A + B)1 .

Apply the identity to A = 1 , B = 3 .

Problem 42. Let A be an n n matrix over R and let u be an n-vector in


Rn (column vector) with u 6= u. In numerical linear algebra we often have to
compute
2uuT
 
In T (1)
u u
where In is the n n identity matrix. Naively we would form the matrix (In
2uuT /uT u) from the vector u and then form the matrix product explicitly with
Miscellaneous 311

A. This would require O(m3 ) flops. Provide a faster computation for expression
(1).

Problem 43. Find all 2 2 matrices A1 , A2 , A3 such that

A1 A2 = A2 A3 , A3 A1 = A2 A3 .

Problem 44. Let A be an n n normal matrix with pairwise different eigen-


values. Are the matrices
n
Y A k In
Pj =
j k
k=1,j6=k

projection matrices?

Problem 45. Let n 2 and = exp(2i/n). Consider the diagonal and


permutation matrices, respectively

1 0 0 ... 0 0 1 0 ... 0

0 0 ... 0 0 0 1 ... 0
0 0 2 . . . 0 , P = ... ... ... ..

D= . . ..
.. .. .. ..

. . 0 0 0 ... 1
0 0 0 ... n1 1 0 0 ... 0

(i) Show that Dn = P n = In .


(ii) Show that the set of matrices

{ Dj P k : j, k = 0, 1, 2, . . . , n 1 }

form a basis of the vector space of n n matrices.


(iii) Show that
P D = DP, P j Dk = jk Dk P j .
(iv) Find the matrix

X = P + 1 P 1 + D + 1 D1

and calculate the eigenvalues.

Problem 46. Let z C. Consider the 2 2 matrices


       
1 z 1 z z 1 z 1
A= , B= , C= , D= .
z 1 z 1 1 z 1 z

Find the condition on z such that A, B, C, D are invertible.


312 Problems and Solutions

Problem 47. Let s = 1/2, 1, 3/2, 2, . . . be the spin. Let n = 2s + 1, i.e. for
s = 1/2 we have n = 2, for s = 1 we have n = 3 etc. Consider the n n matrix
Vs = (Vjk ) with
Vjk = exp(c(s j + 1)(s k + 1))
where j, k = 1, 2, . . . , n and c is a positive constant.
(i) Let s = 1/2, i.e. n = 2. Let
 
1 1 1
R1/2 = .
2 1 1
1
Find R1/2 V1/2 R1/2 .
(ii) Let s be positive integer with n = 2s + 1 and the n n matrix

1 1
.. .

. ..

1 0 1

1
Rs = 0 2 0


2 1 0 1



. . . .

. .
1 1
Find Rs Vs Rs .
(iii) Let s be 1/2, 3/2, . . . with n = 2s + 1 and the n n matrix

1 1
.. .

. ..

1
1 1

Rs = .

2 1 1

. . ..
. .
1 1
Find Rs Vs Rs .

Problem 48. Consider the Bell matrix


1 0 0 1

1 0 1 1 0
B= .
2 0 1 1 0
1 0 0 1
(i) Find all matrices A such that BAB = A.
(ii) Find all matrices A such that BAB is a diagonal matrix.

Problem 49. Let n 2. An invertible integer matrix, A GLn (Z), generates


a toral automorphism f : Tn Tn via the formula
f = A, : Rn Tn := Rn /Zn .
Miscellaneous 313

The set of fixed points of f is given by


#Fix(f ) := { x Tn : f (x ) = x }
Now we have: if det(In A) 6= 0, then
#Fix(f ) = | det(In A)|.
Let n = 2 and  
2 1
A= .
1 1
Show that det(I2 A) 6= 0 and find #Fix(f ).

Problem 50. Consider the symmetric matrix over R



2 1 1
A= 1 1 0 .
1 0 1
Find a invertible matrix B such that B 1 AB is a diagonal matrix.

Problem 51. Let V1 be a hermitian n n matrix. Let V2 be a positive


semidefinite n n matrix. Let k be a positive integer. Show that
tr((V2 V1 )k )
1/2 1/2
can be written as tr(V k ), where V := V2 V1 V2 .

Problem 52. Can one find a 2 2 unitary matrix such that


   
1 0 1 0 1
U U = .
0 1 1 0

Problem 53. Let 1 , 2 , 3 be the Pauli spin matrices. Let A, B be two


arbitrary 2 2 matrices. Is
3   
1 X 1 1
tr(AB) tr(j A) tr(j B) ?
2 j=1
2 2

Problem 54. In the following we count from (0, 0) to (n 1, n 1) for n n


matrices. Let := exp(2i/n). Consider the n n matrices
0 1 0 ... 0

1

0 0 1 ... 0
. .. . .
.

H= . . . ,
G= . ..
.

0 1
n1

1 0 0
314 Problems and Solutions

Then H n = Gn = In , HH = GG = In , HG = GH. Let U be the unitary


matrix
1
U = ( jk )
n
where j, k = 0, 1, . . . , n 1. Show that U HU 1 = G.

Problem 55. The standard simplex n is defined by the set in Rn


n
X
n := { (x1 , . . . , xn )T : xj 0, xj = 1 }.
j=1

Consider n affinely independent points B1 , . . . , Bn n . They span an (n 1)-


simplex denoted by = Con(B1 , . . . , Bn ), that is
n
X
= Con(B1 , . . . , Bn ) = { 1 B1 + + n Bn : j = 1, 1 , . . . , n 0 }.
j=1

The set corresponds to an invertible n n matrix whose columns are B1 , . . . ,


Bn . Conversely, consider the matrix C = (bjk ), where Ck = (b1k , . . . , bnk )T
(k = 1, . . . , n). If det(C) 6= 0 and the sum of the entries in each column is 1,
then the matrix C corresponds to an (n 1)-simplex Con(B1 , . . . , Bn ) in n .
Let C1 and C2 be n n matrices with nonegative entries and all the columns of
each matrix add up to 1.
(i) Show that C1 C2 and C2 C1 are also such matrices.
(ii) Are the n2 n2 matrices C1 C2 , C2 C1 such matrices?

Problem 56. Consider the matrices


   
0 1 0 i
A0 = , B0 = .
1 0 i 0

Let n = 0, 1, 2, . . .. Study the sequence of matrices

An+1 = An Bn , Bn+1 = An .

Discuss. Is the sequence of matrices periodic?

Problem 57. Consider the alphabet = {U, V, W }, axiom: = U and the


set of production rules

U 7 U V W, V 7 U V, W 7 U.

(i) Apply it to U = 1 , V = 2 , W = 3 and matrix multiplication. Is the


sequence periodic?
Miscellaneous 315

(ii) Apply it to U = 1 , V = 2 , W = 3 and the Kronecker product.

Problem 58. (i) Let A, B be n n matrices over C. Assume that the inverse
of A and A + B exists. Show that

(A + B)1 A1 A1 B(A + B)1 .

Apply the identity to


   
1 1 0 1 0 1
A= , B= .
2 0 1 2 1 0

Problem 59. Let M be an inertible n n matrix. Then we can form


1 1
M+ = (M + M 1 ), M = (M M 1 ).
2 2
Let  
1 1
M= .
0 1
Find M+ and M . M is a nonnormal matrix. Are the matrices M+ and M
normal?

Problem 60. Let A, B be n n matrices over C. The two n n matrices A


and B have a common eigenvector if and only if the n n matrix
n1
X
[Aj , B k ] [Aj , B k ]
j,k=1

is singular, i.e. the determinant is equal to 0.


(i) Apply the theorem to the matrices
   
0 1 1 1 1
A= , B= .
1 0 2 1 1

(ii) Apply the theorem to the matrices



1 0 0 cos sin 0
A = 0 cos sin , B = sin cos 0.
0 sin cos 0 0 1

(iii) Give a computer algebra implementation of this condition for two given
matrices.
316 Problems and Solutions

Problem 61. Let M be the Minkowski space endowed with the standard
coordinates x0 , x1 , x2 , x3 with the metric tensor field

g = dx0 dx0 + dx1 dx1 + dx2 dx2 + dx3 dx3

and the quadratic form

q(x) = (x0 )2 + (x1 )2 + (x2 )2 + (x3 )2 .

Let H(2) be the vector space of 2 2 hermitian matrices. Consider the map
: M H(2)  
x0 + x3 x1 ix2
(x) = X =
x1 + ix2 x0 x3
which is an isomorphism. Find the determinant of X. Find the Cayley transform
of X, i.e.
U = (X + iI2 )1 (X iI2 )
with the inverse
X = i(I2 + U )(I2 U )1 .

Problem 62. Consider the four-dimensional real vector space with a basis e0 ,
e1 , e2 , e3 . The Godel quaternion algebra G is defined by the non-commutative
multiplication

e0 ek = ek = ek e0 , (e0 )2 = e0
ej ek = (1)` jk` e` (1)k jk e0 , j, k, ` = 1, 2, 3.

Let q0 , q1 , q2 , q3 be arbitrary real numbers. We call the vector

q = qo e0 + q1 e1 + q2 e2 + q3 e3

a Godel quaternion. We define the basis


1 1
e11 := (e0 + e3 ), e22 := (e0 e3 )
2 2
1 1
e12 := (e1 + e2 ), e21 := (e1 e2 ).
2 2
Show that the ejk satisfy the multiplication law

ejk ers = kr ejs , j, k, r, s = 1, 2.

Show that every Godel quaternion q can be written as


2
X
q= qjk ejk .
j,k=1
Miscellaneous 317

Show that
       
1 0 0 1 0 1 1 0
e0 = , e1 = , e2 = , e3 =
0 1 1 0 1 0 0 1
is a matrix representation.

Problem 63. Let A, B be n n matrices over C. Assume that [A, B] 6= 0n .


Can we conclude that
[eA , eB ] 6= 0n ?

Problem 64. Consider the hermitian 3 3 matrix



0 1 1
A = 1 0 1.
1 1 0

Find A2 and A3 . We know that

tr(A) = 1 + 2 + 3 , tr(A2 ) = 21 + 22 + 23 , tr(A3 ) = 31 + 32 + 33 .

Use Newtons method to solve this system of three equations to find the eigen-
values of A.

Problem 65. The matrices g1 and g2 play a role for the matrix representation
of the braid group B4
1 t t1 t1

t 1 0
g1 = 0 1 0 , g = 1 t2 t1 0
1 1
0 1 t 1 t 0

as generators. Let g2 = g1 g 1 . Find the eigenvalues and eigenvectors of g1 , g2 .

Problem 66. (i) Given an m n matrix over R. Write a C++ program


that finds the maximum value in each row and then the minimum value of these
values.
(ii) Given an mn matrix over R. Write a C++ program that finds the minimum
value in each row and then the maximum value of these values.

Problem 67. Given an mn matrix over C. Find the elements with the largest
absolute values and store the entries (j, k) (j = 0, 1, . . . , m1) k = 0, 1, . . . , n1)
which contain the elements with the largest absolute value.

Problem 68. (i) Let j0 , j1 , k0 , k1 {0, 1}. Consider the tensor

Tkj00,k
,j1
1
.
318 Problems and Solutions

Give a 1 1 map that maps Tkj00,k


,j1
1
to a 22 22 matrix S = (s`0 ,`1 ) with `0 , `1 =
0, 1, 2, 3, i.e

t00 t00 t00 t00 s00 s01 s02 s03



00 01 10 11
t01 t01 t01 t01 s10 s11 s12 s13
00 01 10 11
7 .
t10
00 t10
01 t10
10 t10
11 s20 s21 s22 s23
t11
00 t11
01 t11
10 t11
11 s30 s31 s32 s33

(ii) Let j0 , j1 , j2 , k0 , k1 , k2 {0, 1}. Consider the tensor

Tkj00,k
,j1 ,j2
1 ,k2
.

Give a 1 1 map that maps Tkj00,k ,j1 ,j2


1 ,k2
to a 23 23 matrix S = (s`0 ,`1 ) with
3
`0 , `1 = 0, 1, . . . , 2 1.
(iii) Let n 2 and j0 , j1 , . . . , jn , k0 , k1 , . . . , kn {0, 1}. Consider the tensor
j ,j ,...,j
Tk00,k11 ,...,kn1
n1
.

j ,j ,...,j
(i) Give a 1 1 map that maps Tk00,k11 ,...,kn1
n1
to a 2n 2n matrix S = (s`0 ,`1 )
n
with `0 , `1 = 0, 1, . . . , 2 1.
(ii) Give a SymbolicC++ implementation. The user provides n.

Problem 69. Show that any rank one positive semidefinite n n matrix A
can be written as A = vvT , where v is some (column) vector in Cn .

Problem 70. Let A, B be n n matrices over C. Let v be a normalized


(column) vector in Cn . Let hAi := v Av and hBi := v Bv. We have the
identity

AB (A hAiIn )(B hBiIn ) + AhBi + BhAi hAihBiIn .

We approximate AB as

AB AhBi + BhAi hAihBiIn .

Let  
1 1
A = 1 , B = 2 , u= .
2 1
(i) Find AB and AhBi + BhAi hAihBiIn and the distance (Frobenius norm)
between the two matrices.
(ii) Apply the result to the case n = 2 and
 
1 1
A = 1 , B = 2 , v = .
2 1
Miscellaneous 319

Problem 71. The (n + 1) (n + 1) Hadamard matrix H(n) of any dimension


is generated recursively as follows
 
H(n 1) H(n 1)
H(n) =
H(n 1) H(n 1)

where n = 1, 2, . . . and H(0) = (1). Find H(1), H(2). and H(3).

Problem 72. Consider the symmetric 6 6 matrix over R

0 1 1 1 1 1

1 0 1 1 1 1
1 1 0 1 1 1

A=
1 1 1 0 1 1

1 1 1 1 0 1

1 1 1 1 1 0

This matrix plays a role in the construction of the icosahedron which is a reg-
ular polyhedron with 20 identical equilateral triangular faces, 30 edges and 12
vertices.
(i) Find the eigenvalues of this
matrix.
(ii) Consider the matrix
A + 5I6 . Find the eigenvalues.
(iii) the matrix
A + 5I6 induces an Euclidean structure on the quotient space
R6 /ker(A + 5I6 ). Find ker(A + 5I6 ).

Problem 73. Let A be an arbitrary n n matrix over R. Can we conclude


that A2 is positive semi-definite?

Problem 74. Show that the matrix


1 1 1 1

1 i 1 i
H=
1 1 1 1

1 i 1 i

as the product of two 4 4 matrices A and B such that each of these matrices
has precisely two non-zero entries in each row.
320 Problems and Solutions
Bibliography

Aldous J. M. and Wilson R. J.


Graphs and Applications: An Introductory Approach
Springer (2000)

Armstrong M. A.
Groups and Symmetry
Springer (1988)

Bredon G. E.
Introduction to Compact Transformation Groups
Academic Press (1972)

Bronson R.
Matrix Operations
Schaums Outlines, McGraw-Hill (1989)

Bump D.
Lie Groups
Springer (2000)

Campoamor-Stursberg R.
The structure of the invariants of perfect Lie algebras, J. Phys. A: Math.
Gen. 67096723 (2003)

Carter R. W.
Simple Groups of Lie Type
John Wiley (1972)

Chern S. S., Chen W. H. and Lam K. S.


Lectures on Differential Geometry
World Scientific (1999)

Cullen C. G.

321
322 Bibliography

Matrices and Linear Transformations


Second edition, Dover, 1990

DasGupta Ananda
American J. Phys. 64 14221427 (1996)

Davis P. J.
Circulant Matrices, John Wiley (1997)

de Souza P. N. and Silva J.-N.


Berkeley Problems in Mathematics
Springer (1998)

Dixmier J.
Enveloping Algebras
North-Holland (1974)

Englefield M. J.
Group Theory and the Coulomb Problem
Wiley-Interscience, New York (1972)

Erdmann K. and Wildon M.


Introduction to Lie Algebras,
Springer (2006)

Fuhrmann, P. A.
A Polynomial Approach to Linear Algebra
Springer (1996)

Fulton W. and Harris J.


Representation Theory
Springer (1991)

Gallian J. A.
Contemporary Abstract Algebra, Sixth edition
Houghton Mifflin (2006)

Golan J. S.
The Linear Algebra a Beginning Graduate Student Ougth to Know
Springer (2012)

Harville D. A.
Matrix Algebra from a Statisticians Perspective
Springer (1997)
Bibliography 323

Golub G. H. and Van Loan C. F.


Matrix Computations, Third Edition,
Johns Hopkins University Press (1996)

Gockeler, M. and Schucker T.


Differential geometry, gauge theories, and gravity
Cambridge University Press (1987)

Grossman S. I.
Elementary Linear Algebra, Third Edition
Wadsworth Publishing, Belmont (1987)

Hall B. C.
Lie Groups, Lie Algebras, and Representations: an elementary introduction
Springer (2003)

Helgason S.
Groups and Geometric Analysis, Integral Geometry, Invariant Differential Op-
erators and Spherical Functions
Academic Press (1984)

Horn R. A. and Johnson C. R.


Topics in Matrix Analysis
Cambridge University Press (1999)

Humphreys J. E.
Introduction to Lie Algebras and Representation Theory
Springer (1972)

Inui T., Tanabe Y. and Onodera Y.


Group Theory and its Applications in Physics
Springer (1990)

Isham C. J.
Modern Diffferential Geometry
World Scientific (1989)

James G. and Liebeck M.


Representations and Characters of Groups
second edition, Cambridge University Press (2001)

Johnson D. L.
Presentation of Groups
324 Bibliography

Cambridge University Press (1976)

Jones H. F.
Groups, Representations and Physics
Adam Hilger, Bristol (1990)

Kedlaya K. S., Poonen B. and Vakil R.


The William Lowell Putnam Mathematical Competition 19852000,
The Mathematical Association of America (2002)

Lang S.
Linear Algebra
Addison-Wesley, Reading (1968)

Lee Dong Hoon


The structure of complex Lie groups
Chapman and Hall/CRC (2002)

Miller W.
Symmetry Groups and Their Applications
Academic Press, New York (1972)

Ohtsuki T.
Quantum Invariants
World Scientific, Singapore (2002)

Schneider H. and Barker G. P.


Matrices and Linear Algebra
Dover Publications, New York (1989)

Seber G. A. F.
A Matrix Handbook for Statisticians
Wiley Series in Probability and Statistics (2008)

Steeb W.-H.
Matrix Calculus and Kronecker Product with Applications and C++ Programs
World Scientific Publishing, Singapore (1997)

Steeb W.-H.
Continuous Symmetries, Lie Algebras, Differential Equations and Computer Al-
gebra
World Scientific Publishing, Singapore (1996)

Steeb W.-H.
Bibliography 325

Hilbert Spaces, Wavelets, Generalized Functions and Quantum Mechanics


Kluwer Academic Publishers, Dordrecht (1998)

Steeb W.-H.
Problems and Solutions in Theoretical and Mathematical Physics,
Second Edition, Volume I: Introductory Level
World Scientific Publishing, Singapore (2003)

Steeb W.-H.
Problems and Solutions in Theoretical and Mathematical Physics,
Second Edition, Volume II: Advanced Level
World Scientific Publishing, Singapore (2003)

Steeb W.-H., Hardy Y., Hardy A. and Stoop R.


Problems and Solutions in Scientific Computing with C++ and Java Simula-
tions
World Scientific Publishing, Singapore (2004)

Sternberg S.
Lie Algebras 2004

Varadarajan V. S.
Lie Groups, Lie Algebras and Their Representations
Springer (2004)

Watkins D. S.
Fundamentals of matrix computations
Wiley

Wawrzynczyk A.
Group Representations and Special Functions
D. Reidel (1984)

Whittaker E. T.
A treatise on the analytical dynamics of particles and rigid bodies

Wybourne B. G.
Classical Groups for Physicists
John Wiley, New York (1974)
Index

Associator, 253 Farkas theorem, 32


Floquet theory, 169
Backward idenity matrix, 206 Fock space, 292
Baker-Campbell-Hausdorff formula, 140 Fourier transform, 145
Bell matrix, 312 Free group, 227, 265
Binary matrices, 213
Block form, 8 Godel quaternion, 316
Brad group, 256 Gaussian measure, 292
Braid group, 256, 260 Golden ration, 77
Braid relation, 183, 259 Golden-Thompson-Symanzik inequality,
Braid relations, 264 254
Braid-like relation, 258 Gordans theorem, 31
Brute force method, 105
Hadamard matrix, 7, 185, 319
Cartan matrix, 6 Hadamard product, 182, 185
Cauchy integral formula, 292 Hamiltonian, 301
Cauchy integral theorem, 145 Hankel matrix, 302
Cayley transform, 232 Heisenberg group, 223, 224, 247
Cayley-Hamilton theorem, 66, 165, 166 Hilbert-Schmidt norm, 47
Central difference scheme, 35 Hyperdeterminant, 63
Characteristic polynomial, 143, 144, 153, Hypermatrix, 63
162
Icosahedron, 78, 319
Chevalley basis, 245
Idempotent, 55
Commutator, 223, 247
Conformal transformation, 10 Jacobi method, 208
Cross-ratio, 10 Jacobian, 147
Jacobian matrix, 147
Daubechies wavelet, 301 Jordan algebra, 126
Diamond lattice, 1 Jordan identity, 305
Double angle formula, 209
Kleins inequality, 254
Entire function, 166
Equivalence relation, 34 Lagrange identity, 26
Euler angles, 133, 310 Lagrange interpolation, 142
Exterior product, 46 Laplace equation, 35

326
Index 327

Laplace transform, 144, 153 Spectral norm, 178


Laplacian, 180 Spectral theorem, 285
Legendre polynomials, 48 Spin-1, 307
Levi-Civita symbol, 52 Spinors, 287
Lie-Trotter formula, 159 Spiral basis, 21
Lieb inequality, 254 Splitting, 207
Logarithmic norm, 175 Standard basis, 1
Standard simplex, 17, 314
Moller wave operators, 194 Symplectic, 301
Monge-Ampere determinant, 50 Symplectic matrix, 82
Monodromy matrix, 169
Mutually unbiased, 290 Ternary commutator, 123
Ternutator, 123
Newton interpolation, 142 Tetrahedron, 3, 27, 40, 57
Nilpotent, 24, 66 Three-body problem, 34
Normal, 5, 23 Trace, 38
Normal matrix, 5 Trace norm, 47
Triangle, 11, 40
One-sided Jones pair, 183, 259 Tridiagonal matrix, 104
Orientation, 14 Trotter formula, 139
truncated Bose annihilation operator,
Pade approximation, 173, 208 121
Parallelepiped, 62 type-II matrix, 184
Pascal matrix, 75
Pencil, 72 Universal enveloping algebra, 245
Permanent, 61, 109
Pfaffian, 48 Vector product, 25, 253
Polar decomposition, 133, 136
Potts model, 76 Wronskian, 60
Potts quantum chain, 193
Power method, 209

Quadratic form, 97
Quotient space, 34

Reproducing kernel, 293


Resolvent, 77, 143, 144, 153, 163
Resolvent equation, 77
Resultant, 303
Root vector, 252
Rotation matrix, 133

Schur decomposition, 173


Schur invertible, 184
Similar, 23, 309

You might also like